医学临床三基训练医师分册题库第四版版

上传人:枫** 文档编号:578076255 上传时间:2024-08-23 格式:PDF 页数:194 大小:34.05MB
返回 下载 相关 举报
医学临床三基训练医师分册题库第四版版_第1页
第1页 / 共194页
医学临床三基训练医师分册题库第四版版_第2页
第2页 / 共194页
医学临床三基训练医师分册题库第四版版_第3页
第3页 / 共194页
医学临床三基训练医师分册题库第四版版_第4页
第4页 / 共194页
医学临床三基训练医师分册题库第四版版_第5页
第5页 / 共194页
点击查看更多>>
资源描述

《医学临床三基训练医师分册题库第四版版》由会员分享,可在线阅读,更多相关《医学临床三基训练医师分册题库第四版版(194页珍藏版)》请在金锄头文库上搜索。

1、新版医生三基题库一、( 共74 2 分) 题下选项可能多个正确,只能选择其中最佳的一项1 、颗区外伤引起急性硬膜外血肿,最常见损伤的血管是A、颗浅动脉 B 、大脑中动脉 C 、脑膜中动脉 D 、板障静脉 E 、乙状窦答案:C2 、正骨外科颈骨折损伤腋神经后,肩关节将出现哪些运动障碍内收 D 、A、不能屈 B 、不能伸 C 、不能内收 D 、不能外展 E 、不能旋转答案:D3 、氢化可的松的主要作用是A 、 降低血糖 B 、 减少嗜酸性粒细胞和淋巴细胞 C 、 减少体内水的排出 D 、减少血小板和红细胞 E 、激活儿茶酚氧位甲基转移前答案:B4 、同时影响肾小球滤过和肾小管重吸收的因素是A 、血

2、浆胶体渗透压 B 、滤过膜的通过性 C 、血液中葡萄糖 D 、抗利尿激素 E 、醛固酮答案:A5 、某人的红细胞与B 型血血清凝集,而其血清与B 型血的红细胞不凝集,此人血型为A 、A 型 B 、B 型 C 、0 型 D 、A B 型 E 、R h 型答案:D6 、X肌不会产生强直收缩,其原因是A 、心脏是功能上的合胞体 B 、心肌肌浆网不发达,储C a” 少 C 、心肌有自律性,呈自动节律收缩 D 、心肌的有效不应期长 E 、心肌呈全或无收缩答案:D7、肾素- 血管紧张素系统活动增强时A 、 醛固酮释放减少 B 、 静脉回心血量减少 C 、 体循环平均充盈压减低 D 、交感神经末梢释放递质减

3、少 E 、肾脏排钠量减少答案:E8、最重要的消化液是A、唾液 B 、胃液 C 、胆汁 D 、胰液 E 、肠液答案:D9 、大量饮清水后尿量增多,主要是由于A 、肾小球滤过率增加 B 、血浆胶体渗透压降低 C 、抗利尿激素分泌减少D 、醛固酮分泌减少 E 、囊内压降低答案:C1 0 、切除狗的肾上腺皮质,出现A 、血容量I ,血 钠 I ,尿 钾 f B 、血容量I ,血 钠 t ,尿 钾 f C 、血容量I ,血 钠 f ,尿 钾 I D 、血容量t ,血 钠 I ,尿 钾 f E 、血容量I ,血 钠 I ,尿钾 I新版医生三基题库第1 页答案:E1 1 、基础体温随月经周期变化,与何激素有

4、关A、甲状腺激素 B 、孕激素 C 、雌激素 D 、催乳素 E 、A C TH答案:B1 2 、机体保钠的主要激素是A、醛固酮 B 、氢化可的松 C 、A C TH D 、生长素 E 、A D H答案:A1 3 、下列哪种属于类固醇激素A 、1 , 2 5 - 1 ) 4 引、氢化可的松 C 、卵泡刺激素 D 、甲状旁腺激素 E 、促甲状腺激素答案:B1 4 、决定血浆胶体渗透压的主要物质是A、球蛋白 B 、脂蛋白 C 、糖蛋白 D 、补体 E 、清蛋白答案:E1 5 、用已知B 型人的血液与待测者血液做交叉合血,若主反应凝集,次反应不凝集,待测者的血型是A、B 型 B 、0 型 C 、A 型

5、 D 、A B 型 E 、Rh 阴性答案:B1 6、心室肌的前负荷是指A 、右心房压力 B 、射血期心室内压 C 、心室舒张末期压 D 、大动脉血压 E 、等容收缩期心室内压答案:C1 7、人体安静状态下,哪种器官的动脉血和静脉血含氧量差值最大A、脑 B 、肾脏 C 、心脏 D 、骨骼肌 E 、肝脏答案:C1 8、下述钾的生理功能中,哪项是错的A、参与细胞内糖和蛋白质的代谢 B 、高钾使神经肌肉兴奋性降低 C 、参与静息电位的形成 D 、高钾抑制心肌收缩 E 、维持细胞内的渗透压答案:B1 9、胃泌素的生理作用中,哪项是错的A 、刺激胃粘膜细胞增殖 B 、 刺激胃粘膜细胞分泌盐酸与胃蛋白酶原

6、C 、刺激胃窦与肠运动 D 、刺激胰液、胆汁分泌 E 、刺激幽门括约肌收缩答案:E2 0 、衡量组织兴奋性的指标是A、动作电位 B 、阈电位 C 、肌肉收缩或腺体分泌 I ) 、阈强度 E 、静息电位答案:D2 1 、使重症肌无力病人的肌肉活动恢复正常可给予A 、箭毒 B 、阿托品 C 、新斯的明 D 、 a - 银环蛇毒 E 、甘氨酸答案:C2 2 、机体的内环境是指新版医生三基题库第2 页A、血液 B、细胞内液 C、组织液 D、脑脊液 E、细胞外液答案:E23、下列哪项不是评定心功能的指标A、心指数 B、射血分数 C、心排血量 D、循环血量 E、每搏功答案:D24、下列哪项是左心室的后负荷

7、A、快速射血期心室内压 B、减慢射血期心室内压 C、快速充盈期心室内压D、等容收缩期心室内压 E、主动脉压答案:E25、甘露醇利尿的基本原理是A、肾小球滤过率增加 B、肾小管分泌减少 C、渗透性利尿 D、水利尿E、增加清除率答案:C26、对脂肪和蛋白质消化作用最强的消化液是A、胃液 B、胆汁 C、胰液 D、小肠液 E、唾液答案:C27、心脏正常起搏点位于A、窦房结 B、 心房 C、 房室交界区 D、 心室末梢浦肯野纤维网 E、心室答案:A28、下列哪项可引起心率减慢A、交感神经活动增强 B、迷走神经活动增强 C、肾上腺素 1 )、甲状腺激素 E、发热答案:B29、质粒是A、染色体外的遗传物质,

8、存在于核质中 B、染色体外的遗传物质,存在于胞浆中C、细菌的一种特殊结构 D、细菌的基本结构,存在于核质中 E、细菌生命活动所必需的物质答案:B30、关于外毒素的叙述,下列哪项是错误的A、是活菌释放至菌体外的一种蛋白质 B、主要由革兰阳性菌产生,少数革兰阴性菌也能产生 C、性质稳定,耐热 D、毒性强,引起特殊病变 E、抗原性强答案:C31、病原菌侵入血流并在其中大量繁殖,造成机体严重损伤,引起严重的症状称为A、毒血症 B、菌血症 C、败血症 D、脓毒血症 E、病毒血症答案:C32、能在无生命培养基上生长的最小微生物是A、细菌 B、真菌 C、衣原体 D、支原体 E、立克次体答案:D33、免疫系统

9、包括新版医生三基题库第3页A、胸腺、骨髓 B 、T 细胞、B 细胞 C 、免疫器官、免疫细胞 D 、免疫器官、免疫分子答案:EE 、免疫组织、免疫器官、免疫细胞、免疫分子3 4 、在同种不同个体组织和细胞中存在的不同抗原被认为是A 、同种异型抗原 B 、 异种抗原共同抗原答案:A3 5 、在人血清中含量最高的I g 是C 、 异嗜性抗原D 、 相容性抗原E 、A、 I g M B 、 I g A C 、 I g E D 、 I g G答案:D3 6 、下述细菌编组中,哪一组细菌可引起食物中毒A 、蜡样芽胞杆菌、变形杆菌、金黄色葡萄球菌寒沙门菌 C 、鼠伤寒沙门菌、破伤风杆菌E 、副溶血弧菌、布

10、氏杆菌E 、I g DB 、肉毒杆菌、结核分枝杆菌、伤D 、产气荚膜杆菌、肺炎链球菌答案:A3 7 、下述哪种结构是病毒体A、壳粒 B 、衣壳 C 、核衣壳 D 、包膜 E 、核酸答案:C3 8 、甲型流感病毒中最易发生变异的结构是A、衣壳抗原 B 、核蛋白 C 、膜蛋白 I ) 、H A 、N A E 、核酸答案:D3 9 、B 细胞能识别特异性抗原,是因其表面有A、F c 受体 B 、C 3 b受体 C 、E 受体 D 、S mlg E 、S I g A答案:D4 0 、化验结果:H B s A g G ) 、H B e A g ( + ) 、抗- H B c ( + ) 、抗H B e

11、( - ) 、抗- H B s ( - ) , 该病人为A 、乙型肝炎病毒感染潜伏期 B 、急性乙型肝炎 C 、乙型肝炎恢复期 D 、急性甲型肝炎 E 、乙肝疫苗接种后的反应答案:B4 1 、关于流脑的叙述,下列哪一项是错误的A 、主要致病因素为内毒素 B 、主要通过飞沫传播 C 、人为唯一的传染源D 、暴发型以儿童罹患为主 E 、9 5 % 以上由B 群脑膜炎球菌引起答案:E4 2 、关于补体的生物学活性,下列哪一项是错误的A 、 具有溶菌、 杀菌作用 B 、 具有免疫调理作用 C 、 具有免疫粘附作用 D 、具有趋化功能 E 、能促进抗体大量合成答案:E4 3 、杀灭细菌芽胞最有效的方法是

12、A、煮沸法 B 、巴氏消毒法 C 、 高压蒸汽灭菌法D 、 紫外线照射 E 、9 0 % 乙醇消毒答案:C4 4 、担负体液免疫功能的细胞是新版医生三基题库第4 页A、T 细胞 B 、K 细胞 C 、B 细胞 D 、N K 细胞 E 、巨噬细胞答案:C4 5 、下列抗原与抗体中,哪种一般不能从血标本检测到A、H B s A g B 、H B e A g C 、H B c A g I ) 、抗- H B s E 、抗- H B c答案:C4 6 、免疫活性细胞包括A 、T 细胞 B 、K 细胞、N K 细胞 C 、T 和B 淋巴细胞 D 、B 淋巴细胞 E 、T 和B 淋巴细胞、吞噬细胞答案:C

13、47 、关于I g G 的叙述,下列哪项是错误的A、是一种球蛋白 B、能通过胎盘 C、血清中含量最多 D、I g G i 、I g G 、I g G ” 的Fc段能与S PA 结合 E、其作用与抗体完全一样答 案 : E48 、能通过胎盘的I g 是A、 I g G B、 I g M C、 I g A D, I g D E、 S I g A答案:A49、下列哪种物质参与A DCC效应A、I g G B、I g M C、巨噬细胞 D、T 细胞 E、S m l g答案:A50、青霉素过敏性休克是属于A、I 型超敏反应 B、I I 型超敏反应 C、山型超敏反应 D、I V 型超敏反应 E、免疫耐受答

14、案:A51、OT 试验原理是A 、迟发型超敏反应 B、速发型超敏反应 C、I V 型超敏反应在局部的表现D、 I 型超敏反应在局部的表现 E、免疫排斥反应答案:C52 、接种BCG 的作用是A 、增强机体非特异性免疫能力 B、增强人体细胞免疫能力 C、增强人体体液免疫能力 D、增强机体免疫稳定功能 E、使人体对结核分枝杆菌产生免疫力答案:E53 、注射破伤风抗毒素( T A T ) 的作用是A 、中和白喉外毒素 B、中和破伤风外毒素 C、中和所有的外毒素 D、中和病毒 E、刺激人体产生抗毒素答案:B54、对热抵抗力最强的病毒是A、甲型肝炎病毒 B、乙型肝炎病毒 C、 狂犬病毒 D、 艾滋病毒

15、E、麻疹病毒答案:B55、被狂犬咬伤的伤口最好采用A、弱酸冲洗 B、2 0 % 肥皂水冲洗 C、过氧化氢溶液冲洗 D、食醋冲洗新版医生三基题库第5页E、9 0 % 乙醇冲洗答案:B56、现认为仅含有蛋白成分、不含核酸的病毒是A、类病毒 B、拟病毒 C、肮粒 D、缺陷病毒 E、小DNA 病毒答案:C57 、新生儿抗感染的主要抗体是A 、 I g G B、 I g M C、 I g A D、 I g D E、 I g E答案:B58 、担负细胞免疫功能的细胞是A、T 细胞 B、K 细胞 C、B细胞 D、NK 细胞 E、巨噬细胞答案:A59、哪一类水、电解质紊乱最容易发生低血容量性休克A、低渗性脱水

16、 B、高渗性脱水 C、等渗性脱水 D、水中毒 E、低钾血症答案:A60、4 性肾小球肾炎产生全身性水肿的主要机制是A 、醛固酮分泌增加 B、抗利尿素释放增多 C、肾小球钠水滤过下降 D、肾小球毛细血管通透性升高 E、血浆胶体渗透压减低答案:C61、 某溃疡病并发幽门梗阻病人, 因反复呕吐入院, 血气分析结果为: p H 7 . 49 , P a C O 2 48 m m H g ,H C O 3 3 6 m m o l / L , 该病人应诊断为A、呼吸性碱中毒 B 、呼吸性酸中毒 C 、代谢性酸中毒 D 、代谢性碱中毒 E 、混合性酸碱中毒答案:D6 2 、氧疗对哪型缺氧效果最好A、血液性缺

17、氧 B 、 低张性缺氧 C 、 循环性缺氧 D 、 组织性缺氧 E 、混合性缺氧答案:B6 3 、机体发热时常出现A、低渗性脱水 B 、等渗性脱水 C 、高渗性脱水 D 、水中毒 E 、水肿答案:C6 4、D I C 最主要的病理特征是A、大量微血栓形成 B 、凝血功能失常 C 、纤溶过程亢进 D 、凝血物质大量消耗 E 、溶血性贫血答案:B6 5 、休克早期组织微循环灌流的特点是A、少灌少流,灌少于流 B 、少灌多流,灌少于流 C 、少灌少流,灌多于流D 、多灌少流,灌多于流 E 、多灌多流,灌少于流答案:A6 6 、下列哪项最符合心力衰竭的概念新版医生三基题库第6 页A、心脏每搏输出量降低

18、 B 、静脉回流量超过心排血量 C 、心功能障碍引起大小循环充血 D 、心脏负荷过度引起心功能障碍 E 、心排血量不能满足机体的需要答案:E6 7 、慢性呼吸衰竭并发右心衰的主要机制是A、 外周血管扩张、 阻力降低, 静脉回流量增加 B 、 慢性缺氧后血容量增多 C 、红细胞数目增多,血液粘滞性增高 D 、 肺部病变,肺毛细血管床大量破坏 E 、肺泡缺氧和C 0 ? 潴留引起肺小动脉收缩答案:E6 8 、肝性脑病的正确概念应是A、肝脏疾病并发脑部疾病 B 、肝功能衰竭并发脑水肿 C 、肝功能衰竭所致的昏迷 D 、肝功能衰竭所致的精神紊乱性疾病 E 、严重肝病所致的神经精神综合征答案:E6 9

19、、阻塞性黄疸( 早期) 临床生化测定的特点是A、血清中酯型胆红素含量升高 B 、尿中无尿胆红素 C 、粪中粪胆素原升高D 、尿中尿胆素原升高 E 、尿中尿胆索升高答案:A7 0 、下述哪项最符合急性肾衰竭的概念A 、肾脏内分泌功能急剧障碍 B 、肾脏泌尿功能急剧障碍 C 、肾脏排泄废物能力急剧降低 D 、肾脏排酸保碱能力急剧降低 E 、肾脏浓缩稀释功能降低答案:B7 1 、慢性肾衰竭病人尿量的变化特点是A 、早期多尿,晚期夜尿 B 、早期少尿,晚期多尿 C 、早期多尿、夜尿,晚期少尿 D 、早期夜尿,晚期多尿 E 、早期多尿、血尿,晚期少尿答案:C7 2 、高渗性脱水病人血浆渗透压应A、2 5

20、 0 m 0 s m / L B 、2 7 0 m 0 s m / L C 、2 9 0 m 0 s m / L D 、3 1 0 m 0 s m / LE 3 3 0 m o s m / L答案:D7 3 、输入大量库存过久的血液易导致A、高钠血症 B 、低钠血症 C 、低钾血症 D 、高钾血症 E 、低镁血症答案:D7 4 、对D IC 发病机制的描述下列哪项是不正确的A 、血管内皮细胞损伤,激活凝血因子X I B 、组织严重破坏,大量组织因子进入血液 C 、血细胞大量破坏,释放促凝血物质 D 、羊水进入血液,激活内源性凝血系统 E 、癌细胞进入血液,激活内源性凝血系统答案:A7 5 、有

21、关肝细胞性黄疸的描述下列哪项是错误的A 、血清中酯型胆红素增多 B 、血清中非酯型胆红素增多 C 、肠内粪胆原形成减少 D 、尿中尿胆原排出减少 E 、尿中出现胆红素答案:D7 6 、血液缓冲系统中最重要的是A 、血浆蛋白缓冲系统 B 、磷酸盐缓冲系统 C 、碳酸氢盐缓冲系统 D 、血红蛋白缓冲系统 E 、氧合血红蛋白缓冲系统答案:C7 7 、某肾疾患病人,血气分析结果: p I1 7 . 3 2 , P aC O2 3 0 m m Hg , HC 03- 1 5 m m o l / L , 应诊断为A 、呼吸性碱中毒 B 、呼吸性酸中毒 C 、代谢性碱中毒 D 、代谢性酸中毒 E 、混合性酸

22、碱紊乱答案:D7 8 、某病人血气分析测定A G 3 0 , 说明该病人有A 、代谢性碱中毒 B 、代谢性酸中毒 C 、呼吸性碱中毒 D 、高氯血症E 、高钾血症答案:B7 9 、急性肾衰竭少尿期病人最危险的变化是A、水中毒 B 、高钾血症 C 、少尿 D 、代谢性酸中毒 E 、氮质血症答案:B8 0 、短期内大量丢失小肠液常首先出现A、高渗性脱水 B 、低渗性脱水 C 、等渗性脱水 D 、低钠血症 E 、高钾血答案:C8 1 、肾性水肿首先出现的问题是A、上肢 B 、下肢 C 、腹腔 D 、眼睑 E 、下垂部位答案:D8 2 、下列哪种氨基酸不含极性侧链A、 C ys B 、 T h r C

23、 、 Le u D 、 S e r E 、 T yr答案:C8 3 、多肽链中一级结构的基本单位是A 、- NC C N N C C N N C C N - B 、- C HN O C HN O C HN O - C 、- C O N HC O N I I C O N H- D 、- C N O HC N O HC N O H- E 、- C N HO C N HO C N HQ -答案:C8 4 、下列关于蛋白质四级结构的叙述哪项是错误的A 、由两个或两个以上的亚基以非共价键结合而成 B 、亚基可相同,也可不相同C 、只有一条多肽链的蛋白质无四级结构 D 、胰岛素不具有四级结构 E 、天然蛋

24、白质只具有四级结构答案:E8 5 、蛋白质变性是由于A 、蛋白质一级结构的改变 B 、蛋白质颗粒聚集 C 、蛋白质空间构象的破坏D 、辅基的脱落 E 、蛋白质水解答案:C8 6 、D N A 携带生物遗传信息这一事实意味着什么新版医生三基题库第7 页A 、不论哪一物种的碱基组成均应相同 B 、病毒的侵袭是靠蛋白质转移到宿主细胞而实现 C 、 D N A 的碱基组成随机体年龄和营养状况而改变 1 ) 、同一生物不同组织的D A , 其碱基组成相同 E 、D N A 以小环状结构物存在答案:D8 7 、W a t s o n - C ri c k 的D N A 结构模型A、是一个三链结构 B 、D

25、 N A 双股链的走向呈反向平行 C 、碱基之间以共价结合 D 、碱基A - U 配对 E 、磷酸戊糖主链位于D N A 螺旋内侧答案:B8 8 、t R N A 的分子结构特点是A 、有反密码环和3 - 端C C A - 0 H结构 B 、有反密码环和5 - 端多聚A 尾 C 、有密码环和D HU 环 D 、5 端有Tw环 E 、3端有帽式结构答案:A8 9 、酶的竞争性抑制作用具有下列哪一种动力学效应A、心值降低,V 皿不变 B 、K ” 值不变,心 不变 C 、K . 值降低,V 皿降低 D 、冗值增大,L. 不变 E 、K ” 值不变,V . 增大答案:D9 0 、1 g 分子葡萄糖的

26、有氧氧化净生成的A T P 分子数与无氧氧化净生成的A T P 分子数最接近的比例为下列哪一组A、 2 5 : 1 B 、 1 8 : 1 C 、 1 2 : 1 D 、 9 : 1 E 、 3 : 1答案:B9 1 、磷酸戊糖途径的生理意义是A、提供N A D P H和磷酸戊糖 B 、提供N A D P H和七碳糖 C 、提供N A D P H和能量D 、提供N A D P H ,和磷酸戊糖 E 、提供6 - 磷酸果糖和3 - 磷酸甘油醛答案:A9 2 、乳酸循环不经过下列哪条途径A 、 肌糖原酵解 B 、 肝糖原更新 C 、 磷酸戊糖途径 D 、 肝糖原异生 E 、肝糖原分解成血糖答案:C

27、9 3 、下列哪项代谢过程的细胞内定位不正确A 、酮体合成:线粒体 B 、胆固醇合成:胞液和内质网 C 、脂肪酸活化:胞液D 、三酰甘油:肝内质网 E 、 B - 氧化:细胞膜受体答案:E9 4 、负责转运内源性三酰甘油的血浆脂蛋白是A、 L D L B 、 VL D L C 、 C M D 、 H D 1 E 、 L p ( a)答案:B9 5 、下列有关脂肪酸氧化的叙述中哪项是错误的A、首先脂肪酸活化,生成脂酰C o A B 、经肉毒碱进入线粒体内 C 、在 碳 上进行脱氧、加水、再脱氧,并在a -B碳之间断裂 D 、其产物只有脂酰C o A E 、肉毒碱脂酰转移酶I 是脂酸B氧化的限速酶

28、答案:D9 6 、在尿素合成中下列哪项反应需要A T P 参加新版医生三基题库第8 页A、 精氨酸f 鸟氨酸+ 尿素 B、 草酰乙酸+ G lu -A sp + a-酮戊二酸 C、 瓜氨酸+Aspf 精氨酸代琥珀酸 D、延胡索酸一苹果酸 E、以上4项反应都不需要ATP答案:C97、下列哪一个不是一碳单位A、 -CHjB、 -CHO C、 -CH=NH D、 -CH= E、 C02答案:E98、呼吸链中属于脂溶性成分的是A、CoQ B、FMN C、NADD、Cyt C E、铁硫蛋白答案:A99、以下哪条呼吸链是正确的A、NADHFAD(Fe-S) -CoQ一C y tb fc L C f aa

29、3 f OIkNADHfCoQC y tb fc L C f aa3 f OzC、NADH-FAD(Fe-S) C o Q - C y tc c b a a j- 02D NADH-FMN (Fe-S) - C o Q - C y tb - c cf a a 3 f 0正、F A D HLF M N (Fe-S) - CoQC y tc L c baa3 f O 2答案:D100、下列不符合基因的论述是A、基因是遗传信息的功能单位 B、正常情况下,所有细胞内的基因均处于活性状态 C、 含外显子与内含子 D、 基因是具有特定核甘酸顺序的DNA片段 E、遗传学上,基因是决定或编码某种蛋白质的DNA

30、片段答案:B101、经典的分子遗传学中心法则是翻译 转录A、DNA- RNA-* 蛋白质反转录 翻译B、DNA- RNA- 蛋白质转录 翻译C、DNA-* RNA-* 蛋白质复制 转录D、DNA- RNA蛋白质反转录 转录E、DNA- RNA- 蛋白质答 案 :C102、下列关于真核细胞DNA复制的叙述,哪项是错误的A、半保留复制 B、a和3 -DNA聚合酶起主要作用 C、需要解链酶和拓扑异构酶参与 D、出现复制叉 E、合成的随从链可以顺着解链的方向连续延长答案:E103、合成DNA的原料是A、dATP dGTP dCTP dTTP B、dAMP dGMP dCMP dTMP C、dADP d

31、GDP dCDP dTDPD、 ATP GDP CDP TDP E、 AMP GMP CMP UMP答案:A新版医生三基题库第9页104、受体拮抗药的特点是A、无亲和力,无内在活性 B、有亲和力,有内在活性 C、有亲和力,有较弱的内在活性 D、有亲和力,无内在活性 E、无亲和力,有内在活性答案:D105、药物的灭活和消除速度决定了A、起效的快慢 B、作用持续时间 C、最大效应 D、后遗效应的大小E、不良反应的大小答案:B106、阿托品禁用于A、青光眼 B、感染性休克 C、有机磷中毒 D、肠痉挛 E、虹膜睫状体炎答案:A107、用氯丙嗪治疗精神病时最常见的不良反应是A、 体位性低血压 B、 过敏

32、反应 C、 内分泌障碍 D、 消化系统症状 E、锥体外系反应答案:E108、强心首降低心房纤颤病人的心室率的机制是A、 降低心室肌自律性 B、 改善心肌缺血状态 C、 降低心房肌的自律性 D、降低房室结中的隐匿性传导 E、增加房室结中的隐匿性传导答案:E109、可降低双香豆素抗凝作用的药物是A、广谱抗生素 B、阿司匹林 C、苯巴比妥 I)、氯贝丁酯 E、保泰松答案:C110、在氯霉素的下列不良反应中哪种与它抑制蛋白质的合成有关A、二重感染 B、灰婴综合征 C、皮疹等过敏反应 D、再生障碍性贫血E、消化道反应答案:D1 1 K抗癌药最常见的严重不良反应是A、肝脏损害 B、神经毒性 C、胃肠道反应

33、 D、抑制骨髓 E、脱发答案:D112、下列药物中成瘾性极小的是A、吗啡 B、喷他佐辛 C、哌替嘘 D、可待因 E、阿法罗定答案:B113、糖皮质激素与抗生素合用治疗严重感染的目的是A、增强抗生素的抗菌作用 B、增强机体防御能力 C、拮抗抗生素的某些不良反应 D、通过激素的作用缓解症状,度过危险期 E、增强机体应激性答案:D114、治疗沙眼衣原体感染应选用A、四环素 B、青霉素 C、链霉素 D、庆大霉素 E、干扰素答案:A新版医生三基题库第10页1 1 5 、可诱发心绞痛的降压药是A 、肿屈嗪 B 、拉贝洛尔 C 、可乐定 D 、哌哇嗪 E 、普奈洛尔答案:A1 1 6 、阿托品不具有的作用是

34、A 、松弛睫状肌 B 、松弛瞳孔括约肌 C 、调节麻痹,视近物不清 D 、降低眼内压 E 、瞳孔散大答案:D1 1 7 、水杨酸类解热镇痛药的作用特点不包括A 、能降低发热者的体温 B 、有较强的抗炎作用 C 、有较强的抗风湿作用D 、对胃肠绞痛有效 E 、久用无成瘾性和耐受性答案:D1 1 8 、可乐定的降压机制是A、阻断中枢咪哇琳受体 B 、激动中枢a 2 受体 C 、阻断外周a 受体 D 、阻断射受体 E 、耗竭神经末梢去甲肾上腺素答案:B1 1 9、流行性脑脊髓膜炎首选A、 S M Z B 、 S A C 、 S I Z D 、 S M D E 、 S D答案:E1 2 0 、氨基甘类

35、抗生素不具有的不良反应是A、耳毒性 B 、肾毒性 C 、过敏反应 D 、胃肠道反应 E 、神经肌肉阻断作用答案:D1 2 1 、博来霉素适宜用于A、腮腺癌 B 、肝癌 C 、骨肉瘤 D 、皮肤癌 E 、急性淋巴性白血病答案:D1 2 2 、气体、蒸气和气溶胶形态的环境污染物进入体内途径主要是A 、皮肤 B 、呼吸道 C 、消化道 D 、汗腺 E 、皮脂腺答案:B1 2 3 、可引起温室效应的主要物质是A、S 02 B 、C O C 、C 02 D 、N O , E 、N O答案:C1 2 4 、下列物质中易经完整皮肤吸收的是A、苯、 苯胺、甲苯 B 、二甲苯、二氧化硫 C 、 苯胺、有机磷、鼠

36、氢酸 D 、有机磷、对苯二胺、铅 E 、对苯二胺、甲基汞、氮氧化物答案:C1 2 5 、致癌因素中,最多见的是A 、化学因素 B 、心理因素 C 、物理因素 D 、生物因素 E 、社会因素答案:A1 2 6 、毒物在体内的蓄积量主要受哪些因素的影响A 、毒性、摄入量、生物半减期 B 、摄入量、生物半减期和作用时间 C 、理化新版医生三基题库 第1 1 页特性、毒性、摄入量 D 、摄入量、理化特性、作用时间 E 、蓄积部位、摄入量、生物半减期答案:B1 2 7 、形成酸雨的主要污染物是A 、C O ? 和N O * B 、C O 2 和O 3 C 、N O ? 和C O D 、H C 和C F

37、C E、N O * 和S O ?答案:E1 2 8 1 天然食物中蛋白质生物学价值最高的是A、瘦猪肉 B 、鸡蛋 C 、牛奶 D、鱼 E、黄豆制品答案:E1 2 9 , 按照目前我国膳食习惯,膳食中蛋白质的主要来源是A 、肉、鱼、禽类 B 、豆类及豆制品 C 、蛋、奶类 D、粮谷类 E、薯类答案:D1 3 0 、以下不属于膳食纤维的是A 、纤维素 B 、果胶 C 、半纤维素 D、藻类多糖 E、果糖答案:E1 3 1 、有利于肠道钙吸收的因素有A 、蛋白质、乳糖、维生素D B 、脂肪酸、氨基酸、乳糖 C 、抗酸药、乳糖、钙磷比 D、植酸、蛋白质、抗酸药 E、草酸、维生素D、乳糖答案:A1 3 2

38、 、下列哪一种食物中所含烟酸为结合型,不能被人体吸收利用A、大米 B 、小麦 C 、玉米 D、小米 E、高粱答案:C1 3 3 、引起沙门菌食物中毒的主要食物是A 、 蔬菜、 水果 B 、 豆类及其制品 C 、 谷类 D、 肉类、 奶类及其制品 E、海产品答案:D1 3 4 、引起副溶血性弧菌食物中毒的主要食物是A 、罐头食品 B 、海产品及盐渍食品 C 、奶及奶制品 D、家庭自制豆制品 E、剩米饭、凉糕答案:B1 3 5 、河豚中河豚毒素含量最高的器官或组织是A、皮肤、腮 B 、肝脏、心脏 C 、卵巢、肝脏 D、腮、鳍 E、血液、肝脏答案:C1 3 6 、急性亚硝酸盐食物中毒的解毒药物是A

39、、 小剂量美蓝 B 、 二疏基丙醇 C 、 亚硝酸异戊酯 D、 硫代硫酸钠 E、依地酸二钠钙答案:A1 3 7 、体内合成亚硝基化合物的主要部位是A 、肝脏 B 、胃 C 、肾脏 D、小肠 E、骨髓答案:B1 3 8 、能阻断体内N -亚硝基化合物合成的维生素是A、维生素氏 B 、维生素D C 、维生素C D、维生素A E、维生素B ?答案:C1 3 9 、维生素& 缺乏可引起A、脚气病 B 、佝偻病 C 、坏血病 D、癞皮病 E、口角炎答案:A1 4 0 、关于职业病的特点,下列说法错误的是A、病因明确 B 、存在剂量-反应关系 C 、病因大多数可定量测定 D、凡是接触者均可患病 E、病变早

40、期处理预后较好答案:D1 41 、铅对机体损害的主要早期变化之一是A 、小血管痉挛 B 、叶琳代谢障碍 C 、对神经鞘细胞作用 D 、大脑神经细胞变性 E、神经系统的危害答案:B1 42 、慢性铅中毒主要引起A 、正常细胞性贫血 B 、小细胞低色素性贫血 C 、大细胞性贫血 D 、再生障碍性贫血 E、巨幼红细胞性贫血答案:B1 43 、慢性苯中毒主要损害的系统是A、消化 B 、血液 C 、造血 D 、循环 E、神经答案:C1 44、石棉尘可引起A 、胸膜间皮瘤 B 、膀胱癌 C 、肾癌 D 、皮肤癌 E、肝血管肉瘤答案:A1 45 、吸入高浓度可产生电击样死亡” 的有害气体是A、氮氧化物、H

41、2 s B 、H ? S 、H C N C 、H C N 、H C 1 D 、S O ? 、H C N E、N O ? 、N O答案:B1 46 、氟化物中毒的特效解毒剂是A、N a 2 s B 、N a N 02 C 、细胞色素C D 、小剂量的亚甲蓝E、亚硝酸钠- 硫代硫酸钠答案:E1 47 、尘肺诊断的主要临床依据是A、职业史 B 、症状与体征 C 、肺功能 D 、X 线胸片 E、病理切片答案:D1 48 、吸烟对人体的最大危害是引起A 、肺癌 B 、冠心病 C 、高血压 D 、肺炎答案:A1 49 、不洁性行为最主要的危害是E、慢性支气管炎A 、导致婚姻关系紧张 B 、严重影响子女身心

42、健康C 、性传播疾病D 、新版医生三基题库第1 2 页道德危机 E、社会不安定答案:C1 5 0、显示7 岁女童身高与体重的关系,宜绘制A 、散点图 B 、条图 C 、线图 I) 、直方图 E、半对数线图答案:A1 5 1 、为了由样本推断总体,样本应当是总体中A 、任意一部分 B 、典型部分 C 、有价值的一部分 D 、有意义的一部分E、有代表性的一部分答案:E1 5 2 、欲表示某地区某年各种死因的构成比,可绘制A 、线图 B 、直方图 C 、百分条图或圆图 D 、统计地图 E 、条图答案:C1 5 3 、下列哪些统计图适用于计数资料A 、直条图、直方图 B 、线图、半对数线图 C 、直条

43、图、百分直条图 D 、百分直条图、直方图 E 、散点图、线图答案:C1 5 4 、血清抗体效价的平均水平计算常用A 、算术均数 B 、中位数 C 、几何均数 D 、全距 E 、相对比答案:B1 5 5 、在同一总体中作样本含量相等的随机抽样,有9 9 % 的样本均数在下列哪项范围内A 、彳 2 . 5 8 S7B、牙 1 9 6 * * 4 2 . 5 8 %口 、1 , 9 6 。 和 、2 . 5 8 院答案:C1 5 6 、描述一组偏态分布资料的平均水平,以下列哪个指标较好A 、算术均数 B 、儿何均数 C 、百分位数 D 、四分位数间距 E 、中位数答案:E1 5 7 、殁表示A 、总

44、体均数的离散程度 B 、变量值x 的可靠程度 C 、样本均数的标准差D 、变量值间的差异大小 E 、总体均数标准误答案:C1 5 8 、若仅知道样本率,估计率的抽样误差该用下列哪个指标表示A 、 。P B 、笠 C 、SP D 、S E 、 。答案:C1 5 9 、某医院的资料. ,计算了各种疾病所占的比例,该指标为A 、发病率 B 、构成比 C 、标化发病比 D 、标化发病率 E 、相对比答案:B1 6 0 、用均数与标准差可全面描述下列哪种资料的特征A、正偏态分布 B 、负偏态分布 C 、对称分布 D 、正态分布和近似正态分布 E 任意分布答案:D新版医生三基题库第1 3 页16k横轴上,

45、口 1 . 9 6 。范围内的面积占正态曲线下总面积的A 、6 8 % B 、8 5 % C 、9 5 % D 、9 9 % E 、1 0 0 %答案:C1 6 2 、主诉的写作要求下列哪项不正确A 、提示疾病主要属何系统 B 、提示疾病的急性或慢性 C 、指出发生并发症的可能 D 、指出疾病的发展及预后 E 、文字精练、术语准确答案:D1 6 3 、病程记录的书写下列哪项不正确A 、症状及体征的变化 B 、检查结果及分析 C 、各级医师查房及会诊意见D 、每天均应记录一次 E 、临床操作及治疗措施答案:D1 6 4 、主诉的含义下列哪项正确A 、指病人的主要症状或体征及其看病的时间 B 、指

46、病人的主要症状或体征及其起病的时间 C 、指病人的主要症状或体征及持续的时间( 病程) D 、指病人的主要症状或体征及其发作的频率 E 、指病人的主要症状或体征及其严重的程度答案:C1 6 5 、病历书写不正确的是A 、入院记录需在2 4 小时内完成 B 、出院记录应转抄在门诊病历中 C 、接收记录由接受科室医师书写 D 、转科记录由原住院科室医师书写 E 、手术记录凡参加手术者均可书写答案:E166、有关病志书写不正确的是A 、首志由经管的住院医师书写 B 、病志一般可2 3天记一次 C 、危重病人需每天或随时记录 D 、会诊意见应记在病志中 E 、应记录各项检查结果及分析意见答案:A167

47、 、问诊正确的是A 、你心前区痛反射到左肩吗 B 、 你右上腹痛反射到右肩痛吗 C 、解大便有里急后重吗 D 、你觉得主要的是哪里不适 E 、腰痛时反射到大腿内侧痛吗答案:D168 、各项记录完成的时限,下列哪项有误A 、门( 急) 诊就诊时及时完成 B 、首志应在病人入院后6小时完成 C 、入院记录、再次( 多次) 入院记录应于病人入院后24 小时内完成 D 、 接班记录由接班医生接班后24 小时内完成 E 、转院记录应有主治医师及科主任的签名答案:B169 、有关门( 急) 诊病历书写有误的是A 、急诊病历由接诊医师及时书写,时间具体到分钟 B 、凡急诊死亡病人,病历一律由急诊科保留,不得

48、流失和外借 C 、留观病人最后的归转,应有记录 D 、门诊初诊病历应注明科别和日期( 年、月、日) ,病历记录含主诉、病史、体征,初步处理,诊断意见及签名 E 、留观出院者带药及休息可达7 天答案:E新版医生三基题库第14 页17 0 、作为年龄推断的指标,下列哪项不准确A 、皮肤弹性随年龄增长而减低 B 、牙齿脱落大多数老年人均有 C 、头发变白肯定已属老年 D 、角膜老年环见于60 岁以上的老人 E 、老年人大多肌肉萎缩松弛答案:C1 7k第二性征的区别下列哪项不正确A 、 体毛和阴毛分布的特征 B 、 乳房发育及皮下脂肪 C 、 肌肉发达程度 D 、皮肤色素分布 E 、声音强弱和音调答案

49、:D17 2、肺和胸膜触诊下列哪项不正确A 、呼吸运动度检查病人应作深呼吸 B 、应在胸部对称部位作比较检查 C 、以手掌或掌尺侧缘作语震检查 D、应注意病人是胸式呼吸还是腹式呼吸 E 、胸膜摩擦感以胸侧壁下部较易触及答案:D1 7 3 、肺部比较叩诊不正确的是A 、叩诊顺序,由上至下,由前至后,左右对称比较叩诊 B 、叩诊时应注意音响变化 C 、叩前胸与侧壁时板指与肋间平行 D、叩肩胛间区板指与脊柱平行E 、叩肩胛下区时板指可任意放置答案:E1 7 4 、区别腹部肿块来自腹腔或腹壁最简易的检查方法是A 、超声波检查 B 、胃肠钢餐检查 C 、腹部体格检查E 、同位素扫描答案:C1 7 5 、

50、鉴别右心衰与肝硬化的主要点是A 、有无腹水 B 、有无下肢浮肿 C 、肝脏是否肿大E 、有无脾大答案:D1 7 6 、心脏杂音听诊下列哪项不正确A 、杂音的部位 B 、杂音的时期 C 、杂音的性质E 、杂音传导的方向答案:D1 7 7 、腹部检查下列哪项错误D、腹部X 线平片D、颈静脉是否充倘D、杂音强度均应分级A 、振水声见于幽门梗阻 B 、 肋下扪及肝脏提示肝大 C 、脾脏正常时不能扪及D、肠鸣者消失可见于肠麻痹 E 、腹主动脉搏动正常人可触到答案:B1 7 8、检查发现病人胸廓的前后径等于横径,肋间隙增宽,应考虑为A、扁平胸 B 、鸡胸 C 、正常胸廓 D、漏斗胸 E 、桶状胸答案:E1

51、 7 9 、风湿性心脏病二尖瓣狭窄最重要的临床特征是A 、声音嘶哑 B 、X 线检查右房增大 C 、第一心音亢进 D、皮下小结E 、心尖区舒张期杂音新版医生三基题库第1 5 页答案:E1 80、抬举性心尖冲动最常见于A 、肺心病 B 、心肌炎 C 、右室肥大 D、高心病 E 、心包积液答案:D1 81 、一侧瞳孔直接对光反应消失,间接对光反应存在,病变部位在A 、同侧动眼神经 B 、对侧动眼神经 C 、同侧视神经 D 、对侧视神经E 、视交叉答案:C1 8 2 、三叉神经的功能不包括A 、面部感觉 B 、咀嚼运动 C 、下颌反射 D 、角膜反射 E 、面部表情肌运动答案:E1 8 3 、神经系

52、统病理反射的出现是由于A 、 脊髓反射弧的损害 B 、 神经系统兴奋性增高 C 、 脑干网状结构损害 D 、椎体束受损 E 、基底节受损答案:D1 8 4 、扪查乳房的方法哪项不正确A 、扪查乳房内半侧时,嘱病人举臂 B 、扪查乳房外半侧时,嘱病人垂臂 C 、应用手指掌面循序轻轻触按乳房 D 、乳房下部肿块,采平卧位举臂触诊 E 、抓捏乳房以利鉴别良恶性肿块答案:E1 8 5 、皮肤检查描述哪项不正确A 、皮肤粘膜出血,形成红色或暗红色斑,压之不褪色 B 、皮肤粘膜出血,不高出皮肤,2 m m 者,为出血点 C 、等麻疹呈苍白或片状发红改变,不突出皮肤表面D 、皮肤粘膜出血,高出皮肤,有波动,

53、5 m m 为血肿 E 、皮肤有小红点,高出皮肤表面,压之褪色者为丘疹答案:C1 8 6 、腹部触诊内容哪项不正确A 、压痛及反跳痛 B 、肌紧张度 C 、振水声 D 、肝脏 E 、脾脏答案:C1 8 7 、鉴别颈动脉搏动与颈静脉搏动最关键的是A 、搏动的位置 B 、搏动的范围 C 、搏动部位是否伴有血管杂音 D 、触诊指尖的搏动感,动脉搏动感较强 E 、搏动是否伴随缺氧表现答案:D1 8 8 、某病人右中肺听诊发现呼吸音柔软,呈吹风样 夫 音性质,吸气期较呼气期声音强,音调高,但吸气时间较呼气持续时间长,应诊断为A 、支气管呼吸音 B 、肺泡呼吸音 C 、支气管肺泡呼吸音 D 、减弱的支气管

54、呼吸音 E 、增强的支气管呼吸音答案:B1 8 9 、测血压时,袖带过窄将使测得的血压A 、增高 B 、降低 C 、舒张压降低,脉压增大 D 、脉压变小 E 、不新版医生三基题库第1 6 页受影响答案:A1 9 0 、下列穿刺部位,哪项不正确A、 股静脉穿刺点在腹股沟韧带下方紧靠股动脉外侧0 . 5 c m 处 B 、 颈内静脉穿刺在颈部中段,颈总动脉外侧刺入 C 、颈内静脉穿刺,在胸锁乳突肌锁骨头、胸骨头与锁骨形成的三角区顶部刺入 D、 锁骨下静脉穿刺, 在右锁骨下缘中点或内中1 / 3 或外、中1 / 3 交界处刺入 E 、动脉穿刺常选用股动脉、肱动脉或横动脉答案:A19k胸腔穿刺抽液引起

55、急性肺水肿是由于A、穿刺损伤肺组织 B 、抽液过多、过快,胸内压突然下降 C 、胸膜过敏反应D、穿刺损伤肺血管 E 、空气栓塞答案:B1 9 2 、有关胸腔穿刺的方法,下列哪项不正确A、穿刺抽液时;穿刺点一般取肩胛线7 9 肋间隙或腋中线6 7 肋间 B 、穿刺抽气时,穿刺点取患侧锁骨中线第2 肋间 C 、穿刺时应沿肋骨下缘进针 D、抽液量每次不超过1 0 0 0 m L E 、抽气量每次可大于1 0 0 0 m L答案:C1 9 3 、雾化吸入疗法选用药物时,下列哪些药物特性不适宜A、无强烈刺激性 B 、水溶性差,相对较稳定的药物 C 、无难闻气味 D、耐热稳定 E 、机体对该药不过敏答案:

56、B1 9 4 、下列哪项不宜采用雾化吸入治疗A、痰粘咳不出来 B 、支气管哮喘 C 、纤维支气管镜检查术前麻醉 D、气胸严重呼吸困难 E 、抗过敏或脱敏治疗答案:D1 9 5 、下列哪项禁做骨髓穿刺A、 显著血小板减少 B 、 粒细胞缺乏症 C 、 重度贫血 D、血友病 E 、恶性组织细胞病答案:D1 9 6 、一位休克病人,心率快,测中心静脉压为O . 4 9 k P a ( 5 c m H z O ) , 应采取A、迅速补充液体 B 、控制小量输液 C 、心功能不全,立即给强心、利尿药D、控制输液量,加用强心药 E 、休克与血容量无关答案:A1 9 7 、关于胃插管术,下列哪项是错误的A、

57、在胃扩张、幽门梗阻及食物中毒者可插管进行必要的治疗 B 、肠梗阻者可插管进行胃肠减压 C、对昏迷者,可插管行营养治疗 D 、对食管静脉破裂出血者,可插管观察有无活动性出血 E 、插管抽吸胃液进行分析答案:D1 98 、关于三腔二囊管压迫出血. ,下列哪项错误A 、插管前应检查有无漏气 B 、为证实管道是否进入胃内,可采用通过胃管腔向内注水的方法 C、先充气胃囊,后充气食管囊 D 、每2 3 小时检查气囊内压力1新版医生三基题库第1 7 页次 E 、每8 1 2 小时食管囊放气1 次,进行观察答案:B1 99、气管切开术后出现广泛皮下气肿,最简单的处理方法是A 、抽吸气体 B 、松解伤口缝线 C

58、、更换气管套管 D 、让其自行吸收E 、理疗答案:B2 00、环甲膜穿刺术穿刺针保留时间一般不超过A 、6 小时 B 、1 2 小时 C、2 4 小时 D 、4 8 小时 E 、7 2 小时答案:C20 K清创术下列操作哪项是错误的A 、伤口周围油污应用松节油擦去 B 、伤口周围皮肤用碘酊、乙醇消毒 C、切除失去活力的组织和明显挫伤的创缘组织 D 、深部伤口不宜再扩大 E 、用0. 9 %氯化钠注射液或过氧化氢溶液冲洗伤口答案:D2 02 、深部脓肿切开排脓的指征是A 、局部有红肿、剧痛 B 、全身发热,血白细胞数增高 C、患部运动功能障碍D 、局部有压痛及水肿 E 、局部穿刺有脓液答案:E2

59、 03 、有关气管切开术,下列哪项不正确A 、病人取仰卧位,肩垫高、头后仰、颈伸直 B 、皮肤切口从环状软骨下缘至胸骨上切迹稍上方 C、根据气管环为白色、触之有弹性、穿刺抽出气体来辨认气管D 、自2 3 环正中切开气管壁 E、下呼吸道分泌物阻塞,不适宜气管切开答案:E2 04 、气胸作闭式胸腔引流放置引流管的部位是A、锁骨中线第2 肋间 B 、锁骨中线第3 肋间 C 、腋前线第4 肋间 D 、腋前线第5 肋间 E、胸骨旁线第4 肋间答案:A2 05 、口对口人工呼吸时,将病人头部后仰,托起下颌,最主要的目的是A 、保持脑血液供应 B 、便于口对口接触 C 、避免口、鼻分泌物流出 D 、解除舌后

60、坠造成的咽阻塞 E、有利于保护颈椎答案:D2 06 、成年人胸外心脏按压,下列哪项正确A 、按压部位在胸骨下段,剑突上2 . 5 5 c m 处 B 、按压时以使胸廓下陷2 c m 为度C 、应用冲击式猛压,下压时间与放松时间比为2 : 1 D 、放松时手掌根离开胸骨E、按压频率以3 0次 / m i n 为宜答案:A2 07 、有关开胸心脏按压术,下列哪项错误A 、切口取左胸乳头下一个肋间,从胸骨左缘至腋中线 B 、切开心包 C 、用推压法或按压法按压心脏 D 、按压后心脏开始复跳,应立即关闭胸腔、避免感染E、可配合按压,向左心室注入肾上腺素答案:D新版医生三基题库第18 页2 08 、有关

61、骨折急救处理,下列哪项错误A 、首先应止血及包扎伤口 B 、无夹板时,可用树枝、木棍( 板) 步枪等做临时固定支架 C 、可将伤员上肢缚于胸壁侧面,下肢两腿绑在一起固定 D 、脊椎骨折病人最好俯卧位抬送 E、搬动脊椎骨折病人时,应采取一人抱肩,一人抬腿的方法答案:E2 09 、下列通气方法的适应证哪项错误A 、间歇通气适用于无自主呼吸者 B 、压力支持通气适用于有自主呼吸但通气不足者 C 、呼气末期正压呼吸适用于急性呼吸窘迫综合征 D 、高频通气适用于无自主呼吸者 E 、间歇指令通气和间歇同步指令通气适用于停机过渡的准备答案:D2 1 0 、有关生命医学伦理学基本原则的描述,错误的是A、不伤害

62、 B 、保护 C 、尊重 D 、公正 E 、有利答案:B2 1 1 、生命伦理学的研究领域不包括A 、临床生命伦理学 B 、理论生命伦理学 C 、心理生命伦理学 D 、科技生命伦理学 E 、文化生命伦理学答案:C2 1 2 、诊治伤害现象的划分应不包括A、有意伤害 B 、可知伤害 C 、免责伤害 D 、责任伤害 E 、可控伤害答案:C2 1 3 、影响和制约医疗水平的因素不包括A 、科技发展水平 B 、医务人员的道德水平 C 、病人的合作程度 I ) 、卫生政策和制度的合理性 E 、医务人员的技术水平答案:C2 1 4 、下列各项中不属于医师权利的是A 、诊治病人的疾病权 B 、宣告病人的死亡

63、权 C 、对病人的隔离权 D 、对病人实施 安乐死” 的权 E 、医师的干涉权答案:D2 1 5 、2 8 岁男性病人,急起发热,胸痛和气促6 天,叩诊心界明显扩大,吸气时脉搏变弱。1 小时前呼吸困难急剧加重,心率1 2 4 次 / m i n , 齐,心音低远,血压为8 . 0 / 6 . O k P a (6 0 /4 5 m m H g ) , 颈静脉怒张。最有效的抢救措施为A、静脉注射毛花昔C B 、肌内注射哌替咤 C 、静脉滴注多巴胺与间羟胺D 、持续吸高浓度氧 E 、心包穿刺减压答案:E2 1 6 、病人近3 个月以来夜间频发胸骨后疼痛,2 4 小时动态心电图示胸痛发作时胸导联ST

64、段上移,选用何种药物治疗最为恰当A 、毛花昔C B 、普蔡洛尔 C 、双喀达莫 D 、维拉帕米 E 、布桂嗪答案:D21 7 、高血压心脏病病人,突发呼吸困难,咳吐粉红色泡沫痰,血压为24 . 0 / 1 6 . Ok Pa ( 1 8 0/ 1 20 m n i I I g ) ,以下哪种药物可作为首选治疗用药A、利血平 B 、硫甲丙脯酸 C 、哌哇嗪 D 、硝普钠 E 、尼群地平答案:D21 8 、22岁男性病人,于主动脉瓣第二听诊区闻及舒张期吹风性杂音,以下哪项体征对其诊断最有帮助A 、心尖区舒张期震颤 B 、P2亢进并分裂 C 、交替脉 I ) 、心界向左侧扩大 E 、血压为20 .

65、4 / 5 . 3 k Pa ( 1 5 4 / 4 0 m m H g )答案:E21 9、1 8 岁女性病人,患风心病二尖瓣狭窄4 年,近半个月游走性关节痛,气促,以下哪项最可能提示病人发生了风湿性全心肌炎A 、心脏向双侧扩大 B 、心尖区收缩期山级杂音 C 、心包摩擦音 D 、急性肺水肿发作 E 、心电图P-R间期延长答案:C220 、6 4 岁男性病人,上班时突起持续剧烈左胸痛2小时,心电图示片5 T波明显变为高尖,心室率1 1 0 次 / m i n , 室性期前收缩7 次 / m i n , 为安全起见将病人转送至某医院急救,以下哪项措施最为重要A、静脉滴注硝酸甘油 B 、肌内注射

66、哌替喘 C 、静脉注射利多卡因继以持续滴注 D 、吸氧 E 、滴注极化液答案:C221 、诊断为肥厚型梗阻性心肌病的病人, 一般不宜应用A、地尔硫草B 、普蔡洛尔 C 、维拉帕米 D 、地高辛 E 、地西泮答案:D222、某窦性心动过缓者近年晕厥3 次,可选用下列哪项最安全而乂简便的方法协助诊断A、动态心电图 B 、心电图阿托品试验 C 、窦房传导时间测定 D 、希氏束电图 E 、窦房结恢复时间测定答案:B223 、某病人发生尖端扭转型室速,宜选用以下哪种药物治疗A 、奎尼丁 B 、普罗帕酮 C 、胺碘酮 D 、利多卡因 E 、丙毗胺答案:D224 、以下哪项检查为发现心肌缺血、诊断心绞痛最常

67、用简便的方法A 、心电图 B 、心尖搏动图 C 、心脏B 超 D 、冠状动脉造影 E 、刈 T i心肌显像答案:A225 、以下哪种情况不宜应用B受体阻滞药A 、二尖瓣脱垂 B 、肥厚型心肌病 C 、急性心肌梗死 D 、变异型心绞痛E 、室性期前收缩答案:D226 、慢性肺心病肺动脉高压形成的最主要原因是A 、肺气肿压迫及肺光壁破坏使肺毛细血管床减少 B 、肺小动脉炎 C 、血液粘稠度增加 D 、缺氧引起肺小动脉痉挛 E 、血容量增加答案:D227 、诊断中度慢性阻塞性肺病( C O P D ) , F E % 占预计值是新版医生三基题库第1 9 页A 、 F E V | 28 0 % B 、

68、 F E V 8 0 % , 2 5 0 % C 、 F E V K 5 0 % , 2 3 0 % D 、 F E V H B c A g D I l B x A g E 、p r e S2答案:B455、H B V 慢性感染者具有H B s A g 阳性、H B e A g 阳性、抗H B C 阳性、H B V D N A 阳性,而A L T 正常者,应诊断为A、H B e A g 阳性慢性乙型病毒性肝炎 B 、H B e A g 阴性慢性乙型病毒性肝炎 C 、慢性H B V 携带者 D 、非活动性H B s A g 携带者 E 、隐匿性慢性乙型病毒性肝炎答案:C456 、根据哪方面的临床

69、表现区别中枢性瘫痪和周围性瘫痪A 、肌力的大小 B 、有无感觉障碍 C 、有无病理反射 D 、有无大小便障碍 E 、有无意识障碍答 案 : C457、浅昏迷最有价值的体征是A 、对疼痛刺激有反应 B 、角膜反射消失 C 、无吞咽反射 D 、能执行简单的命令 E 、瞳孔对光反射消失答案:A458、下列情况中不符合急性脊髓炎的临床表现为A 、病前常有呼吸道感染症状 B 、损害平面以下传导束型感觉障碍 C 、大小便障碍 D 、损害平面以下运动障碍 E 、急性起病,早期出现肌张力增高,腱反射亢进答案:E459 1 下列治疗措施中不适合急性脊髓炎急性期治疗者为A、糖皮质激素 B 、 维生素B 族类药物

70、C 、 A T P D 、 E A C A 静脉滴注 E 、加强护理,防治并发症答案:D46 0 、不属于小脑后下动脉闭塞的临床表现为A 、 突发眩晕 B 、 眼球震颤 C 、 双下肢瘫痪 D 、 小脑性共济失调 E 、同侧面部及对侧半身痛温觉减退答案:C4 6K短暂性脑缺血发作的临床特征中不应出现A 、发作突然 B 、恢复较快, 般仅遗留较轻的神经功能缺损 C 、持续时间短暂,症状和体征在2 4小时内应完全消失 D 、常反复发作 E 、用小剂量阿司匹林治疗有效答案:B4 62 、治疗脑血栓形成最好的方法是A 、急性期慎用血管扩张剂 B 、抗凝治疗 C 、降纤治疗 D 、小剂量阿司匹林 E 、

71、r t - P A 静脉溶栓治疗答案:E4 63、蛛网膜下腔出血最常见的病因是A 、 血液系统疾病 B 、 先天性颅内动脉瘤 C 、 钩端螺旋体所致脑动脉炎 D 、新版医生三基题库第4 0 页烟雾病答案:BE 、心脏病4 64 、错觉是指A 、对未经历过的事物有熟悉感 B 、对客观事物能认知,但对其部分属性产生错误的感知和体验 C 、 对已知的事物有陌生感 D 、 对客观事物的歪曲知觉 E 、没有客观事物作用于器官时出现的知觉体验答案:D4 65 、下列哪一项最常见于脑器质性精神障碍A 、诡辩性思维 B 、思维奔逸 C 、思维松弛 D 、病理性赘述 E 、被害妄想答案:D4 66、关于戒酒综合

72、征,错误的说法是A 、与长期、大量饮酒有关 B 、症状出现于突然停止饮酒后4 8 96小时内 C 、可有情绪障碍、 思维障碍、 意识障碍等表现 D 、 为慢性中毒的表现形式之一 E 、可导致死亡答案:B4 67、酒精性震颤请妄为A 、一次大量饮酒出现的精神障碍 B 、长期饮酒后出现的幻觉妄想状态 C 、慢性酒精中毒突然停饮后出现的急性精神障碍 D 、慢性酒精中毒后出现的K o r s a k o v综合征 E 、慢性酒精中毒后出现的W e r n i c k 脑病答案:C4 68、阿尔茨海默( A l z h e i m e r )病病人外出不知归家属于A、行为紊乱 B 、 记忆障碍 C 、

73、意识清晰程度下降 D 、 意志减退 E 、错觉答案:B4 69、精神分裂症最常见的幻觉形式是A 、视幻觉 B 、听幻觉特别是语言性幻听 C 、嗅幻觉与味幻觉 D 、内脏性幻觉 E 、触幻觉答案:B4 7 0、危害行为发生率最高的精神疾病是A、精神分裂症 B 、躁狂症 C 、抑郁症 D 、反应性精神病 E 、偏执性精神病答案:A47K某患者,17 岁,既往学习成绩名列前茅,近半年来头痛、头昏、无力,上课注意力不集中, 成绩下降, 经常闭门思索” 人的尾巴的退化与大脑的发达有什么必然联系 的问题,拟著书立说,而对其母身患癌症毫不在意,被其父强迫来就诊。你考虑最可能的诊断为A、强迫症 B 、神经衰弱

74、 C 、精神分裂症 D 、抑郁症 E 、精神发育迟滞答案:C4 7 2 、病人拇指和食指围成一圈表示大家都团结一致,这是A 、关系妄想 B 、思维破裂 C 、象征性思维 D 、知觉障碍 E 、语词新作答案:C4 7 3 、精神科使用氯丙嗪主要是针对A 、 抑郁 B 、 兴奋躁动, 幻觉妄想, 思维障碍 C 、 衰弱无力, 精神委靡 D 、情感淡漠,精神衰退 E 、记忆力减退,智能障碍答案:B4 7 4 、心境障碍的病程常具有以下特点A 、反复发作,残留阴性症状 B 、反复发作,有可能自行缓解 C 、反复加重,从无缓解期 D 、一次发作,从不缓解 E 、一次发作,终生不发答案:B4 7 5 1某

75、男性,3 4 岁,近半年来情绪低落,对任何事物不感兴趣,对工作学习无信心,整日闷闷不乐,长叹短叹,自感身体乏力,不愿活动,纳差早醒,经常责任自己成了废人,加重了家里负担,曾有厌世念头,曾上吊未遂,系统检查未见异常,该病人诊断应考虑为A、神经衰弱 B 、精神分裂症 C 、抑郁症 D 、痴呆 E 、人格障碍答案:C4 7 6 、情感高涨与欣快症的区别点,下列哪一项是错误的A 、前者对任何事都感兴趣,表现轻松愉快,洋洋自得;后者给人以呆傻,愚蠢的印象B 、前者有较大的感染力;后者缺乏感染力,不能与正常人产生共鸣 C 、前者对知识和智力的利用增加;后者则下降 D 、 前者与环境的协调性保持完整;后者与

76、环境保持不协调 E 、前者是情感性精神病的症状之一;后者是精神分裂症症状之一答案:E4 7 7 、下列哪项属于精神科危机干预的范围A 、木僵状态 B 、知觉妄想 C 、自杀、自伤、伤人、毁物 D 、情感淡漠E 、意志减退答案:C4 7 8 、电休克的禁忌不包括A 、急性全身感染、 发热 B 、昏迷 C 、 严重心、 肝、肾、呼吸系统疾病 D 、精神分裂症木僵状态 E 、骨关节病、青光眼、视网膜脱离答案:D4 7 9 、精神分析理论认为神经症最基本的核心症状是A、焦虑 B 、强迫 C 、恐惧 D 、抑郁 E 、精神衰弱答案:A4 8 0 、神经症病人常见的躯体不适症状不包括A 、头颈疼痛 B 、

77、头昏 C 、心悸气足 D 、眩晕 E 、汗多、尿频答案:D48K某女,一般在细微的外界刺激甚至无明显原因的影响下情感容易引起波动,反应也迅速,有时也较强烈,常因无关紧要的事情而感动得伤心流泪而兴奋激动,无法克制,此女的症状属于A 、情感脆弱 B 、易激惹 C 、焦虑 D 、病理性激情 E 、病理性心境恶劣答案:A新版医生三基题库第4 1 页4 8 2 、下述哪条不属强迫症状A 、出门后反复检查门是否锁好 B 、穿衣服时需要先穿毛衣、外衣,再穿袜子、裤子,如次序打乱就烦躁不安 C 、总是怀疑自己听错了老师布置的作业,需反复向同学、老师验证才放心 D 、总是怀疑自己有病,虽然各种化验证实无病,仍不

78、放心E 、反复洗手,生怕手上有乙型肝炎病毒答案:D4 8 3 【疑病观念的病人,关注自己身体的各种细微变化,过分地注意健康状态,这属于A、注意转移 B 、注意减退 C 、注意涣散 D 、注意狭窄 E 、注意增强答案:E4 8 4 、哪项不属于假性痴呆的特点A 、 有明显的心理因素 B 、 精神刺激后急性发病 C 、 多有瘙症性格特征 D 、智能障碍的严重程度与日常生活有分离现象, 症状有夸张做作表现 E 、多有神经系统体征答案:E4 8 5 、急性渗出性湿疹选择哪种剂型最好A、溶液 B 、洗剂 C 、霜剂 D 、软膏 E 、酊剂答案:A4 8 6 、以下叙述哪项正确A 、角化不全常见于银屑病

79、B 、棘层松解见于湿疹 C 、基层液化见于天疱疮I) 、表皮内水疱见于舟状红斑狼疮 E 、表皮内水疱见于大疱性类天疱疮答案:A4 8 7 、下列哪项是慢性盘状红斑狼疮的特征A 、 面部蝶形红斑, 毛囊性角栓, 基底层液化变性, 真皮附属器周围淋巴细胞浸润 B 、贫血、白细胞减少阳性C 、血小板减少 D 、补体减少E 、红斑狼疮细胞检查答案:A4 8 8 、皮肌炎活动期中,下列叙述哪项是错误的A 、尿中肌酸增加 B 、血清肌酸磷酸激酶( C P K ) 减少 C 、肌电图示肌原性损害D 、乳酸脱氢酶增高 E 、尿肌酊排出量减少答案:B4 8 9 、下列哪种属结缔组织病A、自身敏感性皮炎 B 、湿

80、疹 C 、银屑病 D 、系统性红斑狼疮 E 、天疱疮答案:D4 9 0 、下列哪种疾病不属性传播疾病A、尖锐湿疣 B 、生殖器疱疹 C 、疥疮 D 、阴虱病 E 、外阴湿疹答案:E49K有关银屑病的叙述以下哪项是错误的A 、角化不全 B 、好发于头皮与四肢伸侧 C 、Ni k o l s k y - s 征 ( 棘层松解征) 阳性 D 、A u s p i t z 现象( 点状出血现象) E 、进行期易发生同形反应新版医生三基题库第4 2 页答案:C4 9 2 、急性虹膜睫状体炎时,局部治疗首先应该点用A 、 抗生素 B 、 抗病毒药 C 、 麻痹扩瞳剂抗真菌药答案:CD 、 抗生素加抗病毒药

81、 E 、4 9 3 、泪道冲洗时,水由下泪小点进入,自上泪小点流出,未见脓性分泌物,其诊断为A、泪小管阻塞 B 、泪小点阻塞 C 、泪小管炎 D 、慢性泪囊炎 E 、鼻泪管阻塞答案:E4 9 4 、前房角是指A 、虹膜之后与睫状体之间的间隙 B 、角膜之后的周边部分 C 、巩、角膜与虹膜睫状体之间的隐窝 D 、角膜与虹膜睫状体之夹角 E 、巩膜与虹膜睫状体之夹角答案:C4 9 5 、沙眼的病原体是A、细菌 B 、病毒 C 、真菌 D 、衣原体 E 、螺旋体答案:D4 9 6 、细菌性角膜溃疡中最严重的致病菌是A 、金黄色葡萄球菌 B 、肺炎链球菌 C 、溶血链球菌 D 、铜绿假单胞菌E 、流感

82、嗜血杆菌答案:D4 9 7 、交感性眼炎一般发生在穿透性眼外伤后A 、2 周 B 、2 8 周 C 、1 周 D 、2 8 月 E 、2 8 天答案:B4 9 8 、结膜炎的治疗中,哪一项是错误的A 、 冲洗结膜囊 B 、 冷敷 C 、 局部点用抗生素 D 、 全身应用抗生素 E 、遮盖患眼答案:E4 9 9 、急性闭角型青光眼的瞳孔开大是由于高眼压使A 、瞳孔括约肌麻痹 B 、瞳孔开大肌兴奋 C 、副交感神经抑制 D 、交感神经兴奋 E 、交感及副交感神经功能失调答案:A5 0 0 、视网膜硬性渗出物是由于A 、局部缺血坏死所致 B 、视网膜深层出血所致 C 、神经胶质细胞增殖所致D 、脂质

83、与变性巨噬细胞所致 E 、炎症所致答案:D5 0 1 、以下治疗青光眼的药物中哪一种的降压机制是使阻塞房角开放A、醋哇磺胺 B 、甘露醇 C 、曝吗洛尔 D 、甘油盐水 E 、毛果芸香碱答案:E5 0 2 、急性化脓性中耳炎早期最有效的处理是新版医生三基题库第4 3 页A 、抗生素全身应用及滴耳 B 、抗生素全身应用 C 、抗生素溶液滴耳 D 、2%酚甘油滴耳 E 、咽鼓管吹张答案:B5 0 3 、婴幼儿容易发生急性化脓性中耳炎的主要原因是A 、咽鼓管短、宽、平直 B 、咽鼓管峡部较窄 C 、咽鼓管发育不成熟 D 、婴幼儿抵抗力低 E 、婴幼儿上呼吸道感染较常见答案:A5 0 4 、慢性化脓性

84、中耳炎的诊断中,哪一项是最必需的检查A 、耳部X 线照片 B 、耳镜 C 、听力 D 、咽鼓管吹张 E 、平衡功能答案:B5 0 5 、胆脂瘤型中耳炎最危险的并发症是A、面瘫 B 、耳后喽管 C 、颅内并发症 D 、迷路炎 E 、颈深部感染答 案 : C5 0 6 、慢性化脓性中耳炎骨疡型或胆脂瘤型施行乳突根治手术的目的,下列哪项是最重要的A 、修补鼓膜 B 、保存听力 C 、清除病灶,预防颅内、外并发症 D 、改善中耳腔内压力 E 、防止发生眩晕症答案:C5 07 、鼻外伤引起的脑脊液鼻漏的处理,下述哪项不正确A 、降低颅内压 B 、鼻腔内药物腐蚀疗法 C 、鼻腔填塞 D 、预防感染E 、保

85、守治疗无效者应手术治疗答案:C5 08 、急性鼻窦炎的临床表现,下述哪项是错误的A 、常为多窦感染 B 、全身症状明显 C 、头痛重,有时间规律 D 、处理以全身用抗生素为主 E 、立即做上颌窦根治术及筛窦开放术答案:E5 09 、急性化脓性额窦炎出现头痛的时间是A 、夜间 B 、傍晚 C 、上午 D 、下午 E 、清晨答案:C5 10、鼻腔鼻窦恶性肿瘤临床特点,下述哪项是错误的A、好发于4 06 0岁年龄组 B 、病理以鳞癌为主 C 、大多数为蝶窦癌 D 、以原发为主 E 、就诊时晚期病人多答案:C5 11、鼻咽纤维血管瘤的致命危险是A、中颅底的骨质破坏 B 、大出血 C 、局部压迫 D 、

86、吞咽障碍 E 、呼吸障碍答案:B5 12、鼻咽癌的处理首先应选择A、手术疗法 B 、化疗 C 、放疗 D 、对症处理 E 、中药处理答案:C5 13 、对咽后壁脓肿行检杳或处理时,病儿体位应是新版医生三基题库第4 4 页A、坐位 B 、平卧位 C 、侧卧位 D 、半卧位 E 、仰卧垂头位答案:E5 14 、急性喉梗阻的主要症状是A 、 吸气性呼吸困难 B 、 喉痛 C 、 呼气性呼吸困难 D 、 吞咽困难 E 、阵发性咳嗽和呕吐答案:A5 15 、对小儿喉、气管、支气管炎施行气管切开的适应证为A、I 度呼吸困难 B 、 I I 度呼吸困难 C 、I I I 度呼吸困难 D 、I V度呼吸困难

87、E 、严重声嘶、咳嗽答案:C5 16 、 病人声嘶1个月,左侧真声带中1 / 3 有新生物,前联合及喉室均未受累,声带活动好,组织学诊断:鳞癌H级。处理方式应该采用A 、放射处理 B 、放射处理加喉切除 C 、喉切除 D 、半喉切除 E 、水平半喉切除答案:A5 1 7、混合牙列其年龄范围一般在A、3 7岁 B 、5 1 0 岁 C 、6 1 2 岁 D 、8 1 5 岁 E 、1 0 1 8 岁答案:C5 1 8 、附着于喙突的肌肉是A 、咬肌 B 、颗肌 C 、翼内肌 D 、翼外肌 E 、颊肌答案:B5 1 9、颌骨的血供主要来自于A、舌动脉 B 、面动脉 C 、甲状腺上动脉 D 、颌内动

88、脉 E 、颍浅答案:D5 2 0 、孕妇口腔疾病的最佳治疗时期为A 、1 个月内 B 、1 3 个月 C 、4 6个月 D 、6 8 个月 E 、8 9个月答案:C5 2 1 、下列哪项不是急性龈的特点A、多见于儿童或青年人 B 、质地湿软 C 、又称干性龈 D 、病变进展较快 E 、龈损呈浅棕色答案:C5 2 2 、可复性牙髓炎与不可复性牙髓炎的区别要点是A 、无自发痛,有刺激痛 B 、有自发痛及刺激痛 C 、有自发痛史,刺激去除后疼痛立即消失 D 、有自发痛史,刺激去除后疼痛持续较久 E 、有自发痛,无刺激痛答案:A5 2 3 、局部麻醉时出现注射区疼痛和水肿的原因正确的解释是A 、局部麻

89、醉药物变质 B 、注射针折断 C 、注射过程中刺破血管 D 、局部麻醉药物注入腮腺内 E 、注射时刺伤面神经答案:A新版医生三基题库第4 5 页5 2 4 、拔牙前后不需要常规给予抗生素预防并发症的是A 、糖尿病病人 B 、慢性肝炎病人 C 、慢性肾病病人 D 、风湿性心脏病病人 E 、先天性心脏病病人答案:B5 2 5 、干槽症的主要症状是A 、大量出血 B 、剧烈疼痛 C 、拔牙创面肉芽水肿 D 、骨壁轻微触痛E 、体温升高答案:B5 2 6、嚼肌间隙感染若未及时引流,最易引起的并发症是A 、败血症 B 、脓毒血症 C 、颌骨边缘性骨髓炎 D 、颌骨中央性骨髓炎E 、海绵窦血栓性静脉炎答案

90、:C5 2 7 、下列感染中应该及早切开引流的是A 、舌下间隙感染 B 、脏下间隙感染 C 、颌下间隙感染 D 、咬肌间隙感染 E 、腐败坏死性口底蜂窝织炎答案:E5 2 8 、舌后坠引起的呼吸困难,其主要抢救措施是A 、清除口腔分泌物 B 、头低侧卧位 C 、将舌牵向口外 D 、环甲膜穿刺E 、气管切开答案:C5 2 9 、下列关于儿童颌骨骨折治疗原则的叙述错误的是A 、尽早复位 B 、固定时间较成人短 C 、必须恢复伤前的咬合关系 D 、尽可能采用保守治疗 E 、如要手术治疗,术中应避免损伤牙胚答案:C5 3 0 、颔弓骨折时出现张口受限是由于A 、 局部出血 B 、 面神经受损 C 、

91、颗弓塌陷压迫段突 D 、 肌肉痉挛 E 、颗弓塌陷压迫喙突答案:E53K急性红斑型假丝酵母菌性口炎主要与以下哪种因素有关A 、白假丝酵母菌增殖 B 、长期配戴义齿 C 、新生儿感染白假丝酵母菌病D 、长期应用广谱抗生素 E 、反复感染单纯疱疹病毒答案:D5 3 2 、复发性疱疹性口炎复发的损害为A 、单个大水疱 B 、多个散在小水疱 C 、单个成簇的水疱 D 、多个成簇的水疱 E 、糜烂答案:D5 3 3 、天疱疮病变发生在粘膜或皮肤的结构是A、角质层 B 、颗粒层 C 、棘细胞层 D 、基底层 E 、粘膜下层答案:C5 3 4 、青春期龈炎的主要病因是A 、刷牙习惯不良 B 、牙错合拥挤 C

92、 、口呼吸习惯 D 、戴各种正畸矫治新版医生三基题库第46 页器 E 、青春期内分泌特别是性激素的改变答案:E5 3 5 、在快速进展性牙周炎的治疗中常采用的口服药是A 、四环素 B 、多西环素 C 、甲硝唾 D 、阿莫西林 E 、米诺环素答案:C5 3 6 、青少年牙周炎的主要致病菌为A 、 梭形杆菌 B 、 放线杆菌 C 、 牙龈口卜琳单胞菌 D 、 变形链球菌 E 、白假丝酵母菌答案:B1 7、急性不可复性盘性前移位的主要症状是A 、关节绞锁 B 、关节弹响 C 、张口受限 D 、关节轻微疼痛 E 、关节区肿胀答 案 : C5 3 8、口腔腺样囊性癌最常发生远处转移的脏器是A 、脑 B

93、、骨 C 、肝 D 、肾 E 、肺答案:E5 3 9 、颗下颌关节可复性盘前移位的主要治疗方法是A 、药物 B 、理疗 C 、牙合垫 D 、手术 E 、针灸答案:C5 40 、最易发生种植复发的涎腺肿瘤是A、肌上皮癌 B 、多形性腺瘤 C 、腺淋巴瘤 D 、基底细胞腺癌 E 、嗜酸细胞腺瘤答案:B5 41、如怀疑有颌下腺导管结石,以下哪种X线片检查为首选A、颌下腺造影 B 、下颌体腔片 C 、下颌曲面断层片 D 、颌下腺侧位片+下颌横断合片 E 、下颌骨侧位片+ 下颌横断合片答案:D5 42 、以下肿瘤中具有恶性倾向的是A 、牙龈瘤 B 、血管瘤 C 、淋巴管瘤 D 、乳头状瘤 E 、纤维瘤答

94、案:E5 43 、以下不宜行普通活检的恶性肿瘤是A、舌癌 B 、唇癌 C 、恶性淋巴瘤 D 、恶性黑色素瘤 E 、肉瘤答案:D5 44、属于牙源性囊肿的是A、球上颌囊肿 B 、始基囊肿 C 、鼻唇囊肿 D 、上腭正中囊肿 E 、鲤裂囊肿答案:B5 45 、下列哪种维生素缺乏最易引起牙龈出血A、维生素A B 、维生素B C 、维生素R D 、维生素C E 、维生素E答案:D5 46 、老年人踽病中的常见类型是A、静止龈 B 、猖獗龈 C 、根面龈 D 、活动 E 、窝沟龈答案:C5 47 、康复的对象是A 、 截瘫、 偏瘫病人 B 、 智力低下、 语言障碍病人 C 、 各种功能障碍的人 D 、心

95、肺功能障碍的病人 E 、小儿麻痹症,精神病人答案:C5 4 8 、 - 个步态周期是指A 、 从一侧足的足跟着地起, 到同一侧足跟再次着地为止的连续过程所用的时间 B、从一侧足的足尖着地起, 到同一足的足跟着地止的连续过程所用的时间 C 、 从一侧足尖着地起, 到另一侧足的足跟着地止的连续过程所用的时间 D 、 从一侧足的足跟着地起,到双足的足跟着地止所用的时间 E 、从一侧足跟着地起,到双足再下一次足尖着地止的连续过程答案:A5 4 9 、康复评估的特点是A 、重点是与生活自理、学习、劳动有关的综合功能评估 B、重点是运动能力的评估 C 、主要是医学心理学的检查 D 、职业能力的评估 E 、

96、针对病因的评估答案:A5 5 0 、以下儿种常见的病理步态哪一种是正确的A 、减痛步常见于足下垂 B、回旋步常见于偏瘫病人及足内翻病人 C 、剪刀步常见于小儿麻痹后遗症 D 、斜肩步常见于股四头肌瘫痪 E 、前冲步常见于小脑性共济失调答案:B5 5 1 、矫形器的使用目的包括A 、主要是预防或矫正畸形,减轻疼痛,补偿功能活动,支承体重,稳定肢体 B、主要是防止骨折和扭伤 C 、主要是为了加强肌力训练,发展肌肉 D 、主要是用于各种手术的保护 E 、主要用于纠正足下垂答案:A5 5 2 、根据热的基本移动形式,以下哪一种说法是正确的A 、温湿布、热敷袋、石蜡、热气、蒸汽属传导方式的热 B、高频电

97、流属于辐射方式的热 C 、红外线、电光浴属于“ 内源热 D 、超声波属于转换成高频振动的热 E 、温水的喷淋及冲洗属转换成机械振动的热答案:A5 5 3 、神经纤维生长速度每天平均为A 、1 2 m m B、2 . 5 m m C 、3 m m D 、3 4 m m E 、4 5 m m答案:A5 5 4 、不是超短波疗法的绝对禁忌证的是A 、妇女月经期下腹部 B、使用足够剂量抗癌药物的癌症病人 C 、戴有人工心脏起搏器者 D 、机体极度衰弱者 E 、高热病人答案:B5 5 5 、运动疗法的禁忌证新版医生三基题库第47 页A、脑血管意外 B 、截瘫 C 、急性心肌梗死 D 、颅脑外伤 E 、严

98、重衰弱病人答案:E5 5 6 、高压氧治疗C O 中毒的主要机制是A 、血液中物理溶解氧量增加 B 、血液中结合氧量增加 C 、血液中血红蛋白增加 D 、氧和血红蛋白的亲和力增加 E 、机体的摄氧能力增强答案:A5 5 7 、高压氧的绝对禁忌证之一是A 、急性鼻窦炎病人 B 、有颅骨缺损者 C 、妇女月经期与妊娠期 D 、未经处理的气胸 E 、活动性肺结核答案:D5 5 8 、标准大气压是指下列哪种条件下物体在单位面积上所承受的压力A 、在海平面上温度为4 c时 B 、在赤道海平面上,温度为0C时 C 、在赤道海平面上,温度为4 c时 D 、在纬度为45 的海平面上,温度为0C时 E 、在纬度

99、为45 。的海平面上,温度为4 c时答案:D5 5 9 、在高压氧舱内输液有发生气栓症的危险,主要发生在A 、加压过程中 B 、减压过程中 C 、高压氧治疗整个过程中均可发生 D 、0 . 3M P a 以上的高压氧治疗中 E 、0 . 2 M P a 以下的高压氧治疗中答案:B5 6 0 、氧气加压舱急排放应能使最高工作压到降至表压0 . O I M P a 的时间不超过A、1 分钟 B 、1 . 5 分钟 C 、2 分钟 D 、2 . 5 分钟 E 、3分钟答案:A5 6 1 、高压氧治疗的含义是A 、在常压下呼吸纯氧 B 、在超过常压的环境下吸3 0 % 以下浓度的氧气 C 、在超过一个

100、大气压的密闭的环境下呼吸纯氧或高浓度的氧气 D 、 在超过一个绝对压的环境下吸氧与C O ? 的混合气体 E 、在高压环境下吸空气答案:C5 6 2 、每次治疗完毕,舱内的紫外线空气消毒时间是A、1 0 分钟 B 、2 0 分钟 C 、30 分钟 D 、1 小时 E 、1 . 5 小时答案:C5 6 3、高压氧治疗时临床上常用的压力单位是A 、大气压 B 、表压 C 、绝对压 D 、附加压 E 、氧压答案:C5 6 4、温度不变时; 气体的体积( V ) 与压强( P ) 的关系是A、V1 / V2= P2 / P B 、V . / V2= P , / P 2 C 、V , = K V2P ,

101、 / PL 2 D 、V , = K V2P2 / PR V , V2= P , P2答案:A5 6 5 、常压下连续吸纯氧的安全时限为A、4 6 小时 B 、8 1 2 小时 C 、1 2 2 4小时 D 、2 4 48 小时 E 、48 小时以上答案:C新版医生三基题库第48 页5 6 6 、外界气压降低时,机体中氮的脱饱和速度最慢的组织是A、血液 B 、淋巴 C 、脂肪 D、肌肉 E、脑灰质答案:C5 6 7 、后装治疗中所用的侬I r 放射源刚投入临床使用时的活度为l O Ci ,用国际单位表示,下述正确的是A. 3 . 7 X 1 0l 0Bq B、 3 . 7 X 1 0 “ Bq

102、 C 2 . 7 X 1 0l 0Bq D 2 . 7 X 1 0 Bq E、2 . 7 X 1 01 2Bq答案:B5 6 8 、下列所述放射线,哪项一般不用于外照射A、a 射线 B、高能X 射线 C、高能电子束 D、高能质子束 E、 y射线答案:A5 6 9、立体定向放射手术( S R S )和立体定向放射治疗( S R T )的根本区别是A、立体定向技术不同 B、小野空间聚焦方式不同 C、剂量分割方式不同D、治疗的病变大小不同 E、剂量分布不同答案:C5 7 0 、下面对半价层的定义描述正确的是A、使射线能量减弱至一半所需的吸收体的厚度 B、使射线的强度减弱至半所需的吸收体厚度 C、使吸

103、收系数减弱至一半所需的吸收体厚度 D、使射线的能量和强度都减弱至一半所需的吸收体厚度 E、使射线的能量和吸收系数都减弱至一半所需的吸收体厚度答案:B5 7 1 、T M R 称为组织最大剂量比,它是以下哪个物理量的一个特殊情况A、 PDD B、 BS F C、 T PR D、 S AR E、 O AR答案:C5 7 2 、猝死常见的病因是A、冠心病 B、脑出血 C、急性肺动脉栓塞 D、癫痫大发作 E、急性坏死性胰腺炎答案:A5 7 3 、心脏性猝死最常见的病因是A、心肌病 B、冠心病 C、恶性心律失常 D、急性泵衰竭 E、病态窦房结综合征答案:B5 7 4 、心源性休克最常见的病因是A、急性广

104、泛心肌梗死 B、重症心肌炎 C、严重心功能不全 D、急性心脏压塞 E、心房粘液瘤答案:A5 7 5 、肺栓塞最常见的病因是A、空气栓塞 B、脂肪栓塞 C、血栓栓塞 D、羊水栓塞 E、瘤栓栓塞答案:C新版医生三基题库第4 9页5 7 6 、肾性急性肾衰竭最常见的原因A、急性肾小球性肾炎综合征 B、肾血管性疾病 C、严重的急性间质性肾炎D、急性肾小管坏死 E、慢性肾脏疾病的急性发作答案:D5 7 7 、对肾后性尿路梗阻的诊断,下列哪项不正确A、有泌尿系结石、盆腔脏器肿瘤或手术史 B、突然发生完全性无尿或间歇性无尿C、有肾绞痛与肾区叩击痛 D、尿常规检查无明显改变 E 、B 超或X 线泌尿系检查对诊

105、断帮助不大答案:E5 7 8 、急性腹痛伴休克,最常见的病因是A 、急性心肌梗死 B 、大叶性肺炎 C 、胃、十二指肠溃疡穿孔 D 、急性坏死性胰腺炎 E 、急性坏疽性胆囊炎答案:D5 7 9 、急性上消化道出血最常见的病因是A 、消化性溃疡出血 B 、应激性溃疡 C 、食管静脉破裂出血E 、胆道出血答案:A5 8 0 、一般在服毒后几小时内洗胃最有效A、4 8 小时内 B 、2 4 小时内 C 、1 2 小时内 D 、8 小时内时内答案:E5 8 1 、重金属中毒时用下列哪一种解毒药效果最好A、依地酸钙钠 B 、二乙烯三胺五乙酸三钠钙 C 、二貌丙醇D 、胃癌E 、 4 6小D 、二疏丙磺钠

106、 E 、二貌丁二钠答案:E5 8 2 、治疗氨基甲酸酯类杀虫药中毒时1 下列哪项措施是错误的A 、用肥皂水彻底清洗污染的皮肤 B 、用2 % 碳酸氢钠溶液洗胃 C 、用阿托品拮抗乙酰胆碱的作用 D 、应该用胆碱酯酶复能药治疗 E 、加强输液和利尿答 案 : D5 8 3 、下列哪种临床类型不是毒草中毒的类型A、胃肠炎型 B 、神经型 C 、精神异常型 D 、溶血型 E 、肾坏死型答案:E5 8 4 、抢救巴比妥类中毒所致呼吸衰竭的首要措施是A 、呼吸兴奋药的应用 B 、洗胃 C 、保持呼吸道通畅,人工辅助呼吸 D 、激素的应用 E 、利尿药的应用答案:C5 8 5 、银环蛇咬伤致死的主要原因是

107、A、循环衰竭 B 、D I C C 、呼吸衰竭 D 、肾衰竭 E 、肝功能衰竭答案:C5 8 6、毒蛇咬伤最有效的早期治疗方法是A 、局部注射胰蛋白酶 B 、局部清创 C 、单价抗蛇毒血清 D 、多价抗蛇新版医生三基题库第5 0 页毒血清 E 、中医中药答案:C5 8 7 、毒蛇咬伤最有效的局部早期处理方法是A 、胰蛋白酶局部注射或套封 B 、 拔除毒牙 C 、伤口近心端肢体结扎 D 、局部伤口烧灼 E 、局部外敷中草药答案:A5 8 8 、下列哪一种食物中毒以神经系统症状为主要临床表现,且病死率较高A 、沙门菌食物中毒 B 、嗜盐菌食物中毒 C 、变形杆菌食物中毒 D 、葡萄球菌食物中毒 E

108、 、肉毒杆菌食物中毒答案:E5 8 9 、下列哪一种食物中毒可呈阵发性腹部绞痛,大便呈洗肉水样A 、沙门菌食物中毒 B 、变形杆菌食物中毒 C 、嗜盐菌食物中毒 D 、葡萄球菌食物中毒 E 、肉毒杆菌食物中毒答案:C5 9 0 、海产品或盐腌渍品常引起下列哪一类食物中毒A 、沙门菌食物中毒 B 、嗜盐菌食物中毒 C 、变形杆菌食物中毒 I) 、葡萄球菌食物中毒 E 、肉毒杆菌食物中毒答案:B5 9 1 、对溺水所致呼吸心搏骤停者,其紧急处理措施最重要的是A 、立即倒水 B 、呼吸兴奋药的应用 C 、心内注射肾上腺素 D 、人工呼吸和胸外心脏按压 E 、皮质激素的应用答案:D5 9 2 、减压病

109、最有效的治疗是A、吸入高浓度氧气 B 、 肝素治疗 C 、 成分输血 D 、 高压氧治疗 E 、对症治疗答案:D5 9 3 、自身免疫性溶血性贫血病人输血应首选A 、浓缩红细胞 B 、洗涤红细胞 C 、少白细胞的红细胞 D 、冷冻红细胞E 、照射红细胞答案:B5 9 4 、严重肝脏疾病伴有凝血因子缺乏出血病人输血宜首选A 、保存的液体血浆 B 、新鲜冷冻血浆 C 、洗涤红细胞 D 、竣甲淀粉E 、清蛋白答案:B5 9 5 、C P D - A ( 枸椽酸钠、磷酸盐、葡萄糖腺嘿吟) 全血保存有效期为A 、1 0 天 B 、2 1 天 C 、2 8 天 D 、3 0 天 E 3 5 天答案:E5

110、9 6 、全血保存期的标准是根据输注2 4 小时体内红细胞存活率为A、4 0 % B 、5 0 % C 、6 0 % D 、7 0 % E 、9 0 %答案:D5 9 7 、新生儿溶血病当胆红素超过3 4 2 1 1 m o 1 / L 时,应首选新版医生三基题库第5 1 页A、光照疗法 B 、换血疗法 C 、宫内输血 D 、药物疗法 E 、液体疗法答案:B5 9 8 、多次输注血小板可产生A 、 胆固醇增高 B 、 白细胞增高 C 、 清蛋白增高 D 、 淋巴细胞增高 E 、抗血小板抗体答案:E5 9 9 、血小板输注时手工浓缩血小板剂量成人一般为A 、2 3 U B 、3 4 U C 、4

111、 5 U D 、6 8 U E 、2 0-3 0U答案:D6 00、补充不稳定凝血因子宜选用A、新鲜全血 B 、红细胞制剂 C 、普通血浆 D 、竣甲淀粉 E 、新鲜冷冻血浆答案:E6 01、重型珠蛋白生成障碍性贫血患儿输血宜首选A 、浓缩红细胞 B 、少白细胞的红细胞 C 、洗涤红细胞 D 、冷冻红细胞E 、年轻红细胞答案:E6 02 、临床输血中,非溶血性输血发热反应多考虑与哪种因素有关A 、H L A 抗体 B 、C l F 增高 C 、C , 增高 D 、C 增高 E 、I g A 增高答案:A6 03 、下列关于阴阳学说的基本内容的提法哪项是错误的A 、阴阳的交感相错 B 、阴阳的对

112、立制约 C 、阴阳的互根互用 D 、阴阳的消长和平衡 E 、阴阳的竞争势不两立答案:E6 04 、与发育、生殖关系最密切的脏腑是A 、心 B 、肝 C 、脾 D 、肺 E 、肾答案:E6 05 、中医学认为构成人体的基本物质是A、阴离子、 阳离子 B 、 六气 C 、 营卫之气 D 、 精、 气、 血、 津液 E 、肾气答案:D6 06 、络脉最细小的分支称为A、孙络 B 、细络 C 、浮络 D 、支络 E 、别络答案:A6 07 、所谓 六淫 者,以下哪种说法最准确A、 六气 B 、 风、 寒、 暑、 湿、 燥、 火 C 、 六元 D 、 六种不正常的气候 E 、风、寒、暑、湿、燥、火六种外

113、感病邪的统称答案:E6 08 、望诊面部呈青色时对下列哪种病症的诊断最有意义A、热证 B 、湿证 C 、寒证、痛证、血瘀和惊风 D 、虚证、脱血 E 、肾虚、痰饮答案:c6 09 、五色分属五脏,黑色属A 、肝 B 、脾 C 、肾 D 、肺 E 、心答案:C6 10、脉象:轻取即得,重按稍减而不空,举之泛泛有余,属A 、沉脉 B 、迟脉 C 、浮脉 D 、数脉 E 、洪脉答案:C61 k下列哪项不属中医“ 十问歌” 的内容A、问发热恶寒 B 、问头痛身痛况 E 、问既往患过哪些疾病C 、问大便、小便 D 、问住址和婚姻状答案:D6 1 2 、舌边红赤多见于A 、瘀血内阻 B 、肝胆热盛 C 、

114、心火上炎伤答案:B6 1 3 、身热、汗出、口渴、咳喘、苔黄舌红证属A、邪袭肺卫 B 、气营两燔 C 、阳明热盛隹八 、 、答案:D6 1 4 、腹部刺痛、固定不移,或按之有块者,应辨证为A、气滞 B 、血瘀 C 、寒凝 D 、热结答案:B6 1 5 、淋证的共同特征是A 、无尿 B 尿多 C 尿少、尿痛、尿急短涩、滴沥刺痛,欲出未尽,小腹拘急或痛引腰腹答 案 : E6 1 6 、诸淋临床各具特征,下述哪项是错误的D 、阴虚火旺 E 、气阴两D 、邪热壅肺 E 、热在上E 、虫病D 、尿血 E 、小便频数且A 、石淋,以小便排出砂石为主证 B 、膏淋实证而见小便混浊如米泪水或滑腻如脂膏 C 、

115、热淋,小便灼热刺痛 D 、劳淋,小便淋漓不已,遇劳即发 E 、气淋虚证,小便涩滞,淋漓不宣,少腹满痛答案:E6 1 7 、尿中所含有不使尿相对密度增高的物质是A、右旋醵酊 B 、放射造影剂 C 、尿素 D 、高蛋白质 E 、高葡萄糖答案:C6 1 8 、化学法粪便隐血试验,除愈创木酯法外,前3 天不必禁食A、动物血 B 、肉食 C 、猪肝 D 、含叶绿素食物 E 、果酱答案:E6 1 9 、与梅毒血清学试验无关的检验是A、U SRB 、 E SR C 、 R P RD 、 V D R LE 、 T P H A答案:B新版医生三基题库第5 2 页6 2 0、心肌梗死病人血清C K 值在发病儿小时

116、即开始增高A、2 4 B 、6 - 1 2 C 、3 - 1 0 D 、4 8 E 、1 2 2 4答案:A6 2 1 、周围血片中出现幼红细胞最可能是A、缺铁性贫血 B 、 溶血性贫血 C 、 再生障碍性贫血 D 、 淋巴瘤 E 、脾功能亢进答案:B6 2 2 、下列情况红细胞增多,哪项不是由于血液浓缩A 、连续呕吐 B 、高山居民 C 、反复腹泻 D 、出汗过多 E 、大面积烧伤答案:B6 2 3 、新鲜尿液外观混浊,加热后混浊消失,可能为A、磷酸盐 B 、草酸盐 C 、尿酸盐 D 、碳酸盐 E 、脓尿答案:C6 2 4 、尿微量清蛋白,以下说法哪项是错误的A 、用常规方法不能测出 B 、

117、超过尿蛋白正常范围的上限而定性方法又不能测出C 、可在隐匿型肾炎及肾炎恢复期尿中出现 D 、是比较灵敏的早期发现肾损伤的指标 E 、是指低分子量的蛋白答案:E6 2 5 、尿中有可能发现A、蟒虫雌虫 B 、阿米巴包囊 C 、蛔虫卵 D 、肝吸虫卵 E 、血吸虫尾蝴答案:A6 2 6 、可作为消化道恶性肠道肿瘤筛选检查的是A 、粪便隐血试验 B 、粪便中找癌细胞 C 、粪便中有红细胞 D 、肠纤维镜检查 E 、粪胆原试验答 案 : A6 2 7 、正常成人脑脊液中不可能出现A 、P a n d y 试验弱阳性 B 、蛋白质1 5 0 m g / L C 、葡萄糖3 m m o l / L D 、

118、氯化物 1 2 0 m m o l / L E 、白细胞8 X 1 0 7 L答案:E6 2 8 、作尿液妊娠试验,灵敏度最低,且已被淘汰的方法是A、单克隆酶免疫法 B 、雄蟾除试验 C 、胶乳凝集抑制试验 D 、放射免疫法 E 、红细胞凝集抑制试验答案:B6 2 9 、尿干化学分析仪检查蛋白质,主要检查A 、 球蛋白 B 、 清蛋白 C 、 球蛋白+ 清蛋白 D 、 球蛋白+ 微量清蛋白 E 、清蛋白+ 微量清蛋白答案:B6 3 0 、尿干化学分析仪检查白细胞,主要是检测A 、中性粒细胞 B 、中性粒细胞+ 淋巴细胞 C 、中性粒细胞+ 单核细胞 D 、新版医生三基题库第5 3 页全部白细胞

119、 E 、全部粒细胞答案:A6 3 1 、尿干化学分析仪检查白细胞和红细胞与显微镜检查白细胞和红细胞的关系A 、都有对应关系 B 、都无对应关系 C 、只白细胞有对应关系 D 、只红细胞有对应关系 E 、尿液混浊时才有对应关系答案:B6 3 2、口服胆囊造影宜选择下述哪种对比剂A、碘化油 B 、碘番酸 C 、碘苯酯 D 、碘卡明 E 、碘化钠答案:B6 3 3 、C T 扫描与体层摄影相比较,其最大的优点是A 、密度分辨率高 B 、空间分辨率高 C 、对比度增高 D 、操作方法简单E 、病人无痛苦答案:A6 3 4 、枕骨骨折的最佳摄片位置是A、颅骨前后位 B 、颅骨后前位 C 、水平侧位 D

120、、汤氏位 E 、颅底位答案:D6 3 5 、左、右倾后斜位支气管体层摄影的目的是为了显示A 、气管分叉部 B 、左、右主支气管 C 、中叶或舌段支气管 D 、上叶支气管 E 、下叶支气管答案:C6 3 6 、正常静脉肾盂造影、肾盂肾盏显影最浓的时间是静脉内注射对比剂后A、1 2分钟 B 、3 5 分钟 C 、 6 1 0分钟 D 、1 5 3 0分钟 E 、 6 01 20分钟答案:D6 3 7 、正常总胆管的宽度不超过A 、0. 5 c m B 、1 . 0c m C 、1 . 5 c m D 、2. 0c m E 、3 . 0c m答案:B6 3 8、肺癌空洞常见于A、鳞癌 B 、腺癌 C

121、 、大细胞未分化癌 D 、小细胞未分化癌 E 、细支气管- 肺泡癌答案:A6 3 9 、左侧位心脏照片上,心后缘与食管前间隙消失,提示A、右房增大 B 、右室增大 C 、左房增大 D 、左室增大 E 、肺动脉主干扩张答案:D6 4 0、左房增大最早出现的X 线征象是A 、左心缘第三弓突出 B 、右心缘双边阴影 C 、右心缘双房影 D 、右前斜位食管吞银左房压迹增加 E 、左前斜位左主支气管变窄、抬高答案:D6 4 1 、成人颅高压最常见的X 线征象是新版医生三基题库第5 4 页A、头颅扩大 B 、卤门增宽 C 、颅缝分离 D 、脑回压迹增多 E 、鞍背疏松脱钙答案:E6 4 2、脑膜瘤血管造影

122、的特征表现是A 、肿瘤染色 B 、静脉早显 C 、颈外动脉供血 D 、肿瘤血管栏栅状排列E 、血管弧形包绕移位答案:C6 43 、下列哪种组织对超声传播阻碍最小A、肌肉 B 、脂肪 C 、肝 D 、血液 E 、脾答案:D6 44、随着年龄增大,胰腺回声显示为A 、形状增大,回声强度降低 B 、形状缩小,回声增强 C 、形状增大,回声无变化 D 、形状增大,回声增强 E 、均无变化答案:B6 45 、用链霉素治疗结核病引起耳中毒症状,应采取下列哪一措施A 、换用卡那霉素 B 、换用耳毒性小的核糖霉素 C 、换用阿米卡因 D 、停用链霉素 E 、减低剂量答案:D6 46 、下列哪种抗生素较适用于治

123、疗支原体肺炎A、庆大霉素 B 、两性霉素B C 、氨芾西林 D 、头泡拉定 E 、多西环素答案:E6 47 、下列有关维生素K 临床应用的叙述中。哪一条是错误的A 、任何一种维生素K 都可用于防止由于维生素K 缺乏而引起的出血症 B 、中国药典只收载了维生素I,但维生素限和维生素降也是临床常用止血药 C 、维生素K可用于肌内注射或缓慢静脉滴注 D 、 本类药物是止血药, 凡与此目的不符的应用应认为是不合理用药 E 、肌内注射维生素K 比口服效果好答案:D6 48 、下列止喘药中,哪一种对心脏影响最轻A 、 肾上腺素 B 、 异丙肾上腺素 C 、 麻黄碱 D 、 沙丁胺醇( 舒喘灵) E 、氨茶

124、碱答案:D6 49 、胃溃疡病人宜选用下列哪种解热镇痛药A 、水杨酸钠 B 、阿司匹林 C 、口引躲美辛 D 、毗罗昔康 E 、保泰松答案:D6 5 0 、药物首过效应常发生于哪种给药方式之后A、口服 B 、舌下给药 C 、静脉注射 D 、透皮吸收 E 、吸入答案:A6 5 1 、死亡细胞变为嗜酸性,细胞核的细微结构消失,但细胞和组织结构的轮廓仍存在,这种改变称为A、坏疽性坏死 B 、液化性坏死 C 、干酪性坏死 D 、脂肪坏死 E 、新版医生三基题库第5 5 页凝固性坏死答案:E6 5 2 、股静脉内血栓脱落引起栓塞下列哪一项是不正确的A 、大多数栓塞于肺 B 、都发生出血性梗死 C 、如栓

125、塞于肺动脉主干常引起猝死 D 、伴有心力衰竭时一定发生相应部位的梗死 E 、如有心间隔缺损亦可栓塞于脑答案:B6 5 3 、除了哪一项外,下列均为胃溃疡的肉眼病变特点A、溃疡通常只有一个 B 、溃疡呈圆形或椭圆形 C 、溃疡直径一般大于2 c mD 、溃疡底部干净、光滑、边缘整齐 E 、溃疡深达肌层或浆膜层答案:C6 5 4、肺转移性肝癌指的是A 、肺癌转移至肝 B 、肝癌转移至肺 C 、肝癌和肺癌同时转移至他处 D 、它处的癌转移至肝和肺 E 、肝癌和肺癌互相转移答案:B6 5 5 、乳房出现肿块,质硬,推不动,其可能的诊断是A、乳腺癌 B 、 纤维腺瘤 C 、 乳腺小叶增生 D 、 纤维囊

126、性乳腺病 E 、脂肪瘤答案:A6 5 6 、下列哪项不是阴茎癌的特点A 、常发生于40 岁以上的男性 B 、病人大多有包皮过长 C 、肉眼常呈菜花状或溃疡状 D 、组织学上以低分化鳞状细胞癌最多见 E 、转移发生较早,远处转移少见答案:D6 5 7 、蕈样霉菌病是指A 、真菌感染 B 、淋巴结恶性淋巴瘤 C 、皮肤增生性结核 D 、皮肤瘤型麻风 E 、皮肤T 细胞性恶性淋巴瘤答案:E6 5 8 、绿色瘤是A 、 胆管上皮癌因淤胆所致 B 、 原发性肝细胞癌转移到皮下, 分泌胆汁所致 C 、原始粒细胞在骨组织、骨膜下或软组织中浸润,聚集成肿块 D 、绒毛膜癌阴道转移性结节 E 、血管肉瘤出血,产

127、生胆绿蛋白所致答案:C6 5 9 、脑梗死、短暂性脑缺血发作( T I A ) 的早期诊断,应首选以下哪种诊断手段A、局部脑血流断层核素显像 B 、X - C T 脑扫描 C 、磁共振脑部检查 D 、B型超声诊断 E 、X 线脑血管造影答案:A6 6 0 、核素治疗原理主要是利用哪种射线对病变进行局部照射而达到治疗目的A、Y 射线 B 、B射线 C 、X 射线 D 、中子 E 、质子答案:B6 6 1 、核素诊断肝海绵血管瘤的最恰当的方法是新版医生三基题库第5 6 页A 、肝胶体显像 B 、肝阳性显像 C 、肝血池显像 D 、肝胶体和肝血池联合显像答案:DE 、肝胆显像6 6 2 、甲状腺显像

128、诊断最有独特价值的适应证是A 、甲亢的诊断 B 、甲状腺炎的鉴别 C 、甲状腺癌的判定 D 、甲状腺瘤的判别 E 、异位甲状腺的定位判断答案:E6 6 3 、肝胆系统核素显像诊断对下述哪种疾病最有价值A、黄疸性肝炎 B 、肝硬化 C 、胆石症 D 、慢性肝炎 E 、急性胆囊炎答案:E6 6 4 、核医学诊断的原理是A 、放射性核素标记原理 B 、放射化学原理 C 、放射性示踪原理 D 、摄像原理 E 、生理生化原理答案:C6 6 5 、天然牛奶中缺乏哪种营养素A、优质蛋白质 B 、钾 C 、铁 D 、钙 E 、乳糖答案:C6 6 6 、以下不属于人体必需微量元素的是A、钠 B 、铁 C 、碘

129、D 、硒 E 、锌答案:A6 6 7 、以下属于人体必需氨基酸的是A、半胱氨酸 B 、谷氨酸 C 、酪氨酸 D 、缴氨酸 E 、天冬氨酸答案:D6 6 8 、影响基础代谢的因素不包括A、年龄 B 、性别 C 、体型 D 、内分泌 E 、体力活动答案:E6 6 9 、下列哪种元素是体内葡萄糖耐量因子的重要组成成分A、倍 B 、铜 C 、铁 D 、硒 E 、钥答案:A6 7 0 、动脉硬化病人宜食用A 、 低蛋白饮食 B 、 低纤维膳食 C 、 低胆固醇饮食 D 、 低盐膳食 E 、高钾低钠膳食答案:C6 7 1 、影响谷胱甘肽还原酶活力的营养素是A 、硒 B 、锌 C 、维生素B D 、维生索B

130、 ? E 、铁答案:A6 7 2 、以下哪种食物血糖指数最低A、红枣 B 、黄瓜 C 、柿子 D 、香蕉 E 、红果答案:B673 、低脂膳食不适用于A、脂肪肝B 、胰腺炎C 、胆囊疾患D 、冠心病E 、贫血答案:E674 、属于不饱和脂肪酸是A、棕桐酸 B 、月桂酸 C 、二十二碳六烯酸( D H A ) D 、丁酸 E 、硬脂酸答案:C675 、急、慢性肾衰竭病人宜用下列哪种膳食A、高蛋白饮食 B 、高纤维膳食 C 、少渣膳食 D 、麦淀粉膳食低铜膳食答案:D676、无盐膳食应禁用的食物A、牛奶 B 、咸蛋 C 、猪肉 D 、鱼 E 、鸡蛋答案:B677、医院感染主要发生在A、门诊、急诊病

131、人 B 、探视者 C 、医务人员 D 、住院病人E 、E 、陪护家属答案:D678、下列哪种情况属于医院感染A 、烧伤创面培养出金黄色葡萄球菌B 、经产道感染的乙型肝炎 C 、红霉素静脉滴注后发生静脉炎;拔出的导管在血平皿上滚动培养,细菌菌落数8个 D 、羊肠线吸收不佳的炎症反应 E 、入院后5 天发生的甲型肝炎答案:B679、下列概念中哪项是错误的A 、全面综合性监测可获得医院感染本底感染率 B 、目标性监测是将有限的人力、财力用于解决医院感染的关键问题 C 、 医院感染与传染病相同的是培养出细菌即能确诊 D 、医院感染的现患率高于同期的感染发生率 E 、医院感染漏报率的调查目的是对感染率监

132、测质量的评估答案:C680 、调查医院感染的最佳方法A 、从医院微生物室获得病例来源 B 、临床医师填报病例 C 、专职护士到病室前瞻性发现病例 D 、病室监控护士报告病例 E 、专职护士到病案室从出院病历中找查病例答案:C6 8 K密闭式潴留导尿系统引起感染的危险因素中,哪一项最常见A、潴留导尿持续1周 B 、集尿袋位处膀胱水平之上 C 、抗生素膀胱冲洗D 、集尿管扭结或阻塞 E 、密闭系统的接头反复打开答案:E6 8 2 、当被沾有HB s Ag 阳性血的针头刺伤时,立即应做的主要处理是A、伤口立即涂以碘酊 B 、给受伤者立即注射丙种球蛋白 C 、受伤者立即抽血作乙型肝炎病毒全套抗原抗体检

133、测 D 、乙型肝炎病毒高价免疫血清注射 E 、乙型肝炎病毒疫苗注射新版医生三基题库第5 7 页答案:D6 8 3 、免疫功能低下病人的医院感染预防,下列哪项错误A、保护皮肤粘膜的完整性 B 、广谱抗生素肠道预防感染 C 、保护性隔离D 、治疗局部感染病灶 E 、疫苗注射答案:B6 8 4 、有关隔离的概念,下列哪项是错误的A、咽白喉应严格隔离,皮肤白喉需接触隔离 B 、流行性腮腺炎进行呼吸道隔离,脊髓灰质炎进行肠道隔离 C 、大面积烧伤进行接触隔离 D 、多重耐药株感染应列入接触隔离 E 、活动性肺结核的隔离期是有效治疗后6 周答案:E6 8 5 、近年采用的隔离预防方法是A、A系统7 种隔离

134、方法 B 、B 系统按病采取相应隔离方法 C 、接触隔离与空气隔离 D 、标准预防的隔离方法 E 、普遍预防的策略答案:D6 8 6 、下列监测的控制指标中,哪项是错误的A、凡经消毒的物品不能检出任何微生物 B 、手术室医务人员手上带菌数应少于5个 / c m 、新生儿室、儿科病室的物体表面不得检出沙门菌 D 、手术室、产房、新生儿室空气中含菌量应少于20 0 个 / n u E 、紫外线消毒灯管在使用中的功率应大于7 0 u W /cm2答案:A6 8 7 、有关抗生素管理下列哪项错误A 、把抗菌药物分成一线、二线药物,二线药物应控制使用 B 、二线药物毒副作用大、价贵、疗效低 C 、在同类

135、抗生素中应留有替代性药,在必要时再启用 D 、无药敏结果参考时. , 可按经验选药原则用药 E 、 定期全院菌株的药敏统计结果公布,为临床用药与药剂科进药提供信息答案:B6 8 8 、医院感染常规监测中下列哪项是错误的A 、医院感染高危科室、高危人群监测 B 、漏报率监测 C 、医院感染高危因素监测 D 、医院感染暴发流行监测 E 、传染病报告卡监测答案:E6 8 9 、临床科室医院感染管理小组的职责下列哪项错误A 、制定木科室医院感染管理规章制度 B 、监督检查本科室有关医院感染管理的各项工作,对医院感染可疑病例, 可能存在感染的环节进行监测,并采取有效防治措施C 、对医院感染散发病例按要求

136、登记报告,并立即向当地卫生行政机构报告 D 、按要求对疑似或确诊病例留取临床标本, 进行细菌学检查和药敏试验 E 、 监督检查木科室抗生素使用情况答案:C6 9 0 、有关紫外线消毒效果监测下列哪项规定是错误的A 、对灯管应用时间、照射累计时间进行记录并签名 B 、对紫外线灯管在安装后、使用前应进行强度监测 C 、 使用中的灯管的强度监测应每个月监测1次 D 、新购灯管强度大于l O O u W / cm z ,使用中灯管强度大于7 0 u w / cm ? 为 合格灯管 E 、照新版医生三基题库第5 8 页射前后人工染菌杀灭率达到9 9 . 9%以上为合格灯管答案:C6 9 1、肝炎肠道门诊

137、的管理中下列哪项错误A 、诊室、人员、器械、用具固定 B 、挂号、候诊与普通门诊分开 C 、肝炎按黄疸和无黄疸进行隔离 D 、取药、注射、采血、化验与普通病人分开 E 、接触病人后洗手答案:C6 92 、治疗室的管理中下列哪项错误A 、注射、治疗时,应辅无菌盘,抽出的药液存放不能超过2 小时 B 、治疗室中开启的各种溶媒不得超过2 4 小时 C 、治疗室中开启的无菌溶液需要4 小时内使用D 、碘酊、乙醇瓶应保持密闭,每周更换及灭菌1 2 次 E 、经消毒后的棉球、纱布包,一经打开,保存时间不得超过4 8 小时答案:E6 93 、内镜室的医院感染管理,下列哪项错误A 、内镜室应设检查区和清洗消毒

138、区 B 、病人进行检查前必须先做肝功能、H B V 、H C V等标记物检查 C 、需要隔离者留于最后作检查 D 、每例内镜使用后应用高效消毒剂进行消毒处理,每天工作结束后对内镜进行终末消毒 E 、每天清洗消毒容器答案:C6 94 、对洗衣房的医院感染管理,下列哪项错误A 、布局上要求洁、污分区,路线由污到洁,不能逆行 B 、运送车辆应洁、污分开,浸有血液、 体液的布类应置于防水袋内封闭运送 C 、 洗衣房应有定期清洁消毒制度D 、各类衣服应分类清洗,感染病人与普通病人的衣物应分开清洗 E 、对血污布类应区分其感染性,必要时按感染性布类处理答案:E6 95 、消毒药械管理中下列哪项错误A 、根

139、据消毒的目的选用消毒药械 B 、同一消毒剂消毒不同物品应有同一浓度C 、加强消毒效能监测 D 、防止使用中的消毒剂再污染 E 、注意影响消毒效果的因素答案:B6 96 、在医院中幽门螺杆菌的传播以下列哪种方式最多见A、空气传播 B 、接触传播 C 、血液传播 D 、唾沫传播 E 、内镜操作传播答案:E6 97 、有关内镜消毒下列哪项是错误的A、腹腔镜用环氧乙烷消毒 B 、消化内镜用5 0 0 X 1 0 有效氯的含氯消毒剂消毒C 、直肠镜采用高压灭菌 D 、内镜活检钳使用过频时,采用2 % 戊二醛消毒4 5 分钟E 、磷苯二甲醛消毒内镜5 分钟可灭菌答案:B6 9 8 、卫生部对医院感染有关的

140、检查指标中下列哪项是错误的A 、医务人员接受医院感染在职教育每年不少于6 学时 B 、 常规器械消毒合格率应达1 0 0 % C 、当证实有医院感染爆发时医院应于4 8 小时内报告当地卫生部门 D 、新版医生三基题库第5 9 页医院抗生素使用率力争小于5 0 % E、一级、二级、三级医院感染率应分别控制在W7%、W8%、W 1 0 %答案:C6 9 9 、下列消毒剂中哪一种不能达到灭菌水平A、甲醛 B 、戊二醛 C 、含氯消毒剂 D 、环氧乙烷 E、过氧化氢答案:C7 0 0 、医院内最多见的多重耐药菌株是A、耐甲氧西林/ 苯嘎西林的金黄葡萄球菌( M R S A ) B 、耐万古霉素肠球菌(

141、 V R E)C 、耐万古霉素金黄葡萄球菌( V R S A ) D 、产超广谱醐大肠埃希菌 E、耐青霉素的肺炎链球菌答案:A7 0 1 、我国艾滋病最常见的传播途径是A 、 输血和血制品 B 、 静脉注射毒品 C 、 母婴垂直传播 D 、 性传播 E、儿童疫苗注射答案:B7 0 2 、下列消毒剂中哪一种为高效消毒剂A 、聚维铜碘 B 、碘酊 C 、乙醇 D 、含氯消毒剂 E、酚类消毒剂答案:D7 0 3 、洗手的指征中哪一项不合理A 、新生儿室入室常规规定应彻底洗手 B 、接触污物前、后应洗手 C 、侵入性操作前、后应洗手 D 、接触病人后应洗手 E、脱手套后需洗手答案:B7 0 4 、下列

142、哪项不属医院感染A 、阑尾穿孔引起的腹膜炎 B 、入院7 2 小时后发生的肺炎 C 、无症状导管菌尿症 D 、 烧伤6 小时入院的创面感染 E、 产道分娩吸入阴道分泌物的新生儿肺炎答案:A7 0 5 、M R S A 感染的最佳选药A、去甲万古霉素、万古霉素 B 、头泡呼林 C 、氧氟沙星 D 、磷霉素E 、阿米卡星答案:A7 0 6 、产超广谱酶( E S B L s ) 菌株所致感染的最佳选药是A、阿莫西林 B 、头泡西丁 C 、头剂曝的 D 、氨曲南 E 、哌拉西林答案:B7 0 7 、 医院感染管理规范对抗菌药物合理使用的要求中哪一项错误A 、医院应安排抗感染专家或有经验医师负责医院抗

143、感染药物使用的指导、咨询工作B 、提高细菌培养的送检率,促进抗菌药物合理使用水平 C 、医院抗菌药物使用率力争控制在6 0 % 以下 D 、 检验科和药剂科分别负责定期公布医院主要病原菌及其药敏试验结果 E 、有条件的医院应开展抗感染药物临床应用的监测答案:C7 0 8 、抗菌药物应用中,下列哪项属错误A 、病毒感染不用抗菌药物 B 、疑为细菌感染,应先送培养标本而后经验性选药新版医生三基题库第6 0 页C、培养结果出来,根据其药敏结果调整药物D、当应用抗菌药物24小时未见效立即改选其他药物E、一般感染抗菌药物用至症状体征消失后3 4天答案:D709、艾滋病人感染的预防,下列哪项错误A、晚期病

144、人应接受保护性隔离B、病人血中C D 淋巴细胞 0. 2义107L时应接受抗肺泡子虫感染的预防C、 旦发病即应用广谱抗菌药预防全身感染D、医务人员受HIV污染针头刺伤, 应立即报告并接受药物(AZT)预防期抗生素治疗答案:C710、从医院获得幽门螺杆菌感染的高危人群中主要是E、 确诊并发感染应早A、检验人员 B、消化内科医师E、口腔科医师答案:B7 1 K目前输血后肝炎以哪一种最多见C、消化内科护士D、腹部外科医师A、丁型肝炎B、乙型肝炎C、庚型肝炎D、TTV肝炎E、丙型肝炎答案:E712、下列抗菌药物中哪一种为窄谱抗菌药A、万古霉素B、阿米卡星C、阿奇霉素D、磷霉素E、环丙沙星答案:A713

145、、对厌氧菌有治疗作用的抗菌药物是A、妥布霉素答案:DB、红霉素C、头抱他定D、哌拉西林E、氨曲南714、小儿流感嗜血杆菌感染首选的药物是A、青霉素答案:BB、氨茶西林C、磺胺嗑咤D、头泡唾琳E、林可霉素715、潜伏病毒的激活感染是A、麻疹恢复期 B、隐性乙型脑炎病毒感染单纯疱疹反复发作 E、慢性活动性肝炎答案:D716、关于胃镜检查的适应证,下列哪项不正确C、乙型肝炎病毒携带者D、A、上腹痛原因未明B、呕血原因未明C、胃溃疡性质未明D、咯血查因E、锁骨上淋巴结肿大查因答案:D717、关于胃镜检查的禁忌证,下列哪项不正确A、严重心衰 B、精神病不合作者急性期 E、食管癌有吞咽梗阻者答案:E718

146、、关于胆道镜检查,下列哪项应慎重C、溃疡病急性穿孔者D、吞腐蚀剂A、可疑胆道残余结石的诊断B、胆道出血的定位或止血C、进行选择性肝内胆管造影D、胆总管十二指肠屡病人E、高龄或高危胆道结石新版医生三基题库第61页答案:D7 1 9 、下列哪项不宜做纤维支气管镜检查A 、 原因不明的咯血 B 、 原因不明的咳嗽 C 、 原因不明的喉返神经麻痹 D 、痰检结核菌阳性, X 线胸片肺无病灶 E 、肺心病并肺门肿大,原因未明, P a O 2 4 0 mmH g答案:E7 2 0 、有关纤维支气管镜检,下列哪项不正确A 、术前应禁食4 6 小时 B 、术前应作X 线胸片检查 C 、术前均应做肺通气功能检

147、查及血气分析 I) 、术前半小时注射阿托品及苯巴比妥 E 、术后应禁食2小时答案:C7 2 1 、下列哪项不宜通过纤维支气管镜检进行治疗A、取气管、支气管内异物 B、肿瘤的电凝、电切或激光治疗 C、病灶局部药物注射 D 、止血治疗 E 、气胸时经支气管抽气治疗答案:E7 2 2 、有关纤维支气管镜检查,下列哪项不正确A、可直接窥视1 4 级支气管内肿块 B、可发现叶、段支气管腔阻塞 C、可进行选择性支气管造影 D 、可直接窥视肺野浸润性病灶 E 、可进行肺浸润性病灶或肺外周肿块的活检答案:D7 2 3 、关于肺功能检查应用范围下列哪项是错的A、确定肺功能障碍的程度 B、判定肺功能障碍的类型 C

148、、可以发现肺部较小的病变 D 、可用以判断某些药物的疗效 E 、可以区别心源性和肺源性呼吸困难答案:C7 2 4 、诊断心房颤动最重要的证据是A、出现异常的P 波 B、P 波消失 C、Q - R 间期不规则 D 、Q R S 波群形态不一致 E 、心室率快答案:B7 2 5 、诊断急性心肌梗死最重要的心电图表现是A、病理性Q 波或Q S 波 B、S T 段弓背向上型上移 C、T 波倒置 D 、对应导联S T 段下移 E 、多发室性期前收缩答案:A7 2 6 、在鉴别室上性阵速与室性阵速时,以下哪项对室性阵速的诊断有肯定的意义A、 心室夺获及室性融合波 B、 Q R S 波群宽大畸形 C、 频率

149、2 00次 / m i n D 、T 波与Q R S 波群方向相反 E 、以上都不是答案:A7 2 7 、以下哪项最有助于房性期前收缩的诊断A、P - R 间期0. 1 2 s B、P 波倒置 C、Q R S 波群正常 D 、代偿间隙完全 E 、以上都不是答案:A7 2 8 、出现下述哪种波即可肯定病人处于轻睡期新版医生三基题库第6 2 页A、高波幅8节律 B、 无 a节律 C、 额 部 。 活动较多 D 、 阵发性短程1 2 1 6 波 /s E 、颍部尖波答案:D7 2 9 、脑电图检查每一病人应至少记录A、1 0分钟 B、2 0 3 0分钟 C、1 小时 D 、2 小时 E 、2 4 小

150、时答案:B7 3 0、精神运动性发作的脑电波异常波为A、额叶棘波 B 、双侧对称同步3 波 / s 棘慢综合 C 、高幅失律 D 、撅叶放电 E 、各导多棘慢波综合答案:D73 1 、同心针电极能探测A、针极周围1 m m 左右范围内的电活动 B 、整块肌肉的电活动 C 、 个完整的运动单位范围的电活动 D 、一根神经纤维支配肌肉范围的电活动 E 、一根肌纤维的电活动答案:A73 2 、束颤电位代表A 、肌纤维兴奋性增高 B 、肌束兴奋性增高 C 、运动单位兴奋性增高 D 、肌束兴奋性降低 E 、运动单位兴奋性降低答案:C73 3 、一侧耳蜗神经核受损,将导致A 、 同侧耳全聋 B 、 对侧耳

151、全聋 C 、 两耳全聋 D 、 两耳听力均减弱 E 、两耳听觉均正常答案:A73 4 、下列动脉中,哪些没有分支到胃A 、腹腔干 B 、肠系膜上动脉 C 、肝固有动脉 D 、脾动脉 E 、胃十二指肠动脉答案:B73 5 、腹股沟淋巴结收集A、下肢的淋巴 B 、腹壁的淋巴 C 、腰背部的淋巴 D 、会阴部的淋巴E 、臀部的淋巴答案:A73 6 、心肌的血液来自A 、胸主动脉的分支 B 、主动脉弓的分支 C 、左右冠状动脉 I) 、胸廓内动脉 E 、心包膈动脉答案:C73 7、不从内囊后脚通过的纤维束为A、皮质脊髓束 B 、皮质脑干束 C 、视辐射 D 、听辐射 E 、丘脑皮质束答案:B73 8

152、、迷走神经为A 、内脏运动纤维支配全身平滑肌运动 B 、内脏运动纤维支配全身腺体分泌活动新版医生三基题库第6 3 页C 、内脏运动纤维支配咽喉肌运动维管理全身粘膜感觉D 、内脏运动纤维支配心肌 E 、内脏感觉纤答案:D73 9 、手指夹纸试验是检查A、腋神经 B 、梯神经 C 、尺神经 D 、肌皮神经 E 、正中神经答案:C74 0 、关于网膜孔的描述,何者错误A 、上方有肝尾叶 B 、下方有十二指肠球门静脉 E 、是腹膜腔与网膜囊的通道答案:D74 1 、右主支气管的特点是A、细而短 B 、粗而短 C 、细而长斜答案:BC 、前方有胆总管 D 、后方有D 、粗而长 E 、较左主支气管倾74

153、2 、以下何者不是精索的结构A 、输精管 B 、睾丸动脉 C 、蔓状静脉丛 D 、提睾肌E 、射精管答案:E四、( 共2 96分) 每题可有一项或多项正确, 多选或少选均不得分1 2 73 、增强神经- 肌肉接头传递的因素A 、C o . 24B 、新斯的明C 、K D 、胆碱酯酶E 、箭毒答案:A . B1 2 74 、胃次全切除的病人引起贫血与下列哪些因素有关A 、F e2 4B 、维生素氏C 、维生素%D 、维生素EE 、内因子答案:C . E1 2 75、用已知A 型血与待测者血做交叉配血,若主反应凝集,次反应不凝集,待测者血型可能为A、A B 型B 、0 型C 、A型D 、B 型E

154、、A 。 型答案:B . E1 2 76、影响血钙水平的激素A 、降钙素B 、1 , 2 5( 0 H )2D3C 、胰岛素I ) 、1 1 - 去氧皮质酮E 、甲状旁腺激素答案:A , B . E1 2 77、糖皮质激素的生理作用A 、促进蛋白质分解B 、使淋巴细胞减少C 、升高血糖D 、使胃酸和胃蛋白酶增加E 、刺激I I 型肺泡细胞产生二软脂酰卵磷脂答 案 :A ,B ,C ,D ,E1 2 78、孕激素的生理作用A、助孕B 、促进排卵C 、安胎D 、促进乳房腺泡发育E 、产热答案:A , C , D , E1 2 79、内脏痛觉的特点A、定位精确B 、有牵涉痛C 、对牵拉烧伤敏感D 、

155、对炎症、切割敏感E 、对缺血敏感答案:B . E1 2 80 、哪些是胆碱能神经纤维A 、交感节前纤维B 、支配汗腺的交感节后纤维C 、副交感节后纤维D 、交感舒血管纤维E 、躯体运动神经纤维答 案 :A ,B ,C ,D ,E1 2 8 K 感受器有哪些共同生理特征A 、需适宜刺激B 、有感觉阈值C 、容易疲劳D 、有适应现象E 、有换能作用新版医生三基题库第97页答案:A , B , D , E1 2 82 、使瞳孔缩小的因素A 、肾上腺素B 、视近物C 、副交感神经兴奋D 、阿托品E 、有机磷农药答案:B , C . E1 2 83 、突触传递有何特征A、单向传递B、总和C 、相对不易疲

156、劳D 、中枢延搁E 、对内环境变化敏感答 案 . A B D F1 2 8 4 :弧漏内分泌功能A、分泌肾素B、分泌前列腺素C 、分泌活性维生素炉D 、分泌肾上腺素E 、分泌促红细胞生成素答 案 : A, B, C , E1 2 8 5 、M 样作用A、心跳加快、增强B、支气管平滑肌舒张C 、血压升高I ) 、缩瞳肌收缩E 、胃肠道平滑肌收缩答案:D . E1 2 8 6 、瞳孔反射A、强光时瞳孔缩小,弱光时瞳孔变化不大B、光照一侧瞳孔时,两侧瞳孔都缩小C 、看近物时,瞳孔扩大D 、看近物时,晶状体前凸E 、看近物时: 副交感神经兴奋答 案 : B, D , E1 2 8 7 、R 质粒包括A

157、、r 决定因子B、R T FC 、F 质粒D 、异染颗粒E 、中介体答案:A. B1 2 8 8 、食物中毒的诊断标准是A、发病与进食有关B、发病有群体性C 、有急性胃肠炎症状D 、发病人数超过5 0 %E 、从呕吐物、粪便及剩余食物中分离出同一病原体答 案 : A, B, C , E1 2 8 9 、( ) T 试验的临床意义有A、协助对儿童结核病诊断B、诊断成年人结核病C 、选择BC G 接种对象D 、是成年人细胞免疫功能指标之一E 、可作为BC G 接种效果的检测指标答案:A, C , D , E1 2 9 0 、引起非典型肺炎的病原体有A、肺炎支原体B、S AR S 冠状病毒C 、肺炎

158、双球菌D 、肺炎衣原体E 、结核分枝杆菌答案:A, B, D1 2 9 1 、病毒灭活的概念是A、失去感染性B、保留抗原性C 、保留血凝特性I) 、保留细胞融合特性E 、保留遗传特性答案:A, B. C1 2 9 2 、乙型肝炎传播的途径有A、消化道传播B、呼吸道传播C 、母婴传播D 、性接触传播E 、血行传播答案:C , D . E1 2 9 3 、引起性病的病原体有A、淋病奈瑟菌B、梅毒螺旋体C 、衣原体新版医生三基题库第9 8 页D 、H I VE H A V答 案 : A , B , C , D1 2 9 4 、免疫三大标记技术是A 、免疫荧光技术B 、酶免疫测定C 、放射免疫测定D

159、、协同凝集E 、免疫电泳答 案 : A , B , C1 2 9 5 、自然疫源性疾病的特点有A 、自然界长期有病原体存在B 、节肢动物为传播媒介C 、发病有地方性D 、发病有季节性E 、局部地区突发性烈性传染病答 案 : A , B , C , D1 2 9 6 、下列哪些病原体可引起食物中毒A、霍乱弧菌B 、肉毒杆菌C 、蜡样芽胞杆菌D 、黄曲霉E 、产气荚膜梭菌答案:B , C , D , E1 2 9 7 、引起脑膜炎的病原体有A 、脑膜炎奈瑟菌B 、结核分枝杆菌C 、新型隐球菌D 、钩端螺旋体E 、白喉棒状杆菌答案:A , B , C , D1 2 9 8 、甲型流感病毒中抗原漂移的

160、概念是指A、H A 和N A 变异幅度小B 、形成新的亚型C 、可引起中小型流行D 、H A 和N A 系量变E 、H A 和N A 发生质变答案:A , C . D1 2 9 9 、立克次体的特点是A 、大多是人畜共患病原体B 、节肢动物常为传播媒介C 、在活细胞内以二分裂方式繁殖I ) 、所致疾病多为自然疫源性疾病E 、对所有抗生素及磺胺类药物敏感答 案 : A , B , C , D1 3 00, 引起间质性肺炎的病原体有A、肺炎链球菌B 、呼吸道合胞病毒C 、肺炎支原体I ) 、肺炎衣原体E 、E C H O 病毒答案:B , C . D1 3 0 K 高渗性脱水易出现A、口渴 B 、

161、休克 C 、尿少 D 、脱水热 E 、皮肤弹性降低答案:A , C . D1 3 02 、低钾血症可引起A 、骨骼肌兴奋性降低 B 、心肌兴奋性降低 C 、心肌传导性升高 D 、心肌自律性升高 E 、平滑肌兴奋性降低答案:A , D , E1 3 03 、低钾时心电图的变化是A 、 T 波低平 B 、出现u 波 C 、 Q R S 波群增宽 D 、 P - R 间期缩短 E 、 Q - T间期缩短答案:A, B , C1 3 04 、清钾浓度过高者可采取的措施有A 、葡萄糖和胰岛素同时静脉注射 B 、腹膜透析C 、阳离子交换树脂灌肠或口服 D 、补充钙剂使细胞外液C a ?, 增多 E 、补充

162、钠盐使细胞外液N a * 增多答案:A , B , C , D , E13 05、导致有效胶体渗透压下降的因素有A 、 血浆清蛋白浓度下降 B 、 微血管通透性降低 C 、 毛细血管血压增高 D 、淋巴回流受阻 E 、组织间液胶渗压降低答案:A . D13 06 、导致血管内外液体失平衡而形成水肿的基本因素有A 、毛细血管有效流体静压升高 B 、有效胶体渗透压降低 C 、淋巴回流受阻D 、血浆清蛋白含量升高 E 、微血管通透性降低答案:A , B . C13 07 、肾病综合征产生全身性水肿的主要机制有A 、 血浆胶体渗透压下降 B 、 醛固酮分泌增多 C 、 肝脏合成清蛋白减少 D 、抗利尿

163、激素分泌增多 E 、肾小球滤过率增加答案:A , B . D13 08、下述哪些物质属内生致热原A、白细胞介素T B 、前列腺素E C 、干扰素 D 、肿瘤坏死因子 E 、巨噬细胞炎症蛋白T新版医生三基题库第99页答案:A , C , D , E13 09、A G i E 常型的代谢性酸中毒可见于A、严重心力衰竭 B 、饥饿 C 、肾小管酸中毒 D 、过量使用乙酰哇胺E 、摄入大量阿司匹林答案:C . D13 10、缺氧初期心排血量增加的机制是A 、心率加快 B 、心肌收缩力增强 C 、静脉回流增加 D 、呼吸运动增强E 、心肌耗氧量增加答 室 . A R C D1 3 1 k 定蓄金质的生物

164、合成中A、氨基酸的氨基与t R N A 结合 B 、t R N A 的3 C C A - 0H 携带氨基酸 C 、m R N A 起模板作用 D 、 s n R N A 是合成蛋白质的场所 E 、原核生物和真核生物的I F 明显不同答案:B, C. E1 3 1 2 、终止密码是指A 、 U A A B、 U A G C、 U G G D 、 U G A E、 A U G答案:A , B, D1 3 1 3 、翻译的特点是A 、 沿m R N A 的5 3 方向进行 B、 起始密码子位于m R N A 开放阅读框的5 端 C、终止密码位于m R N A 开放阅读框的3 端 D 、多肽链合成方向

165、是从C端一N 端延伸E、需要消耗A T P 和G T P答 案 : A , B, C, E1 3 1 4 、关于氨基酸活化的正确叙述是A、在细胞质中进行 B、需氨基酰t R N A 合成酶催化 C、氨基酸以非共价键结合到特异的t R N A 分子上 D 、消耗A T P E、消耗G T P答案:A , B. D1 3 1 5 、通过G 蛋白耦联通路发挥作用的有A 、 胰高血糖素 B、 肾上腺素 C、 甲状腺素 D 、 促肾上腺皮质激素 E、抗利尿激素答案:A , B, D , E1 3 1 6 、下列哪些物质儿乎仅由肝脏合成A、尿素 B、脂肪酸 C、胆固醇 D 、酮体 E、糖原答案:A . D

166、1 3 1 7 、新斯的明临床用于A 、重症肌无力 B、麻醉前给药 C、手术后腹胀气与尿潴留 D 、阵发性室上性心动过速 E、筒箭毒碱中毒答案:A , C, D , E1 3 1 8 、,鼠套上腺素扩瞳作用的特点是A 、维持时间短 B、升高眼内压 C、不升高眼内压 D 、引起调节麻痹E、不引起调节麻痹答案:A , C, E1 3 1 9 、症册性休克首选肾上腺素,主要与其下述作用有关新版医生三基题库第1 0 0 页A 、兴奋心脏3受体,使心排血量增加 B、兴奋支气管跖受体,使支气管平滑肌松弛 C、兴奋眼辐射肌a受体,使瞳孔开大 D 、兴奋血管a受体,使外周血管收缩,血压升高;使支气管粘膜血管收

167、缩, 降低毛细血管的通透性,利于消除支气管粘膜水肿,减少支气管分泌 E、抑制肥大细胞释放过敏性物质答案:A , B , D , E1 3 2 0 、下列药物为保钾利尿药的是A 、螺内酯 B 、阿米洛利 C 、吠塞米 D 、答案:A , B . D1 3 2 1 、对晕动病所致呕吐有效的药物是A、苯海拉明 B 、异丙嗪 C 、氯丙嗪 D 、答 案 : A , B , D , E1 3 2 2 、诱发强心昔中毒的因素有A、低钾血症 B 、低氯血症 C 、高钙血症症答案:A , C1 3 2 3 、应听力损害的药物有A、头抱氨革 B 、卡那霉素 C 、依他尼酸答案:B , C , D , E1 3

168、2 4 、环境污染引起的疾病有A、传染病 B 、尘肺、中毒性疾病 C 、公害病氨苯蝶噬东葭若碱D 、低钠血症D 、链霉素D 、职业病E 、氢氯曝嗪E 、美克洛嗪E 、高钾血E 、吠塞米E 、食原性疾病答 案 : A , C , D , E1 3 2 5 、维生素B ? 缺乏引起的疾病有A、脂溢性皮炎 B 、口角炎C 、舌炎D 、脚气病E 、唇炎答案:A , B , C , E1 3 2 6 、低盐或无盐膳食适用于A 、缺血性心力衰竭的病人 B 、高血压的病人 C 、肝硬化腹水的病人 D 、肾脏疾病的病人 E 、浮肿的病人答 案 : A , B , C , D , E1 3 2 7 、低蛋白膳食

169、适用于A 、急性肾炎的病人 B 、尿毒症的病人 C 、心脏病的病人 D 、肝功能衰竭者的病人 E 、中毒烧伤的病人答 案 : A , B . D1 3 2 8 、职业性损害包括A、工作有关疾病 B 、职业性外伤 C 、职业病害病答案:A , B , C1 3 2 9 、以下哪些属于计量资料A、身高 B 、脉搏数 C 、血压 D 、体重答案:A , B , C , D , E1 3 3 0 、福林向数差别的显著性检验用t 检验的条件是D 、食物中毒E 、白细胞数E 、公新版医生三基题库第1 0 1 页A 、两总体符合正态分布 B 、两总体均数相等 C 、两总体方差( 标准差的平方)相等 D 、两

170、样本例数很大 E 、两样本分组一致答案:A . C1 3 3 1 、标准差的应用包括A 、反映一组观察值的离散程度 B 、计算标准误 C 、计算变异系数 D 、估计医学参考值的范围 E 、计算相对数答案:A , B , C , D1 3 3 2 、儿科特殊病史应包括A、生产史 B、喂养史 C、生长发育史 D、预防接种史 E、生活史答案:A , B, C, D, E1 3 3 3 、过去病史包括下列哪儿项A 、传染病史及接触史 B、手术外伤史 C、家族遗传史 D、局灶病史E、预防接种史及药物过敏史答 案 : A , B, D, E1 3 3 4 、下列哪些体征属于脑膜刺激征A 、 Ker n i

171、 g 征 B、 La s eg u e征 C、 Br u d z i n s k i 征 D、 Ba b i n s k i 征 E、Go r d o n 征答案:A . C1 3 3 5 、呼吸三凹征是指吸气时下列部位内陷A、胸骨上窝 B、锁骨上窝答案:A , B. E1 3 3 6 、引起腹压痛的原因有A、腹部炎症 B、肿瘤浸润欠妥答案:A , B, C, D, E1 3 3 7、奇脉常见于下列疾病A、冠心病 B、心肌炎炎答案:D, E1 3 3 8、导尿操作,下列哪些正确C、肋间肌 D、腹上角 E、肋间隙C、脏器淤血 D、肠寄生虫病 E、手法C、心肌病 D、心包腔积液 E、缩窄性心包A

172、、男性消毒从尿道口开始C、导尿管插入深度为2 . 5 c mE、以上都不是答案:A , C1 3 3 9 、下列哪些情况不能洗胃B、女性消毒从大腿内侧开始,由外向内顺序进行D、膀胱过度充盈,应立即插入导尿管,快速放尿A 、幽门梗阻 B、腐蚀性胃炎 C、胃扩张 D、严重食管、胃底静脉曲张E、以上都不是答案:B, D1 3 4 0 、下列哪些情况不宜使用常频呼吸机通气A 、中量以上咯血 B、肺大泡或气胸 C、低血容量休克 D、急性呼吸窘迫综合征 E、以上都不是新版医生三基题库第1 0 2 页答案:A , B, C1 3 4 1 、下列疾病常用的洗胃液哪些是正确的A 、原因不明急性中毒用温水洗胃 B

173、、有机磷农药敌百虫中毒用碳酸氢钠溶液洗胃C、重金属中毒用茶水洗胃 D 、有机磷农药对硫磷( 1 6 0 5 ) 中毒用高镒酸钾洗胃E 、以上都不是答案:A , B . C1 3 4 2 、颅内压增高时,常出现高原波,亦称A 波。下列描述哪些是正确的A 、 高原波的发生是颅内压增高发展的一个过程, 表示此时空间代偿已完全丧失 B 、当高原波出现时, 颅内压力已经很高 C 、 开始出现高原波时, 颅内压力为中度增高,不伴任何症状,如进一步发展,颅 内压迅速增加,高原波持续时间越长,症状越明显D 、睡眠时,由于C O ,潴留,颅内血管扩张,血容量增加,因而易出现高原波 E 、高原波反复发作,加重了脑

174、血液循环障碍,加重了脑功能损伤,因此要尽快降低颅内压,中断高原波的发生答案:A ,C ,D ,E1 3 4 3 、常见的误诊、漏诊的原因包括下面哪儿种A 、病史资料不完整、不确切 B 、观察不细致或检验结果误差 C 、先人为主、主观臆断 D 、医学知识不足、缺乏临床经验 E 、疾病的临床表现不同答案:A ,B ,C ,D1 3 4 4 、临床思维的基本原则有A 、实事求是的原则, 一元论” 原则 B 、用发病率和疾病谱观点选择诊断的原则C 、首先考虑器质性疾病的诊断,然后考虑功能性疾病的原则I) 、首先考虑可治的疾病的原则,简化思维程序的原则答案:A , B , C , D , E1 3 4

175、5 、综合的临床诊断应包括A 、病因诊断 B 、病理解剖诊断期 E 、并发症及伴发疾病诊断答案:A ,B ,C ,D ,E1 3 4 6 、道德起源的理论有A、实践道德论 B 、天赋道德论E 、“ 神启论E 、见病见人的原则C 、病理生理诊断C 、人的自然本性论D 、疾病的分型与分D 、心理道德论答案:A , B . D1 3 4 7 、道德的特点包括A 、稳定性 B 、规范性 C 、天赋性 D 、社会性 E 、层次性答案:B , C . E1 3 4 8 、医学伦理学研究的对象包括A 、医务人员与病人及其家属的关系B 、医护人员相互之间的关系 C 、病人与病人之间的关系 D 、医务人员与社会

176、的关系 E 、病人与社会之间的关系答案:A . B , D . E1 3 4 9 、生命伦理学的研究领域包括A 、理论生命伦理学 B 、临床生命伦理学 C 、道德生命伦理学 D 、文化生命伦理学 E 、未来生命伦理学答案:A , B . D1 3 5 0 、医学人道观、人权观的核心内容包括A 、尊重病人生命 B 、尊重病人的人格 C 、尊重病人的家属 D 、尊重病人平等的医疗权利 E 、尊重病人的习惯答 案 . A B D1 3 5 1 :1裁道德情感包括A 、同情感 B 、责任感 C 、事业感 D 、成就感 E 、愧疚感答案:A , B ,C1 3 5 2 、病人的权利包括A、基本医疗权 B

177、 、保护隐私权 C 、要求赔偿权 D 、要求” 安乐死权E 、知情同意权密 室 . A R ( F1 3 5 3 / 显前纷发生的原因包括A 、 社会舆论的缺陷 B 、 医疗部门自身的缺陷 C 、 病人家属行为的缺陷 D 、病人就医行为的缺陷 E 、医疗纠纷调解行为的缺陷答案:B , D1 3 5 4 、人类生态环境保护的道德原则包括A 、尊重自然的道德原则 B 、合理利用资源的道德原则 C 、系统综合的道德原则 D 、同步效应的道德原则 E 、面向未来的道德原则答案:A ,B ,C ,D ,E1 3 5 5 、根据移植用器官的供者和受者关系,器官移植可分为A、自体移植 B 、同质移植 C 、

178、同种异植 D 、人造器官移植 E 、异种移植答案:A , B , C , E1 3 5 6 最 的 高 血 压 急 性 并 发 症 有A、脑出血 B 、肾衰竭 C 、脑梗死 D 、急性冠脉综合征 E 、急性左心室衰竭答 案 . A B C E1 3 5 7 蚕雇)前收缩常见于A、冠心病 B 、心肌病 C 、风湿性心脏病 D 、二尖瓣脱垂 E 、高血压答案:A ,B ,C ,D1 3 5 8 、阵发性室上性心动过速的治疗下列哪些是正确的A 、腺背快速静脉 B 、米力农静注 C 、洋地黄静注 D 、艾司洛尔静注E 、维拉帕米静注答案:A , C , D , E1 3 5 9 、下列快速心律失常中哪

179、些首选电复律A 、心室颤动 B 、心室扑动 C 、房颤伴预激伴血压下降 D 、快速室速伴血压下降 E 、室上性心动过速答案:A , B , C , D1 3 6 0 、电复律的禁忌证有新版医生三基题库第1 0 3 页A 、心室扑动 B 、心房颤动伴完全性房室传导阻滞 C 、持续室性心动过速D、病态窦房结综合征伴快速房颤 E、心室颤动答案:B . D1 3 6 1 、急性心肌梗死患者听诊心脏时,可有以下体征A 、心包摩擦音 B 、胸骨左缘3 4 肋间收缩期杂音 C 、心尖区收缩中晚期喀喇音 D、第四心音 E、心尖区第一心音增强答案:A , B , C , D1 3 6 2 、以下疾病常有晕厥发作

180、并可能猝死A 、预激综合征 B 、肥厚型心肌病 C 、室间隔缺损 D、主动脉瓣狭窄E、室性心动过速答案:B . D1 3 6 3 、慢性支气管炎可分为A、单纯型 B 、气肿型 C 、喘息型 D、混合型 E、慢性迁延型答案:A . C1 3 6 4 、下列哪些项目可作为诊断慢性肺心病的条件A 、慢性肺、胸疾病史 B 、左室肥大或左心衰 C 、肺动脉高压表现 D、右室肥大或右心衰 E、心律失常答案:A , C . D1 3 6 5 、下列哪些项目符合渗出液改变A 、相对密度2 1 . 0 1 8 B 、蛋白质3 0 g / L C 、粘蛋白试验( R i l l a t a 试验) 阴性D、细胞数

181、2 0 0 个 / u L E、胸液蛋白/ 血清蛋白 0 . 5答 案 : A , B , D1 3 6 6 、下列疾病抗感染治疗的原则,哪些正确A、肺炎链球菌肺炎首选青霉素G 治疗,体温正常3 天后可停药。 B 、金黄色葡萄球菌肺炎选用苯哇西林或第一代头泡菌素、万古霉素治疗 C 、支原体、衣原体肺炎选用 6内酰胺类抗生素治疗 D、急性吸入性肺脓肿首选大剂量青霉素G 治疗 E、急性吸入性肺脓肿抗生素治疗至体温正常,咳痰基本消失时停药答案:A , B . D1 3 6 7 、下列哪些药物具有舒张支气管作用A、B受体阻滞药 B 、胆碱能受体激动药 C 、茶碱 D、 3受体激动药E、 B 2 受体激

182、动药答案:C . E1 3 6 8 、尿胆原阳性可见于A 、 肝细胞性黄疸 B 、中毒性肝炎 C 、 溶血性黄疸 D 、 胆总管癌 E 、再生障碍性贫血答案:A, B , C1 3 69 、胃液分析结果为胃酸缺乏可见于A、慢性浅表性胃炎 B 、慢性A型萎缩性胃炎 C 、慢性B 型萎缩性胃炎 D 、胃癌 E 、胃溃疡答案:B . D1 3 70 、幽门螺杆菌( H p ) 感染相关性疾病有新版医生三基题库第1 0 4 页A、慢性胃炎 B 、平滑肌瘤 C 、溃疡病 D 、血管瘤 E 、胃癌答案:A, C , E1 3 71 、血反性肝癌伴癌综合征的表现有A、伴高糖血症 B 、伴红细胞增多症 C 、

183、伴低钙血症 D 、伴高胆固醇症E 、伴血小板增多症答案:B , D . E1 3 72 、溶血性贫血红细胞破坏过多的实验室根据为A、血红蛋白血症 B 、血浆结合珠蛋白增高 C 、尿胆原排出增多 D 、血清中以直接胆红素增高为主 E 、网织红细胞计数增高答案:A. C1 3 73 、周围血片中出现幼红细胞的疾病有A、再生障碍性贫血 B 、急性粒细胞白血病 C 、脾功能亢进 D 、骨髓纤维化 E 、血友病答案:B , D1 3 74 、糖尿病酮症酸中毒治疗中如果补碱过多过快,会出现哪些严重并发症A、脑水肿 B 、加重组织缺氧 C 、碱中毒 D 、缺钾 E 、低血糖答 案 : A, B , C ,

184、D1 3 75 、测定血清T S H 的方法有A、放射免疫学 B 、免疫放射学 C 、免疫化学发光法 D 、酶联免疫法E 、时间分辨免疫荧光法答案:A. B . C1 3 76、血红蛋白尿的临床表现为A、尿液静置后有红色沉淀 B 、尿色呈红葡萄酒色 C 、全身无溶血的表现I) 、显微镜检无红细胞或少数红细胞 E 、振荡时呈云雾状答案:B , D1 3 77、5进性肾炎与急性肾炎的鉴别是前者具有A、大量蛋白尿 B 、持续性少尿或无尿 C 、显著高血压 D 、迅速发生并加重的肾功能损害 E 、水肿答案:B , D1 3 78、皮肌炎的酶学检查中,有诊断价值的是A、谷草转氨酶 B 、醛缩酶 C 、乳

185、酸脱氢酶 D 、肌酸磷酸激酶 E 、以上都不是答案:B , D1 3 7 ; 、露确诊系统性进行性硬化症的指标有A 、皮肤对称性弥漫性水肿性硬化 B 、红细胞沉降率增快 C 、皮肤病理检查示胶原纤维肿胀 D 、类风湿因子阳性 E 、以上都不是答案:A . C1 3 8 0 、肥皂水刷手法,下列哪项是正确的A、必须从指尖顺序地刷洗双手 B 、刷洗2 遍、共5分钟 C 、刷洗肘关节以上上臂1 0 c m 处 D 、刷洗完后穿无菌手术衣 E 、不慎污染已刷洗部位应重新刷洗一遍新版医生三基题库第1 0 5页答案:A , C1 3 8 : 、学术室常用的化学消毒剂有A 、 0 . 1 %苯扎滨镂 B 、

186、 1 0 % 甲醛溶液 C 、 器械溶液 D 、 9 5 % 乙醇 E 、聚维酮碘答案:A , B , C , E1 3 8 2 、高钾血症的处理原则是A 、积极防治心律失常 B 、立即停止钾盐摄入 C 、降低血清钾浓度 D 、原发病治疗 E 、改善肾功能答案:A , B , C , D , E1 3 8 3 、等渗性缺水常见的病因有A 、 肠瘦 B 、大量呕吐 C 、大创面慢性渗液 D 、高热、大量出汗 E 、腹腔内感染答案:A , B , E1 3 8 4 、/ 东克病人的一般监测项目包括A 、中心静脉压 B 、肺动脉楔压 C 、血压 D 、心脏指数 E 、动脉血气分析答案:C1 3 8

187、5、常见的手术后并发症有A 、 手术后出血 B 、 切口感染 C 、 肺部感染和肺不张 D 、 尿路感染 E 、呃逆答案:A , B , C , D1 3 8 6 、破伤风病人较常见的并发症有A、角弓反张 B 、窒息 C 、酸中毒 D 、高热 E 、循环衰竭答案:B , C . E1 3 8 7 、对损伤修复的不利因素有A、感染 B 、低蛋白血症 C、服 用 口 引 口 朵 美 辛 D 、糖尿病 E 、肝硬化答案:A, B, C, D , E1 3 8 8 、烧伤病人的治疗原则包括A、镇静止痛等对症治疗 B、预防和治疗低血容量或休克 C、尽早手术治疗促进创面愈合 D 、预防和治疗M S O F

188、 E 、治疗局部和全身的感染答案:B, D , E1 3 8 9 、月褊病人行化疗后常见的不良反应有A、便血 B、血尿 C、毛发脱落 D 、免疫能力降低 E 、皮肤粘膜改变答案:B, C, D1 3 9 0 、急性化脓性腹膜炎手术治疗的指征A、腹腔内病变严重 B、盆腔器官感染引起的腹膜炎 C、腹膜炎严重,无局限趋势而病因不明者 D 、病人一般情况差,中毒症状严重,有休克表现 E 、经1 2 小时保守治疗,腹膜炎症状加重者答案:A, C, D , E1 3 9 1 、结肠手术前肠道准备新版医生三基题库第1 0 6 页A、术前2 3 天进流质 B、术前2 3 天服用抗生素 C、术前2 天服用泻剂D

189、 、术前1 天禁食 E 、手术前晚清洁灌肠答案:A, B, C, E1 3 9 2 、右半结肠癌的临床表现为A、肠刺激症状 B、便血 C、全身中毒症状较明显 D 、腹部可扪及肿块E 、有慢性肠梗阻表现答案:A, C, D , E1 3 9 3 、门腔分流术有A、脾肾静脉分流术 B、脾腔静脉分流术 C、T I P S D 、门腔静脉分流术E 、肠系膜上、下腔静脉分流术答 案 : A, B, C, D , E1 3 9 4 、AO S E 的主要临床表现为A、腹痛 B、寒热 C、黄疸 D 、休克 E 、神经症状答 案 : A, B, C, D , E1 3 9 5 、急性重症胰腺炎的诊断要点为A、

190、 心率 1 2 0 次 / m i n , 心律失常, 低血压或休克 B、 血钙 1 2 m m o l / L , 血糖/ L C、呼吸困难,呼吸率 3 0 次 / m i n , P a 02 1 0 0 m m H g D 、尿量 4 0 m L / h , 血尿素氮增高 E 、D I C发生答案:A, D , E1 3 9 6 、下肢静脉曲张可出现的并发症有A、肢体缺血坏死 B 、血栓性静脉炎 C 、曲张静脉破裂急性出血 D 、湿疹或溃疡形成 E 、干性坏疽答案:B , C . D1 3 9 7 、颅内压增高是神经外科常见的临床病理综合征,是以下疾病的共有征象A 、颅脑损伤 B 、脑肿

191、瘤 C 、脑出血 D 、脑积水答案:A , B , C , D , E1 3 9 8 、急性颅内压增高常见于A 、急性颅内血肿 B 、慢性硬膜下血肿 C 、颅内肿瘤血 E 、高血压脑出血答案:A . E1 3 9 9 、脑疝的病因有以下儿种A 、颅脑外伤 B 、颅内感染性疾病如脑脓肿 C 、颅内肿瘤E 、颅内炎症D 、蛛网膜下腔出D 、颅内寄生虫病及其他肉芽肿性病变 E 、颅高压病人不适当的腰穿放液答 案 : A , B , C , D , E1 4 0 0 、造成婴儿脑积水的常见病因有A 、产伤后颅内出血 B 、颅内感染 C 、蛛网膜下腔或蛛网膜颗粒粘连 D 、颅脑的先天畸形 E 、病因不明

192、答案:A , B , C , D , E1 4 0 1 、成年人的颅内肿瘤多为A、胶质瘤 B 、髓母细胞瘤 C 、脑膜瘤 D 、垂体瘤 E 、转移癌答案:B . C . D新版医生三基题库第1 0 7 页1 4 0 2 、颅内肿瘤应与下列哪些疾病鉴别A 、脑脓肿 B 、脑结核瘤 C 、慢性硬膜下血肿 D 、假性脑瘤 E 、先天性脑积水答 案 : A , B , C , D , E1 4 0 3 、降低颅内压增高的综合治疗措施有A 、冬眠低温或亚低温治疗 B 、激素的治疗 C 、限制水钠的输入量 D 、保持呼吸道通畅 E 、合理的体位答案:A , B , C , D , E1 4 0 4 、伽玛

193、刀是利用立体定向技术与计算机辅助的放疗设备,它的主要特点是A 、治疗精度高 B 、照射能量大 C 、适用于所有的颅内肿瘤 D 、适用于脑内神经核团或神经通路的定向毁损 E 、适用于范围较局限的脑动静脉畸形答案:A , B , D , E1 4 0 5 、闭式胸膜腔插管引流术的指征为A 、开胸手术者 B 、气、血胸经反复抽吸无效者 C 、脓胸、脓气胸经反复抽吸无效者 D 、中等量以上血胸 E 、脓胸并存支气管胸膜屡者答 案 : A , B , C , D , E1 4 0 6 、关于胸壁的恶性肿瘤,下述正确的是A 、肉瘤多见 B 、 骨软骨瘤多见 C 、 生长迅速 D 、 表面血运丰富 E 、少

194、见病理性骨折答案:A , C . D1 4 0 7 、急性脓胸的治疗措施包括A 、抗生素治疗 B 、全身支持疗法 C 、控制原发病灶 D 、胸腔闭式引流E 、胸腔穿刺抽脓答 案 : A , B , C , D , E1 4 0 8 、确诊肺癌的依据包括A 、咳嗽、痰中带血 B 、胸部X 线平片 C 、胸部C T 检查 D 、痰细胞学检查E 、纤维支气管镜检查及活检答案:D , E1 4 0 9 、早期食管癌的症状是A、症状不明显 B 、吞咽困难 C 、持续胸背痛 D 、吞咽哽噎感 E 、吞咽食管内异物感答案:A , D . E1 4 1 0 、食管癌发病可能与下列哪些因素有关A 、 亚硝胺 B

195、 、 缺乏维生素 C 、 缺乏某些微量元素 D 、 遗传易感性 E 、病毒感染答案:A , B , C , D1 4 1 1 、关于胸腺瘤的描述,正确的有A 、多位于前上纵隔 B 、男女发病率相近 C 、恶性胸腺瘤多伴有重症肌无力D 、恶性胸腺瘤病人术后应予放疗 E 、重症肌无力病人多伴有胸腺瘤或胸腺异常增生答案:A , B , C , D , E新版医生三基题库第1 0 8 页1 4 1 2 、法洛四联症是指A 、 室间隔缺损 B 、 肺动脉口狭窄右心室肥大答 案 : A , B , D , E1 4 1 3 、关于主动脉夹层描述,正确的是A 、高血压是引起主动脉夹层的主要原因C 、 主动脉

196、疾病的死亡首因是主动脉夹层C 、 右心房肥大 D 、 主动脉骑跨 E 、B 、年龄 4 0 岁病因多为马方综合征D 、 常采用的分型为S t a n f o r d 分型 E 、不管是急性或慢性夹层均是手术治疗的绝对适应证答案:A , B , C , D1 4 1 4 、春嘉京道口异常,尿道下裂可分哪些类型A 、阴茎头型 B 、阴茎型 C 、阴囊型 D 、会阴型 E 、球部尿道型答案:A , B , C , D1 4 1 5 、肾损伤的主要症状是A 、发热 B 、血尿 C 、休克 D 、腰部肿块 E 、腰腹部肿块答 案 : A , B , C , D , E14 16 、急性肾盂肾炎的主要症状

197、有A、发热 B 、肉眼血尿 C 、腰痛 D 、尿少或无尿 E 、膀胱刺激症状答案:A , C . E14 17 、皮质醇症的主要临床表现包括A、向心型肥胖 B 、高血压 C 、糖尿病 D 、性腺功能混乱 E 、皮肤菲薄和多毛答 案 : A , B , C , D , E14 18 、原发性醛固酮增多症的主要临床表现有A、高血压 B 、烦渴多尿 C 、满月脸 D 、肌无力 E 、糖尿病答案:A , B . D14 19 、正常精液指标包括A 、乳白色不透明,有相当粘度 B 、5 3 0分钟内液化 C 、pH 7 8 D 、精子计数不少于2 000万 / m L , 精子活动度超过60 %,正常形

198、态精子超过6 0% E 、2 -6 m L答案:A , B , C , D , E14 2 0、隐睾的危险为A 、造成不育 B 、隐睾恶变 C 、睾丸扭转 D 、影响心理健康 E 、易并发腹股沟疝答案:A . B14 2 1、跌倒手撑地,可能发生A 、柯雷骨折 B 、尺横骨双骨折 C 、肩胛冈骨折 D 、锁骨骨折 E 、肩关节脱位答案:A , B , D , E14 2 2 、肱骨牌上骨折可压迫A、尺神经 B 、税神经 C 、正中神经 D 、腋动脉 E 、腋神经答案:A , B . C14 2 3 、肩关节脱位用足蹬法复位的要点是A 、病人仰卧床边,术者一足置伤侧腋窝,双手握腕部作对抗牵引 B

199、 、左肩脱位术者用右足,右肩脱位术者用左足 C 、左肩脱位术者用左足,右肩脱位术者用右足D 、牵引时逐渐外展外旋患肢,直至复位 E 、牵引时逐渐内收内旋患肢,直至复位答案:A , C . E14 2 4 、手部清创时对伤口和深部组织损伤的处理为A 、创面新鲜清洁时,清创后同时修复深部组织损伤和缝合伤口 B 、创面污染重,清创后缝合伤口,二期手术修复肌腱、神经损伤 C 、创面污染重,清创后缝合伤,二期手术修复骨折和脱位 D 、 尽管创面污染重, 清创后也要同时处理手部骨折和脱位 E 、受伤时间较长,污染严重的伤口,清创后延期缝合伤口答案:A , B , E1 4 2 5、对手部清创术,下列哪项是

200、正确的A、争取在伤后6 8 小时内进行 B 、争取在伤后1 0 1 2 小时内进行 C 、手指外伤可用指根麻醉, 局部麻醉药内加肾上腺素可延长麻醉时间 D 、 手外伤范围较广泛者,应用臂丛麻醉 E、术中尽量不用气囊止血带,以防影响对组织活力的判断答案:A .D1 4 2 6、关于骨折合并神经血管损伤,下列哪项是正确的A 、腓骨颈骨折可合并腓总神经损伤 B 、股骨下段骨折可合并坐骨神经 C 、肱骨中段骨折可合并槎神经损伤 D 、胫骨上段骨折可合并腓动脉损伤 E、股骨颈骨折可合并坐骨神经损伤答案:A, B , C1 4 2 7 、2 : 9男性病人,外伤后自觉腰背痛,摄片发现胸1 2 压缩性骨折,

201、此处骨折较常见,其原因有A 、两个生理弧度交汇处 B 、此处骨质疏松 C 、此处活动度大 D 、此处应力较集中 E、稳定的胸椎与活动度大的腰椎交界区答案:A , C , D , E1 4 2 8 、加拿反摩适合于下面哪几型颈椎病的保守治疗A 、脊髓型颈椎病 B 、食管型颈椎病 C 、交感神经型颈椎病 D 、椎动脉型颈椎病 E、神经根型颈椎病答案:B , C , D , E1 4 2 9 、急性血源性化脓性骨髓炎治疗原则为A 、增加全身抵抗力 B 、联合使用大量抗生素 C 、局部外固定 D 、病灶清除 E、病灶冲洗引流答 案 : A , B , C , E1 4 3 0、慢性化脓性骨髓炎手术目的

202、为A 、死骨分离且包壳充分形成作死骨摘除 B 、窦道引流不畅时宜扩大引流 C 、窦道周围皮肤恶变需截肢 D 、病理性骨折考虑病灶清除,并行手术复位内固定E、经久不愈且引起恶病质者行截肢答案:A , B , C , E1 4 3 1 、治疗4 8 岁的先天性髓关节脱位的正确方法是A 、手法复位和蛙式石膏固定 B 、切开复位或s al t e r ( 沙尔待) 骨盆旋转截骨术新版医生三基题库第1 0 9 页C 、行C hi a ri ( 查理) 骨盆内移截骨术 D 、可附加股骨转子下截骨术 E 、懿臼上方加盖术答 案 : B, D , E1 432 、重型胎盘早剥的处理是A 、密切观察血压、脉搏、

203、宫底高度等病情变化 B、输液、输血纠正休克 C 、用止血药物止血 D 、人工破膜 E 、积极终止妊娠答案:A , B. E1 433、诊断葡萄胎的方法有A 、病史、体征 B、H C G 测定 C 、A F P 测定 D 、超声波检查 E 、C A 1 2 5测定答案:A , B, D1 434、产科病房母乳喂养的规定包括A 、 早吸吮 B、 2 4小时同室 C 、 每天喂奶68次 D 、 开奶前不喂食 E 、婴儿吸吮困难时用奶瓶喂答案:A , B, D1 435、疝席病性手足搐搦症主要由于A、维生素D 缺乏,骨样组织钙化不良 B、腹泻肠吸收钙减少 C 、维生素D 缺乏时采用突击疗法后诱发 D

204、、 维生素D 缺乏而甲状旁腺反应迟钝 E 、 长期服用苯妥英钠类药物,促进维生素D 分解失去活性答案:D1 436、腺病毒肺炎的临床特点是A 、多为稽留热 B、肺部体征出现较晚 C 、早期即有全身中毒症状 D 、喘憋,呼吸困难 E 、可并发渗出性胸膜炎答 案 : A , B, C , D , E1 437、小儿髓外造血表现为A 、肝脾淋巴结肿大 B、黄骨髓参与造血 C 、周围血象可见幼红细胞和/或幼稚粒细胞 D 、H b F 明显升高 E 、周围血中出现异形淋巴细胞答案:A , C1 4 3 8 . . 常规检查中白细胞分类计数在传染病诊断中的正确概念是A 、白细胞数显著增多常见于流脑、败血症

205、、猩红热 B 、伤寒、副伤寒与布氏菌病白细胞数正常或减少 C 、流感、登革热、病毒性肝炎时白细胞数常减少或正常D 、寄生虫感染时嗜酸性粒细胞增多 E 、嗜酸性粒细胞减少见于伤寒、流脑答 案 : A , B , C , E1 4 3 9 、下列哪项属于主动免疫制剂A、疫苗 B 、菌苗 C 、抗毒素 D 、类毒素 E 、丙种球蛋白答案:A , B .D1 4 4 0、根据我国传染病防治法,对下列哪些疾病应采取甲类传染病的预防、控制措施A 、鼠疫病人及病原携带者 B 、霍乱病人及病原携带者 C 、艾滋病人 D 、肺炭疽病人 E 、麻风病答案:A , B , C , D , E1 4 4 1 、艾滋病

206、的传播方式包括新版医生三基题库第1 1 0页A 、 性接触传播 B 、 注射途径传播 C 、 母婴传播 D 、 器官移植传播 E 、人工授精传播答 案 : A , B , C , D , E1 4 4 2 、H B V 基因组织负链中的开放读码框架包括A 、S 区 B 、C 区 C 、P区 D 、X 区 E 、M 区答案:A, B , C , D1 4 4 3 、端弓盍慢性病毒携带者的肝炎病毒是A、 H A V B 、 H B V C 、 H C V D 、 H D V E 、 H E V答案:B, C , D1 4 4 4 、6虚种丙种球蛋白进行被动免疫预防的疾病是A 、甲型病毒性肝炎密切接

207、触者 B 、麻疹密切接触者 C 、丙型病毒性肝炎密切接触者 D 、脊髓灰质炎密切接触者 E 、戊型病毒性肝炎密切接触者答案:A , B .D1 4 4 5 、可从血培养获得病原体的是A 、败血症 B 、菌血症 C 、毒血症 D 、脓毒血症 E 、变应性亚败血症答案:A , B , C , D1 4 4 6 、弓 I 总9袭性腹泻,导致排粘液血便的病原体是A、志贺菌 B 、 空肠弯曲菌伤寒沙门菌C 、 侵袭性大肠埃希菌 D 、 葡萄球菌 E 、答案:A , B . C1 4 4 7 、引起分泌性腹泻,导致排水样便的病原体是A 、葡萄球菌 B 、霍乱弧菌 C 、产肠毒素性大肠埃希菌 D 、肉毒杆菌

208、E 、幽门螺杆菌答案:A , B . C1 4 4 8 、脑血栓形成在急性期的治疗方法可选用A 、抗血小板粘附聚集 B 、尼莫地平拮抗细胞内钙超载 C 、头部或全身亚低温治疗 D 、降纤酶降纤治疗 E 、用依达拉奉清除氧自由基答 案 : A , B , C , D , E1 4 4 9 、深昏迷的临床表现包括A、肌肉松弛 B 、B a b i n s k i 征阳性 C 、生命体征无变化 D 、角膜反射消失 E 、腱反射存在答案:A . D1 4 50 、根性感觉障碍表现A 、受损范围呈带状 B 、出现感觉分离 C 、投射性疼痛 D 、症状在病变对侧 E 、受损范围呈节段性答案:A . C1

209、4 51 、腰穿的禁忌证为A 、小脑肿瘤 B 、病毒性脑膜炎 C 、腰椎外伤畸形并颅内感染 D 、蛛网膜下腔出血 E 、腰部局部皮肤发炎答案:A . C . E新版医生三基题库 第1 1 1 页1 4 52 、坐骨神经痛的临床表现是A 、沿坐骨神经经路的典型放射性疼痛 B 、疼痛位于臀部,并向股后部、小腿后外侧、足外侧放射 C 、呈持续性钝痛,并有阵发性加剧 D 、可为刀割样或灼样痛,夜间常加重 E 、病变多为单侧性答案:A , B , C , D , E1 4 53 、原发性三叉神经痛的临床表现是A 、多发生于中老年人,女略多于男 B 、疼痛限于三叉神经分布区的一支或两支,以第二、第三支最多

210、见,三支同时受累者极为罕见 C 、通常无预兆,开始和停止都很突然,间歇期可完全正常 D 、病程可呈周期性,每次发作期可为数日、数周或数月不等 E 、神经系统检查一般无阳性体征答 案 : A, B , C , D , E1 4 5 4 、急性脊髓炎的临床表现A、病前常有感染或疫苗接种史 B 、急性起病,较早出现脊髓休克 C 、损害平面以下传导束型感觉障碍 D 、脑脊液压力增高明显 E 、可有大小便功能障碍答 案 : A, B , C , D , E1 4 5 5 、以下对脑血栓形成的描述不正确的是A、活动中发病较多 B 、发病年龄多在6 0 岁以上 C 、脑脊液无色透明 D 、颅内压增高明显 E

211、 、因其起病速度较快,故多数病人意识障碍较重答案:A, D . E1 4 5 6 、对高血压动脉硬化性脑出血急性期血压处理正确的是A、首选氯丙嗪注射 B 、快速降压 C 、缓慢降压 D 、降至正常血压数值E 、降压速度不宜过快,根据情况确定最佳水平答案:C . E1 4 5 7 、血栓形成性脑梗死和脑出血最具有鉴别意义的是A、发病年龄 B 、起病状态 C 、起病速度 D 、有无高血压病史 E 、神经体征答案:B , C1 4 5 8 、病毒性脑膜炎常见的症状是A、发热 B 、头痛 C 、脑膜刺激征 D 、全身中毒症状 E 、严重的脑实质受损的症状答 案 . A R C D1 4 5 9 :捻

212、和 i 癫痫的特点是A、病年龄较小 B 、脑内未发现器质性病变 C 、以全面性发作为主 D 、以部分性发作为主 E 、因大脑半球病变引起的癫痫发作答 案 : A, B , C , D1 4 6 0 、器质性精神障碍的特点是A、有明显的器质性病因 B 、有明显的病理形态学改变 C 、有智能、记忆、人格、意识障碍 D 、精神刺激常是主要病因 E 、可包括脑外伤性精神障碍、躯体疾病所致的精神障碍、中毒性精神障碍答案:A, B , C , E1 4 6 1 、下列哪些是影响精神分裂症预后差的因素新版医生三基题库 第1 1 2 页A、 有明显的阳性家族史 B、 有分裂样的性格特征( 敏感多疑, 依赖性大

213、, 离群孤独,固执任性) C 、发病年龄较早 D 、起病隐袭,反复发作 E 、早期发现,早期治疗答案:A, B, C , D1 4 62 、鼠而脸裂症青春型的特点有A 、思维奔逸,情绪高涨,动作增多 B、思维破裂离奇,令人难以理解 C 、情绪不稳,喜怒无常,行为紊乱,动作愚蠢 D 、性格改变,智力减退 E 、定向障碍答案:B, C1 4 61 、山感淡漠是精神分裂症的常见症状,由于主要通过观察来判断,给临床工作带来, 定的难度,但可通过以下方面来进行观察A、面部表情 B、自主动作 C 、姿势性语言 D 、目光对视 E 、反应灵活性答案:A , B, C , D , E1 4 64 、心境障碍中

214、,常见的情绪障碍有A 、情绪不稳 B、情绪低落 C 、情感淡漠 D 、情绪高涨 E 、情感倒错答案:B.D1 4 65 、抑郁症发作除典型的三联症外,常伴有下列症状A 、兴趣缺乏 B、精力不足,易疲劳 C 、言语少,动作迟缓 D 、自责和厌食 E 、性欲下降答 案 : A , B, C , D , E1 4 66、躁狂发作除了典型的三联症外,还可有A 、自我评价过高 B、易激惹 C 、睡眠需要减少 D 、食欲增加,性欲亢进 E 、爱管闲事答案:A , B, C , D , E1 4 67、神经衰弱的主要临床表现A 、精神易兴奋,脑力易疲劳 B、头昏、头痛、睡眠障碍 C 、多种躯体不适感和焦虑的

215、情绪 D 、有不同程度的意识障碍 E 、人格障碍答 案 : A , B.C1 4 68、下列哪些症状属广泛性焦虑症A 、过分的紧张、恐惧、心慌、意乱、惶惶不可终日 B、局促不安,来回走动,搓手顿足,坐卧不宁 C 、思维迟缓,反应迟钝,无进取心 D 、胸闷、气促、尿频、尿急、口干、出汗、面红耳赤、胡思乱想 E 、反复的洗手、关门答案:A , B , D1 4 6 9、艾滋病的传播途径有A 、性接触 B 、血及血制品传染 C 、共用HI V 污染的注射器和针头 D 、母婴传染 E 、消化道传染答案:A, B , C , D1 4 7 0 、需“ 百犯部位不同,假丝酵母菌可分为哪几种类型A 、肺念珠

216、菌病 B 、内脏念珠菌病 C 、皮肤念珠菌病 D 、肠道念珠菌病新版医生三基题库第1 1 3 页E 、粘膜念珠菌病答案:B . C . E1 4 7 1 、梅毒血清试验目前较常用的方法为A、V D R L 法 B 、U S R 法或R P P 法 C 、T P P A 法 D 、F T A - A B S t e st 法 E 、K a h n t e st 法和W a sse r m a n t e st 法答案:A , B , C , D1 4 7 2 、需亲而床特点,有关下述几种疾病分型的叙述哪些是正确的A 、皮肌炎分为3 型 B 、红斑狼疮分为3 型 C 、足癣分为3 型 I) 、银屑

217、病分为4 型 E 、天疱疮分为5型答 案 :B , C , D , E1 4 7 3 、重型药疹包括哪几种A 、固定型药疹 B 、麻疹样药疹 C 、重症多形红斑型药疹 D 、大疱性表皮松解型药疹 E 、剥脱性皮炎型药疹答案:C , D , E1 4 7 4 、交感性眼炎的病因未明,但其诱因是A 、眼球穿孔伤 B 、眼眶骨折 C 、眼内出血 D 、眼球挫伤 E 、眼内异物残留答案:A1 4 7 5、角膜的组织结构特点是A 、具有透明性 B 、含丰富的感觉神经 C 、无血管 D 、角膜前面有泪膜E 、由致密且相互交错的胶原纤维组成答 案 :A , B . C1 4 7 6 、视力障碍包括A 、视力

218、下降 B 、夜盲 C 、复视 D 、视野缩小 E 、眼前黑影飘动答案:A , B , C , D , E1 4 7 7 、睑板腺囊肿的治疗方法为A 、可定期观察 B 、局部热敷 C 、局部应用糖皮质激素 D 、手术切除E 、全身应用糖皮质激素答 案 : A , B , C , D1 4 7 8 、急性泪囊炎治疗方法正确的是A 、早期局部热敷 B 、全身和局部使用敏感抗生素 C 、尽快行泪道冲洗,排出泪囊内的脓液 D 、脓肿形成后应尽快切开排脓 E 、应尽快行泪囊鼻腔吻合术答案:A , B .D1 4 7 9 、结膜炎常见的眼部临床表现是A 、发痒 B 、异物感 C 、烧灼感 D 、流泪 E 、

219、结膜充血答案:A ,B ,C ,D ,E1 4 8 0 、角膜疾病主要有A、炎症 B 、外伤 C 、先天性异常 D 、变性和营养不良 E 、肿瘤答 案 : A , B , C , D , E1 4 8 1 、下面哪些疾病可以引起并发性白内障A、葡萄膜炎 B 、青光眼 C 、视网膜脱离 D 、糖尿病 E 、眼球挫伤新版医生三基题库第1 1 4 页答 案 : A ,B ,C1 4 8 2 、属于青光眼危险因素的有A、高眼压 B 、糖尿病 C 、心血管疾病 D 、近视眼 E 、青光眼家族史答案:A , B , C , D , E1 4 8 3 、下列叙述正确的是A、正常人眼压呈正态分布 B 、正常人

220、眼压通常在1 0 2 1mmH g C 、不能认为2 1 m m H g 的眼压为病理值 D 、2 1 m m H g 的可能只是高眼压症 E 、眼压在1 0 2 1 m m H g 属于安全眼压,不会发生青光眼答案:B .C .D1 4 8 4 、视网膜中央动脉阻塞的临床特征是A 、一眼突然发生无痛性完全失明 B 、常见视网膜出血 C 、樱桃红斑 D 、视网膜动脉变细 E 、视网膜混浊水肿答案:A , C , D , E1 4 8 5 、以下哪些疾病可导致视神经炎A、脱髓鞘疾病 B 、麻疹 C 、葡萄膜炎 D 、带状疱疹 E 、脑炎答 案 : A , B , C , D , E1 4 8 6

221、 、屈光不正包括A、老视 B 、近视 C 、远视 D 、散光 E 、屈光参差答案:B ,C ,D ,E1 4 8 7 、准分子激光角膜切削术的缺点是A 、远视治疗的预测性差 B 、手术难度大 C 、高度近视回退较大度近视的疗效比中低度差 E 、以上都不是D 、高答案:A .C1 4 8 8 、由糖尿病引起的眼部并发症有A 、虹膜红变 B 、新生血管性青光眼 C 、虹膜睫状体炎 D 、晶状体屈光度变化 E、白内障答案:A , B , C , D , E1 4 8 9 、耳源性颅内并发症有A、脑膜炎 B 、迷路炎 C 、脑脓肿 D 、面瘫 E、B e s e l d 脓肿答案:A , C1 4 9

222、 0 、良出血的主要局部原因包括A 、鼻和鼻窦外伤 B 、鼻中隔疾病 C 、鼻腔炎症 I ) 、肿瘤 E、变应性鼻炎答 案 : A , B , C , D1 4 9 1 、诊断腺样体肥大,下列哪些检查有价值A、X 线鼻咽侧位片 B 、C T 扫描 C 、鼻咽部触诊 D 、颈部触诊 E、纤维鼻咽镜检查答 案 : A , B , C , E1 4 9 2 、鼻咽癌的发病特点为A 、无地域分布及种族分布特点B 、黄种人鼻咽癌发病率高C 、鼻咽癌高发于新版医生三基题库第1 1 5 页少年 D 、鼻咽癌与EB 病毒感染有密切关系 E、鼻咽癌与饮酒有密切关系答案:B .D1 4 9 3 、急性会厌炎的临床

223、表现为A、发热 B 、咽喉痛 C 、咳血 I ) 、呼吸困难答案:A ,B ,D ,E1 4 9 4 、下列哪些部位癌可称为声门上型癌A 、会厌 B 、真声带答案:A , C , D , E1 4 9 5 、具有升颌作用的肌肉有A 、翼内肌 B 、翼外肌答案:A , C , D1 4 9 6 、口腔超声洁治不宜用于A 、放置心脏起搏器的病人C 、室带 D 、杓会厌裳C 、咬肌 D 、频肌B 、乙型病毒性肝炎病人E、吞咽困难E、杓间区E、二腹肌C 、肺结核病人D 、肿瘤病人 E、艾滋病病人答案:A , B , C , E1 4 9 7 、弓冠牙齿楔状缺损的原因有A 、细菌感染 B 、横刷牙法刷牙

224、 C 、过多食用糖类 D 、牙颈部结构薄弱E、牙颈部应力集中答案:B .D .E1 4 9 8 、以下哪些指标是用于确定牙周组织破坏程度的A 、牙石量 B 、牙龈炎症程度 C 、牙周袋深度 D 、结缔组织附着丧失量E、牙槽骨吸收程度答案:C , D .E1 4 9 9 、牙根距离上颌窦底很近,拔牙时易将牙根推入上颌窦内的牙齿有A 、上颌中切牙 B 、上颌双尖牙 C 、上颌第一磨牙 D 、上颌第二磨牙E 、上颌尖牙. B , C , D1 5 0 0 .9的儿上颌骨骨髓炎的感染途径主要有A 、血源性感染 B 、 外伤性感染 C 、 接触性感染 D 、 牙源性感染 E 、医源性感染答案:A , B

225、 .C1 5 0 1 、下颌骨骨折的临床表现可有A 、骨折段移位咬合错乱 B 、张口受限 C 、眼镜征 D 、骨折的异常活动E 、上唇麻木答案:A , B , D1 5 0 2 、颍下颌关节紊乱病的发病因素包括A、精神因素 B 、 社会心理因素 C 、 外伤及微小创伤 D 、 始因素 E 、免疫因素答 案 : A , B , C , D , E1 5 0 3 , 颌骨囊肿包括A 、根端囊肿 B 、始基囊肿C 、胎囊肿 D 、鳏裂囊肿 E 、角化囊肿答案:A , B , C , E1 5 0 4 、有关腺样囊性癌描述正确的有A 、又名圆柱瘤 B 、浸润性不强 C 、血行转移率高 I) 、区域淋巴

226、结转移率高 E 、肿瘤细胞常沿神经生长答案:A .C .E1 5 0 5 、高压氧治疗气性坏疽的作用是A 、抑制梭状芽胞杆菌的生长 B 、抑制a - 外毒素的产生 C 、阻止组织坏死,促进伤口愈合 D 、增强抗毒血清的作用 E 、增强抗生素的效力答案:A , B , C1 5 0 6 、横旋型氧中毒发生的原因可能是A 、压力在0 . 2 5 M P a 上 B 、脑内酪氨酸生成减少 C 、脑 内 浓 度 升 高 D 、常压下持续吸氧超过8 小时 E 、乙酰胆碱酯酶活性降低答案:A , B , C , E1 5 0 7 、医用氧气的质量标准应达到A、无杂质,无有害气体 B 、氧浓度不少于9 9

227、. 5 % C 、水汽不高于5 m L /瓶D 、温度不高于2 2 c E 、二氧化碳浓度不高于0 .0 5 %答案:A , B , C , E1 5 0 8 、高压氧对循环系统的影响包括A 、 心率减慢 B 、 心排血量减少 C 、 血流减慢 D 、 心脏负荷加重 E 、血循环时间缩短答 案 : A , B . C1 5 0 9 、高压氧治疗气性坏疽的治疗原则是A 、 , 经确诊,简单清创,立即行高压氧治疗 B 、对疑似气性坏疽病人也应做预防治疗 C 、应同时使用广谱抗生素及注射抗毒血清 D 、待截肢后再行高压氧治疗 E 、体温超过4 0 时不宜行高压氧治疗答案:A , B , C1 5 1

228、 0 、人在高气压环境下并不会被压扁,这是因为A 、人体是有弹性的 B 、水的不可压缩性 C 、人体有强大骨架的支持 D 、人体各部位均匀受压 E 、高压氧治疗的压力人体尚可耐受答案:B . D1 5 1 1 、在高压氧下哪些微生物生长会受到抑制A、厌氧菌 B 、某些兼性厌氧菌 C 、某些需氧菌 D 、各种细菌 E 、病毒答 案 : A , B . C1 5 1 2 、高压氧对血液系统的影响主要包括A 、红细胞减少 B 、红细胞沉降率加快 C 、白细胞增加 D 、凝血时间延长 E 、血液粘度下降答案:A , B , C , D1 5 1 3 、氧瓶使用后,瓶内应保留l k g / c m /

229、的剩余压力,目的在于A 、表明该瓶未作过其他用途 B 、外界杂质不易进入瓶内 C 、再充气时,瓶无需清洗 D 、保护减压器不易损坏 E 、备取样验证气体性质新版医生三基题库第1 1 6 页答案:A . B . C1 5 1 4 、影响减压病发生的因素包括A 、机体所受压力的大小 B 、高压下暴露时间 C 、减压速度 D 、环境温度 E 、病人体质答案:A , B , C , D1 5 1 5 、惊厥型氧中毒可发生在A 、0 . 2 M P a 以上高压氧治疗吸氧时 B 、常压下持续吸氧8 小时以上 C 、0 . 2 M P a以上高压氧治疗的减压过程中 D 、 高压氧治疗吸氧停止4 小时以后

230、E 、 0 . 1 5 M P a下吸氧2 小时以上答案:A . C1 5 1 6 、恶性肿瘤全身转移的治疗包括A 、化疗 B 、手术治疗 C 、免疫治疗 D 、放疗 E 、中医药治疗答案:A , C , E 7 、, 加骤停的诊断标准是A 、大动脉摸不到搏动 B 、瞳孔散大 C 、心音遥远 D 、神志突然丧失E 、抽搐答 案 : A , B , D1 5 1 8 、海水淹溺时,其水、电解质紊乱的原因是A、严重低血容量 B 、血液稀释 C 、血钠增高 D 、镁浓度增加 E 、血浆蛋白减少答案:A , C , D , E1 5 1 9 、张力性气胸病人A 、胸腔抽气后压力不再上升 B 、肺萎陷轻

231、 C 、纵隔移位明显 D 、胸腔压力常呈正压 E 、常需采用胸腔闭式引流答案:C , D , E1 5 2 0 、急性肾小管坏死少尿期尿液检查常表现为A 、尿量4 0 0 ni L / 2 4 h , 或正常 B 、尿相对密度多在L 0 1 5 以下 C 、尿渗透压浓度3 5 0 mmo l / L D 、尿钠含量降低,多在4 0 mmo l / L 以下 E 、尿肌酊与血肌酊之比常 1 0答案:A , B . E1 5 2 1 、急性肾衰竭高钾血症最有效的处理方法是A 、限制入水量,使中心静脉压维持在6 l O c m乂 O 之间 B 、血液透析 C 、注意补镁 D 、静脉缓慢注射钙剂 E

232、、服用利尿药螺内酯答案:A , B . D1 5 2 2 、急性肾小管坏死常见的病因是A 、急性失血性休克 B 、蛇毒、鱼胆及毒蕈等生物毒素中毒 C 、异型输血D 、急性药物过敏 E 、急性肾小球性肾炎答 案 . A B C D1 5 2 3 :血 缶 , 经毒类毒蛇的是A、竹叶青 B 、银环蛇 C 、眼镜蛇 D 、五步蛇 E 、金环蛇答案:B . E新版医生三基题库 第1 1 7 页1 5 2 4 、大量输血时可致A 、心脏负荷加重 B 、凝血异常 C 、枸椽酸中毒 D 、高脂血症 E 、低蛋白血症答案:A , B . C1 5 2 5 、A B O 血型改变” 可见于A、肠道腺癌病人 B

233、、白血病病人 C 、 骨髓瘤病人 D 、 肺炎病人 E 、自身免疫性溶血性贫血病人答案:A , B , C1 5 2 6 、失神的表现有A 、神昏澹语、循衣摸床 B 、精神倦怠、气短懒言 C 、卒倒神昏、手撒遗尿D 、突然神清、喋喋多言 E 、反应迟钝、面色少华答案:A . C1 5 2 7 、张某,男,7 5 岁。久泻未愈,每天黎明前登厕,泻下清稀,形寒肢冷,腰膝酸软,苔白,脉沉细,治法应该用A 、健脾 B 、理气 C 、温肾 D 、固涩 E 、滋阴答案:A , C . D1 5 2 8 、 卜二经脉的互相衔接,下述哪些是正确的A 、阴经与阳经在四肢 B 、阴经与阴经在胸部 C 、阳经与阳经

234、在头部 D 、表里在胸腹部 E 、阴经与阳经在腹部答案:A , B , C1 5 2 9 、下列标本,在排除外界污染的情况下,培养出细菌即有确诊意义的是A 、血 B 、粪便 C 、脑脊液答案:A . C1 5 3 0 、可使血小板数升高的因素有A 、运动 B 、新生儿 C 、饱餐D 、咽拭子 E 、痰D 、妇女月经前 E 、脾功能亢进答案:A , C1 5 3 1 、条离子型碘制剂是A、泛影葡胺 B 、胆影葡胺 C 、磁显葡胺D 、欧乃派克 E 、优维显答案:D , E1 5 3 2 、M R I 成像与C T 扫描相比较,具有的优点是A 、多参数成像 B 、可获取任何方位图像D 、钙化病变显

235、示清楚 E 、成像速度快答案:A , B , C1 5 3 3 、5血异物严禁进入M R I 扫描区,是为了避免A 、磁场对人体的损伤 B 、磁场强度减低振信号过于增强 E 、幽闭恐怖症答案:A , C15 3 4 、血型的胆囊结石图像表现A 、胆囊内出现强光团 B、强光团后方有声影C 、” 流空效应使血管直接显影C 、磁场均匀度破坏 D 、磁共C 、未粘连或嵌顿者强光团可随体位改变位置 D 、探头下存在压痛答案:A , B. CE 、增厚的胆囊壁内出现小的囊泡状喑区新版医生三基题库第118 页15 3 5 、胎儿死亡的非特异性指征A、羊水混淆 B、大脑镰消失 C 、脑室扩大 D 、双顶径缩小

236、 E 、胎头双环轮廓答案:A . B. D . E15 3 6 、二尖瓣狭窄的二维切面声像特征A、瓣叶增厚 B、开放受限呈 弓形 C 、左房右室扩大 D 、二尖瓣前叶舒张期呈现城墙样改变 E 、瓣口狭小,有时可见左房附壁血栓答案:A , B, C , D15 3 7 、慢性支气管炎可导致A、支气管扩张症 B、肺水肿 C 、支气管腔狭窄 D 、肺癌 E 、肺出血性梗死答案:A, B, C15 3 8 、扇 i 萎缩性胃炎的病变特点是A 、腺体减少并有囊性扩张 B、肠上皮化生 C 、粘膜固有层内淋巴细胞、浆细胞浸润 D 、胃穿孔 E 、并发幽门瘢痕形成答案:A , B. C15 3 9 、鼻咽癌的

237、特点是A 、早期鼻咽部就有明显肿块 B、组织学类型以低分化鳞癌多见 C 、往往早期发生淋巴管转移 D 、涕血 E 、头痛、耳鸣答案:B. C , D . E15 4 0 、核医学检查的特点是A 、一种功能性显像,对疾病可进行早期诊断 B、 - 种特异性显像方法 C 、既可显示解剖结构改变,又能进行动态功能的观察 D 、安全非创伤性检查 E 、主要缺点是价格昂贵答案:A B C D1 5 4 1 , 方强4 核素动态显像,其图像分析的要点是A 、掌握受检脏器显像的顺序 B 、观察显像图像放射性浓聚程度 C 、注意时相变化 D 、观察脏器的位置、大小和形态 E 、观察病变与周围组织对比情况答案:A

238、 . C1 5 4 2 、骨骼核素显像的适应证有A 、寻找恶性肿瘤的早期转移病灶 B 、判断骨肿瘤的部位、范围 C 、诊断外伤性骨折 D 、早期骨髓炎与蜂窝织炎鉴别诊断 E 、对关节疾病、代谢性骨病等早期判断答案:A , B , C , D , E1 5 4 3 、以拳看孑抗氧化的微量营养素是A、硒 B 、维生素D C 、维生素B D 、维生素C E 、维生素E答案:A , D , E1 5 4 4 、膳食纤维包括A、纤维素 B 、半纤维素 C 、果胶 D 、木质素 E 、抗性淀粉答案:A , B , C , D1 5 4 5 、痛风的营养治疗原则有新版医生三基题库第1 1 9 页A、限制嘿吟

239、 B 、 低热量饮食 C 、 低脂饮食 D 、 摄入充足的液体 E 、避免饮酒及酒精饮料答案:A , B , C , D , E1 5 4 6 、以下促肠功能代偿的物质有A、中、短链脂肪酸 B 、纤维素( 特别是果胶) C 、谷氨酰胺 D 、蛋氨酸E 、亮氨酸答案:ABC1 5 4 7 . 而夕卜营养的并发症有A、气胸 B 、空气栓塞 C 、血胸 D 、导管性败血症 E 、误吸答案:A , B , C , D1 5 4 8 、: 翥后肠内营养制剂的特点是A 、营养全面 B 、无需消化即可直接或接近直接吸收 C 、成分明确 D 、不含残渣或残渣极少 E 、适口性差答 案 : A , B , C

240、, D , E1 5 4 9 、医院感染的概念,下列哪项正确A 、医院感染的形式可以是显性感染,也可以是隐性感染 B 、医院感染可来自别人,也可来自自身 C 、医院感染有的可以预防,有的不可以预防 D 、医院感染可发生在入院前,也可发生在住院中 E 、医院感染中新生儿感染可来自产道,也可来自宫内答案:A , B .C1 5 5 0 、人体正常菌丛的作用有下列几项A 、抵制病原菌的入侵 B 、提高机体免疫力 C 、合成人体需要的部分维生素D 、引起自身感染 E 、合成抗生素答案:A , B , C , D1 5 5 1 、输血可引起哪些感染A、梅毒 B 、丙型肝炎 C 、巨细胞病毒感染 D 、弓

241、形体病 E 、伤寒答案:A , B , C , D1 5 5 2 、卡歹布毒剂中属于杀菌水平者是A、石炭酸 B 、环氧乙烷 C 、乙醇 D 、过氧化氢 E 、苯扎溪镂答案:B .D1 5 5 3 、下列消毒剂中属于低效水平者是A、聚维酮碘 B 、氯己咤 C 、次氯酸盐 D 、苯扎澳筱 E 、戊二醛答案:B .D1 5 5 4 、产房非限制区包括A、换鞋区 B 、更衣洗涤区 C 、值班室 D 、厕所 E 、医师办公室答案:A , B , C , D1 5 5 5 、设备科对一次性医疗用具的每次购置必须进行质量验收,包括A、查生产许可证 、 卫生许可证 、 推销员 证件 B 、证件与汇寄账号、生产

242、企业一致 C 、检验合格证、消毒日期 D 、进行热原和微生物学检查 E 、严格保管,包装破损或证件不全者不得用于临床答案:A , B , C , E新版医生三基题库第1 2 0 页1 5 5 6 、产房半限制区包括A 、护士办公室 B 、待产室 C 、厕所 D 、洗涤室 E 、敷料准备室答案:A , B , D , E1 5 5 7 、有关母婴同室下列哪项错误A、每个婴儿应有独立床位 B 、病房每床净使用面积应为5 . 5 6 . 5 m ? C 、母乳在4 冰箱内存放时间不得超过4 8 小时 D 、母乳库中应有操作台、冰箱、加热消毒装置E 、不限制探视答案:C , E15 5 8 、看准预防

243、的具体措施有A 、视一切血液、体液均有传染性而采取相应措施 B 、强调病人与医务人员间的双相防护 C 、接触隔离 D 、空气隔离 E 、微粒隔离答案:A , B , C , D , E15 5 9 , 舄庙请疝道内镜进行治疗的有A 、电凝电切息肉 B 、胆道取石 C 、肝癌切除 D 、食管曲张静脉套扎E 、食管曲张静脉碘化剂治疗答案:A , B , D , E15 6 0、阴道镜检查,下述各项中正确的是A 、以复方碘溶液涂布宫颈可发现碘不着色区 B 、遇典型图像可摄像 C 、不会发生并发症 D 、对异常区可做活检 E 、观察血管可加绿色滤光器答 案 . A B D F1 5 6 k 兔 i 主

244、气管镜检查的并发症有A、出血 B 、并发感染 C 、心搏骤停 D 、喉返神经麻痹 E 、气胸答案:A , B , C , E15 6 2 、心室率缓慢的心电图可见于A 、房性期前收缩二联律下传受阻 B 、窦性心动过缓 C 、房颤伴HI 度A V B 交界区自转性心律 D 、室性自转性心律 E 、非阵速室性自转性心律答 案 : A , B , C , D15 6 3 、做脑电图前应要求受检者做好下列哪些准备A 、检查前1天用肥皂水洗头 B 、检查前应禁食 C 、检查前1天应停服镇静、安眠药 D 、检查前停用抗癫痫药1 3 天 E 、穿衣质量不受限制答案:A, C , D15 6 4 . & 卞哪

245、些疾病可通过膀胱镜检查及尿路逆行造影明确诊断A、输尿管肿瘤 B 、输尿管透X 线结石 C 、输尿管狭窄 D 、先天性巨输尿管症 E 、肾囊肿答案:A , B , C , D1 5 6 5 、以下哪些情况属于膀胱镜检查禁忌证A 、前列腺肥大症 B 、尿道狭窄 C 、急性膀胱炎及急性尿道炎 D 、膀胱肿瘤 E 、结核性挛缩膀胱答 案 : B , C .E1 5 6 6 、与眼有关的神经包括A 、三叉神经 B 、动眼神经 C 、展神经 D 、滑车神经 E 、面神经新版医生三基题库第1 2 1 页答案:A,B,C,D,E1567、坐骨神经在梨状肌下孔处受损伤后,将表现为伤侧的A、髓关节不能屈 B、骸关

246、节不能伸 C、大腿的皮肤感觉丧失 D、膝关节以下的皮肤感觉丧失 E、膝关节以下运动丧失答案:E1568、两眼瞳孔不等大,左 右,可能由于A、左侧动眼神经损伤 B、右侧颈交感干损伤 C、左侧动眼神经副核损伤D、顶盖前区损伤 E、脊 髓 胸 段 二节右半损伤答 案 : A,B,C,E五、( 共558分) 判断下列说法正确与否,( 纸答卷正确用J 错误用义表示)1569、正常成人脊髓下端达第1腰椎下缘水平。答案:( J )1570、腓总神经损伤后产生的主要症状是足不能背屈、外翻。答案:( J )1571、鱼际肌由正中神经支配而小鱼际肌和蚓状肌由尺神经支配。答案:( X )1572、睫状肌收缩时,睫状

247、小带绷紧,晶状体变凸,适于看近物。答案:( X)1573、男性膀胱底后方,有输尿管越过输精管的前上方。答案:( 义)1574、骨髓分黄骨髓和红骨髓,黄骨髓没有造血潜能。答案:( X)1575、机体内环境相对恒定是指细胞内液的化学成分与理化性质经常在一定范围内变动。答案:( X )1576、体重50Kg的正常人的血液总量为3. 5 4. 0L答 案 : ( V)1577、由于胆汁中含有脂肪酶,所以胆汁促进脂肪的消化和吸收。答案:( X)1578、血液运输C02的主要物质是血红蛋白。答案:( X )1579、呆小病是幼年时生长激素分泌不足。答案:( X)1580、兴奋在神经纤维上的传导是单向的,互

248、不干扰,且不易疲劳。答案:( X )1581、甲状旁腺分泌的降钙素,有使血钙降低的作用。答案:( X)1582、睾丸不能分泌雌激素,卵巢不能分泌雄激素。答案:( X )1583、孕激素有助孕、促进排卵和抑制子宫收缩的作用。答案:( X )1584、内脏痛的特点是有牵涉痛,定位准确。答案:( X)1585、交感神经由中枢发出后直达效应器官,支配效应器官的活动。答案:( X )1586、躯体运动神经属胆碱能神经。答案:( J )1587、呼吸的频率与深浅对肺通气量影响很大。答案:( X )1588、人体只有心肌才有自动节律性。答案:( X)1589、胆囊炎病人吃油腻蛋白食物可诱发胆绞痛。答案:(

249、X)1590、AB0血型是根据血清中所含抗体的不同而命名的。答案:( X )1591、Ca*降低神经肌肉的兴奋性。答案:( V)1592、心肌的有效不应期很长, 0.3 0.4秒。答案:( V)1593、促胰液素可抑制胃液分泌。答案:( J )1594、基础代谢率不是机体最低水平的代谢率。答案:( J )1595、妇女月经流出的血不易形成凝块是因为流出的血液中血小板含量低。答案:( X)1596、使血沉加快的决定性因素在红细胞本身,而不在血浆的变化。答案:( X )1597、毒血症是指病原菌以及它产生的毒素均进入血液。答案:( X)1598、类毒素是外毒素经甲醛处理之后,去其毒性而保留抗原性的

250、用于自动免疫的生物制剂。答案:( J )1599、用高压蒸汽灭菌即可破坏溶液中的热原质。答案:( 义)1600、人体肠道菌群中99. 9%是厌氧菌,大肠杆菌等仅占0.1% 。答案:( J )1601、卡介苗是人型结核分枝杆菌的死菌苗,用于预防结核病。答案:( X)1602、衣原体、立克次体及支原体均属原核细胞型微生物,它们与细菌一样能在无生命培养基上生长。答案:( X )1603、病毒属非细胞型微生物,其增殖方式为自我复制。新版医生三基题库第122页答案:( J )1 6 0 4 、干扰素具有广谱抗病毒的作用,它能直接抑制病毒的复制。答案:( X )1 6 0 5 、免疫是抗体对异己成分的识别

251、及排除抗原性异物的一种特异性生理反应。答案:( V)1 6 0 6 、 临床上常用抗毒素治疗外毒素所致疾病, 而抗毒素进入人体, 它既是抗体又是抗原。答案:( J )1 6 0 7 、异嗜性抗原是一类与种属特异性无关的,存在于人、动物、植物、微生物等中的性质相同的抗原。答案:( V )1 6 0 8 、参与A D C C 效应的免疫球蛋白是Ig G和Ig M 。答案:( X )1 6 0 9 、免疫应答分为3 个阶段,即识别阶段,淋巴细胞活化、分化增殖阶段以及效应阶段。答案:( J )1 6 1 0 、A B 0 血型不合而引起的溶血反应的发生机制属H 型变态反应。答案:( V)1 6 1 1

252、 、初次血清病发生的机制属I 型变态反应。答案:( X )1 6 1 2 、流行性乙型脑炎、狂犬病、钩端螺旋体病均为自然疫源性疾病。答案:( J )1 6 1 3 、艾滋病的病原体是人类免疫缺陷病毒( HIV ) 。答案:( J )1 6 1 4 、甲型肝炎病毒属于肠道病毒7 2 型的D N A 病毒。答案:( X )1 6 1 5 、引起成人T 淋巴细胞白血病的病毒是人类嗜T 细胞病毒,即HT L V 。答案:( V)1 6 1 6 、引起沙眼的病原体是沙眼衣原体。答案:( J )1 6 1 7 、根据近代死亡概念,整体死亡的标志是脑死亡,即全脑功能的永久性消失。答案:( J )1 6 1

253、8 、等渗性脱水病人既有低渗性脱水的部分症状,又有高渗性脱水的部分症状。答案:( J )1 6 1 9 、小儿失钾最重要的原因是经肾失钾。答案:( X )1 6 2 0 、长期输入0 . 9 %氯化钠注射液可引起低钾血症。答案:( V)1 6 2 1 、D 1 C 病人出现贫血的主要原因是因大量出血后,血液稀释所致。答案:( X )1 6 2 2 、在休克期,乂称可逆性失代偿期,微循环出现淤血,病人出现休克的典型症状。答案:( V)1 6 2 3 、动脉瓣膜关闭不全可导致心脏前负荷过重。新版医生三基题库第1 2 3 页答案:( J)1 6 2 4 、左心衰病人可出现肺顺应性降低,使呼吸肌做功和

254、耗能增加。答案:( J)1 6 2 5 、肺性脑病时; 引起脑细胞脱水而导致脑功能障碍。答案:( X )1 6 2 6 、肝性脑病时; 肝脏鸟氨酸循环障碍,导致血氨升高。答 案 : ( V)1 6 2 7 、肺水肿时; 气体弥散距离增大,导致气体弥散障碍。答 案 : ( V)1 6 2 8 、吸入烟雾和毒气可引起急性呼吸窘迫综合征。答案:( J)1 6 2 9 、测定血清转氨酸水平可反映细胞受损状况。答案:( J)1 6 3 0 , 外源性肝性脑病是继发于肝以外的疾病的脑病。答案:( 义)1 6 3 1 、血氨的清除主要是在肝脏经鸟氨酸循环合成尿素,再经肾排出体外。答案:( V)1 6 3 2

255、 、肝性脑病病人若有便秘,最好用肥皂水灌肠。答案:( X )1 6 3 3 、新生儿时由于肝细胞对胆红素的分泌排泄功能不成熟而发生新生儿生理性黄疸。答案:( 义)1 6 3 4 、肾前性急性肾衰竭,常见于各种休克的早期,又称功能性急性肾衰竭。答案:( J)1 6 3 5 、慢性肾衰竭病人,随着病情进一步发展,可出现等渗尿。答 案 : ( V)1 6 3 6 、慢性肾衰竭早期可出现低钾血症。答 案 : ( V)1 6 3 7 、高钾血症可引起代谢性酸中毒。答案:( Q1 6 3 8 、代谢性酸中毒时S B降低。答案:( J)1 6 3 9 、血液性缺氧又称等张性缺氧。答案:( J)1 6 4 0

256、 、当红细胞内C O ? 增多时,氧解离曲线左移。答案:( X )1 6 4 1 、存在于自然界的氨基酸有3 0 0 余种,但组成蛋白质的只有2 0 种。答案:( V )1 6 4 2 、非共价键包括肽键、双硫键、氢键和离子键。答案:( X )1 6 4 3 、血红蛋白与肌红蛋白的氧解离曲线完全相同。答案:( X )新版医生三基题库第1 2 4 页1644、组成DNA的基本碱基是A、C、G、T;而组成RNA的基本碱基是A、C、G、U。答案:( V)1645、Leu、Pro和Thr 3个氨基酸只有1 2个遗传密码。答案:( X)1646、 + S = S = + 尸反应很好地解释了酶的活性中心理

257、论。答案:( X)1647、三痰酸循环反应的第一步是异柠檬酸脱竣变成a - 酮戊二酸。答案:( X)1648、ATP合成酶由底部、头部和柄部组成,含有5个亚基( a 、 B 、 丫、 3 和 。 的 3 亚基可能构成质子通道。答案:( V)1649、人体内只有肝脏是合成胆固醇的场所,其合成原料是丙二酸单酰CoA。答案:( X)1650、高HDL血症会引起动脉粥样硬化症,必须严密检测其血浆水平。答案:( X)1651、药物通过生物膜转运的主要方式是主动转运。答案:( X)1652、新斯的明与毒扁豆碱均能抑制胆碱酯酶,故均用于治疗重症肌无力。答案:( X)1653、阿托品与去氧肾上腺素均可用于扩瞳

258、,但前者可升高眼内压,后者对眼压无明显影响。答案:( V)1654、纠正氯内嗪引起的降压作用,可选用肾上腺素。答案:( X)1655、阿司匹林的镇痛作用部位主要在外周末梢化学感受器。答案:( V)1656、硝酸甘油抗心绞痛的主要原理是选择性扩张冠脉,增加心肌供血供氧。答案:( X)1657、弱酸性药物在碱化尿液后排泄增加。答案:( V)1658、硫喷妥钠维持时间短主要是在肝脏代谢极快。答案:( x )1659、可乐定降压作用的主要机制是直接扩张外周血管。答案:( X)1660、吗啡中毒时用纳洛酮解救。答案:( V)1661、地高辛不宜用于心房颤动。答案:( X)1662、对正在发病的间日疟病人

259、,应给予氯喳加用伯氨喳。答案:( J )1663,与醛固酮产生竞争作用的利尿药是螺内酯。新版医生三基题库第125页答案:( J )1 6 6 4 、为增加利福平的抗结核作用,常与对氨水杨酸同时服用。答案:( X)1 6 6 5 、L V / E D s o 愈大,药物毒性越大。答案:( X)1 6 6 6 、药物产生不良反应的药理学基础是该药作用的选择性低。答 案 : ( V)1 6 6 7 、雷尼替丁能阻断此受体,因而抑制胃酸分泌。答 案 : ( V)1 6 6 8 、糖皮质激素可用于治疗角膜溃疡。答案:( X)1 6 6 9 、氨革西林对青霉素G 的耐药金黄色葡萄球菌有效。答案:( X)1

260、 6 7 0 、初级卫生保健就是低级卫生保健。答案:( 义)1 6 7 1 、初级卫生保健是实现2 0 0 0 年人人健康目标的关键。答案:( J)1 6 7 2 、计数资料和计量资料是不能互相转化的。答案:( J)1 6 7 3 、检验是对两个样本均数的差别作显著性检验的方法之一。答案:( J)1 6 7 4 、统计数据经过显著性检验后,P 值大于0 . 0 5 ,表示两样本的差别无统计学意义。答案:( J)1 6 7 5 、化验结果的阳性或阴性是属于计数资料。答 案 : ( V)1 6 7 6 、如果观察单位相同,均数相近,则标准差大者,说明观察值变异度大。答 案 : ( V)1 6 7

261、7 、癞皮病是由于食物中缺乏维生素P P 引起的。答案:( Q1 6 7 8 、一般植物蛋白质消化率高于动物蛋白质。答案:( X)1 6 7 9 、食物中蛋白质的生物学价值的高低主要取决于必需氨基酸的含量与比值。答案:( J)1 6 8 0 、脂肪的营养价值主要取决于脂肪中饱和脂肪酸的含量。答案:( X)1 6 8 1 、膳食中膳食纤维含量愈高,结肠炎、结肠癌发病率愈高。答案:( X)1 6 8 2 、凡是在尿中查出有毒物质就可诊断为毒物中毒。答案:( 义)1 6 8 3 、接触石棉尘的工人可引起肺癌和胸膜间皮瘤。答案:( V)新版医生三基题库第1 2 6 页1684、高频听力损伤是噪声作业工

262、人的早期听力改变。答案:(J )1685、转科记录应由接收科室的经管医师书写,不另立专页。答案:(X)1686、手术记录应由手术医师书写或指派的第- - 助手记录,并由手术医师审阅后签名,另立专页。答 案 :( V )1687、新病人的入院记录应在入院后24小时完成。答 案 :( V )1688、系统回顾是协助病人回顾被遗漏的所患疾病的方法。答案:( Q1689、有经验的医师一望即知疾病所在,不必询问病史。答案:(X )1690、正常人心尖冲动均可见于左第5肋间锁骨中线内。答案:( 义)1691、为减轻痛苦,病人所采取的体位称强迫体位。答案:(J )1692、皮肤弹性检查,常检查手背及前臂内侧

263、部位。答案:(J )1693、左锁骨上窝淋巴结发现转移性癌症时,原发病灶一定来自胃肠道。答案:( 义)1694、下腔静脉梗阻综合征病人,曲张的腹壁静脉血流方向均向下。答案:(X )1695、头围测量是通过枕骨粗隆和眉间所测的一周长度。答 案 :( V )1696、正常人常见的胸廓横径与前后径之比是1.5:1。答 案 :( V )1697、发现病人呼吸由浅、慢变深、快,然后由深、快变浅、慢,乃至暂停,530秒后,周而复始,称为库斯毛尔( Kussmaul)呼吸。答案:(X )1698、昂伯征主要是查感觉性共济失调。答案:(J )1699、发育正常的人身高等于两手平展之间距离,坐高等于下肢长度。答

264、 案 :( V )1700、哮喘发作端坐呼吸属于被动体位。答案:(X )1701、心界叩诊宜采用轻叩法。答案:(J )1702、肝硬化腹壁静脉曲张时,脐上静脉血流向上,脐下静脉血流向下。答案:( V )1703、吸气时脉搏增强,呼气时减弱,称为奇脉。答案:( X)1704、收缩压 140mmHg,舒张压 90mmHg则为高血压。答案:( J )1705、放止血带后可引起再灌损伤。答案:( J )1706、人工呼吸机通气影响心脏循环的原因是正压通气。答 案 : ( V)1707、止血带压力不足可加重出血。答 案 : ( V)1708、口服磷化锌中毒应服用鸡蛋、牛奶、油类解毒。答案:( X )17

265、09、胸外心脏按压的主要目的是造成胸腔内压力的变化,促使血液循环。答 案 : ( V)1710、检查如发现病人楼动脉搏动消失,应立即心内注射肾上腺素及胸外心脏按压。答案:( 义)1711、皮下注射是将药液注射于表皮和真皮之间。答案:( 义)1712、2岁以下婴幼儿,不宜选用臀大肌进行肌内注射。答案:( J )1713、锁骨下静脉输液时,为防止空气进入血管,不能使输液瓶滴空或使一段输液管低于病人心脏水平。答案:( J )1714、腹腔穿刺抽出迅速凝固的血样液体,说明腹腔内出血。答案:( X)1715、肝脏穿刺活检术,进针深度应超过8cm。答案:( X )1716、骨髓穿刺取材做细胞学检查,抽吸骨

266、髓液要求0. 5mL。答案:( X)1717、决定中心静脉压高低的主要因素是血容量及心脏排血功能。答案:( J )1718、鼻饲流质,每次不宜超过200mL,间隔时间为2小时。答案:( J )1719、气管导管前端套囊,一般注气5mL左右,每6小时应放气1次,5 10分钟后再注气。答 案 : ( V)1720、手术人员洗手消毒时: 化学消毒剂需浸泡至肘上6cm。答案:( V)1721、严重的急性失血性休克,不宜采用动脉切开,动脉输血。答案:( X)1722、清创术适用于新鲜创伤的伤口。答案:( J )1723、临床思维方法是指对疾病现象进行调查研究、分析综合、判断推理等过程中的一系新版医生三基

267、题库第127页列思维活动,由此认识疾病、判断鉴别,做出决策的一种逻辑方法。答案:( V)1 7 2 4 、诊断疾病的步骤包括搜集资料、分析综合资料及形成印象、验证或修正诊断3 个步骤。答案:( J)1 7 2 5、疾病诊断过程中,临床思维时应坚持 多元论 原则。答案:( X)1 7 2 6 、医学伦理与医学道德是相同的概念,两词可以通用。答 案 : ( V )1 7 2 7 、医学是没有阶段性的。答案:( Q1 7 2 8 、医学道德是永恒不变的。答案:( X)1 7 2 9 、性病病人有权要求医务人员为其保密。答案:( J)1 7 3 0 、我国医师法规定,医师进行试验性临床医疗,应经医院批

268、准,但不需征病人本人或家属的同意。答案:( X)1 7 3 1 、 在医学人体实验中, 对照实验使用安慰剂和进行双盲法试验, 不必征得病人的同意。答案:( J)1 7 3 2 、在特殊情况下,为了查清死者的病因,判断诊断治疗的谬误,有利于医学科学的发展, 虽未征得死者生前同意或家属的首肯, 经有关特定部门的批准, 也可以进行尸体解剖。答案:( J)1 7 3 3 、生育控制的方法主要包括避孕、人工流产和绝育。答 案 : ( V )1 7 3 4 、对确实患有严重遗传性疾病的人,可以强制实施绝育。答案:( X)1 7 3 5、确定病人死亡的医师不得同时是实施该死亡病人器官移植的手术者。答案:(

269、Q1 7 3 6 、在双方自愿的条件下,为实施器官移植挽救病人生命,可以进行器官的买卖。答案:( X)1 7 3 7 、干细胞研究目标,在于治疗严重的、难治的疾病,这种人类胚胎干细胞研究,应予支持。答 案 : ( V )1 7 3 8 、医师是医疗工作的主体。答案:( V )1 7 3 9 、冠心病病人发生左心衰竭,其心尖区闻及舒张期奔马律,以吸气末期最响。答案:( X)1 7 4 0 、第二心音的固定分裂见于房间隔缺损,负分裂( 逆分裂) 见于左束支传导阻滞。答案:( V)1 7 4 1 、心房颤动病人,心室率为4 2 次 / m i n , 0 R S 波形宽大畸形,但节律整齐,可以诊断为

270、新版医生三基题库第1 2 8 页心房颤动合并第三度房室传导阻滞。答案:( V)1 7 4 2 、某风心病二尖瓣狭窄合并心房颤动病人,病史3 年,心室率为7 0 次 / m i n , 其心房颤动治疗可以考虑奎尼丁或电复律,也可以地高辛口服治疗。答案:( X)1 7 4 3 、某病人服地高辛1 0 天,心电图示S T 段呈斜形向下偏移( 鱼钩形下移) ,提示洋地黄中毒,应立即停用该药。答案:( X)1 7 4 4 、风心病主动脉瓣关闭不全者最易并发感染性心内膜炎,而二尖瓣狭窄者则较少发生这种并发症。答案:( Q1 7 4 5 、近年发现血液中高密度脂蛋白( H D L ) 及其亚组分H D L

271、2 降低者, 冠心病的发生率升高,故这两种脂蛋白均为抗动脉粥样硬化性因子。答案:( J)1 7 4 6 、某预激综合征病人,突发阵发性室上性心动过速,可首选毛花昔C或普蔡洛尔静脉注射治疗。答案:( X)1 7 4 7 、交替脉与颈静脉充盈、肝颈静脉回流征的临床意义相同,三者均提示病人有右心功能不全。答案:( X)1 7 4 8 、心电图上未发现正常P波,O R S 波群呈室上性型,心室率1 1 4 次 / m i n , 节律不规则,据此可以诊断为快速性心房颤动。答案:( J)1 7 4 9 、高血压心脏病病人,心室率1 2 4 次 / m i n , 心电图示房性期前收缩,心尖区闻及舒张期奔

272、马律,双肺基底部细湿啰音,宜首选维拉帕米治疗。答案:( X )1 7 5 0 、应用多巴胺静脉滴注治疗心力衰竭,如剂量太大,有可能出现心室期前收缩。答案:( J)1 7 5 1 、血性胸腔积液可以排除结核性渗出性胸膜炎。答案:( X )1 7 5 2 、缺氧不一定有发维,发组不一定有缺氧。答案:( J)1 7 5 3 、急性呼吸窘迫综合征是一种非心源性肺水肿。答 案 : ( V )1 7 5 4 、胸腔渗出性积液都是感染性积液。答案:( X )1 7 5 5 、阻塞性睡眠呼吸暂停综合征,呼吸暂停时,呼吸动力亦消失。答案:( X )1 7 5 6 、溃疡病病人都有上腹痛,出现并发症后其疼痛的节律

273、性丧失。答案:( X )1 7 5 7 、慢性萎缩性胃炎病人均有慢性贫血的临床表现。新版医生三基题库第1 2 9 页答案:( X )1 7 5 8 、十二指肠球后溃疡是指溃疡发生于十二指肠球部的后壁。答案:( X )1 7 5 9 、普蔡洛尔是8受体阻滞药而甲氧氯普胺是a多巴胺受体的拮抗药。答案:( J)1 7 6 0 、阻塞性黄疸病人其尿胆原、尿胆素均为阴性。答案:( X )1 7 6 1 、通过胃粘膜活检尿素酶试验阳性,提示胃内有H p 感染的存在。答案:( V )1 7 6 2 、反流性食管炎病人禁用抗胆碱能药物如阿托品、普鲁苯辛等。答案:( Q1 7 6 3 、阵发性睡眠性血红蛋白尿的

274、确诊试验是抗人球蛋白试验。答案:( X )1 7 6 4 、凝血活酶生成能被正常硫酸钢吸附血浆纠正,不能被正常血清所纠正,则可确定为I X因子缺乏。答案:( X )1 7 6 5 、慢性粒细胞白血病周围血中性粒细胞碱性磷酸酶活性是增高的。答案:( X )1 7 6 6 、口服糖耐量试验异常加上尿糖阳性即可诊断为原发性糖尿病。答案:( X )1 7 6 7 、糖尿病病人尿酮体为阳性即可诊断为酮症酸中毒。答案:( X )1 7 6 8 、肥胖型2 型糖尿病初发者可首选二甲双肌; 类或睡哇烷二酮类口服降糖药。答案:( J)1 7 6 9 、S o mo y g i 效应是由于胰岛素用量过大所造成的。

275、答案:( J)1 7 7 0 、糖皮质激素抑制替代治疗方案中所使用的糖皮质激素剂量为生理剂量。答案:( X)1 7 7 1 、血清总/和总出现分离现象( 即其中一个正常、一个升高或降低) 只见于甲亢复发的早期。答案:( X)1 7 7 2 、碘剂既可用以治疗甲亢,又可引起甲亢。答案:( J)1 7 7 3 、口服避孕药对甲状腺摄I 率无影响。答案:( X)1 7 7 4 、某甲亢病人用抗甲状腺药物治疗已两年,目前无任何甲亢症状,血清总T s 、总Z正常,据此可以停用抗甲状腺药物而不会复发。答案:( X)1 7 7 5 、血红蛋白尿显微镜下可见大量红细胞。答案:( X)1 7 7 6 、尿三杯试

276、验,如三杯尿中均有血液则提示血尿来自肾脏、输尿管或膀胱内弥漫性出新版医生三基题库第1 3 0 页血。答案:(J )1777、肾病性水肿主要原因是由于肾小球滤过率下降。答案:(X)1778、白细胞管型对肾盂肾炎的诊断有重要价值。答案:( C1779、肾病综合征的临床特点是浮肿、血尿和高血压。答案:( X )1780、隐匿性肾炎一般无浮肿和高血压。答案:(J )1781、结缔组织的疾病,都是自身免疫性疾病。答案:(X )1782、ANA阳性,就可诊断为红斑性狼疮。答 案 :(X)1783、系统性红斑性狼疮有C%。 、C,下降,示SLE有活动性。答案:(J )1784、皮肤、肌肉病变是诊断皮肌炎不可

277、缺少的依据。答案:( V )1785、以泪腺、唾液腺病变为主的自身免疫性疾病是干燥综合征。答案:(J )1786、高渗性缺水在外科最常见,乂称急性缺水。答 案 :(X )1787、大量出汗而引起的缺水应属等渗性缺水。答案:(X )1788、低渗性缺水血钠低于130nlmol / L。答案:( X )1789、肝硬化腹水大量放液后为高渗性缺水。答案:(X )1790、人体主要依赖血液缓冲系统来调节酸碱平衡。答案:(X )1791、急性胰腺炎病人出现手指麻木应考虑低钙血症。答案:(J )1792、一般代谢性碱中毒主要依赖输注等渗盐水纠正。答案:答 )1793、代谢性酸中毒纠正后应注意补镁。答案:答

278、 )1794、成人每天生理需要钾34g和葡萄糖200250g。答案:(X )1795、胆胰屡病人易发生低渗性缺水与高钾血症。答案:(X )1796、低渗性缺水按临床表现轻重分为三度。新版医生三基题库第131页答案:( X)1 7 9 7 、低钾血症最主要诊断依据为病史和心电图典型改变。答案:( X)1 7 9 8 、休克时的病理生理变化主要是血压下降,尿少和酸中毒。答案:( X)1 7 9 9 、休克病人的最佳体位是头低足高。答案:( X)1 8 0 0 、甲亢行甲状腺大部切除术后并发手足抽搐主要是因甲状旁腺被误切所致。答案:( J)1 8 0 1 、按T N M 分期法, 第一期乳腺癌应为“

279、 T 2 , N0, M 。 。答案:( V )1 8 0 2 、直疝多见于妇女。答案:( X)1 8 0 3 、嵌顿性疝内容物仅为部分肠壁称为R i c ht r e 疝。答案:( J )1 8 0 4 、盲肠或膀胱下移组成疝囊壁的一部分称滑动性疝。答案:( V )1 8 0 5 、外伤性肝破裂临床症状可有呕血和便血的表现。答案:( V )1 8 0 6 、脾破裂时腹腔穿刺均可抽到血液。答案:( X)1 8 0 7 、腹部透视膈下无游离气体说明胃肠道无破裂。答案:( X)1 8 0 8 、胃、十二指肠溃疡并发出血均需外科治疗。答案:( X)1 8 0 9 、胃、 卜二指肠溃疡穿孔病人腹部透视

280、膈下均可见有游离气体。答案:( X)1 8 1 0 、胃、十二指肠溃疡穿孔病人,症状轻、腹膜炎局限者可行非手术治疗。答案:( V )1 8 1 1 、绞窄性肠梗阻,早期可出现休克。答案:( V )1 8 1 2 、腹痛同时伴有便血和腹部出现肿块是小儿肠套叠的典型临床征象。答案:( V )1 8 1 3 、各种类型的肠梗阻均需手术治疗。答案:( X)1 8 1 4 、结肠癌手术前应充分做好肠道准备。答案:( V )1 8 1 5 、急性化脓性阑尾炎可并发门静脉炎。答 案 :( V )1 8 1 6 、阑尾周围脓肿均需手术治疗。答案:( X)1 8 1 7 、直肠癌其下缘距肛门1 0 c m以上可

281、行D i x o n 手术。答案:( V )1 8 1 8 、有梗阻黄疸病史是行胆总管探查指征之一。答 案 :( V )1 8 1 9 、胰岛素瘤发生于胰腺B细胞。答案:( V )1 8 2 0 、坏死性胰腺炎血、尿淀粉前均可不增高。答案:( V )1 8 2 k 下肢静脉曲张的手术行高位结扎大隐静脉即可。答案:( 义)1 8 2 2 ,椎管内肿瘤包括脊髓本身及其邻近组织的原发或转移性肿瘤。答案:( V )1 8 2 3 、C T 诊断颅内肿瘤完全是依靠直接征象来判断的。答案:( X )1 8 2 4 、颅脑损伤后一侧瞳孔进行性散大,光反射迟钝或消失,伴对侧偏瘫与昏迷,是脑疝形成的临床征象。答

282、案:( V )1 8 2 5 、颅内压升高持续超过2 0 k Pa 时可诊断为颅内压增高。答案:( J )1 8 2 6 、脑震荡是指头部外伤后引起短暂的脑功能障碍而无确定的脑器质改变。答 案 :( V )1 8 2 7 、最易发生肋骨骨折的部位是第4 第 7 肋。答案:( V )1 8 2 8 、开放性胸部损伤是指胸腔有伤口。答案:( X )1 8 2 9 、开放性气胸急救处理的原则是,立即将开放性气胸变为闭合性气胸。答案:( J )1 8 3 0 、脓胸的脓液呈巧克力色,可以认定是结核分枝杆菌感染。答案:( X )1 8 3 1 、声嘶和胸腔积液均为早期肺癌的临床表现。答案:( X )1

283、8 32 、早期食管癌是指病变小于3c m , 且无转移。答案:( X )1 8 33、缺氧性晕厥常见于法洛四联症。答案:( V )1 8 34、急性尿潴留见于膀胱出口以下尿路严重梗阻病人。答案:( V )1 8 35 、隐睾症不容易发生恶变。答案:( X )1 8 36 , 非淋菌性尿道炎最常见的病原体是沙眼衣原体和支原体。答案:( V )新版医生三基题库第1 32 页1 8 37 、阴茎癌绝大部分发生在有包茎或包皮过长的病人。答案:( V )1 8 38 、膀胱的正常容量男性为5 0 0 m L , 女性为6 0 0 m L 。答案:( X )1 8 39 、 侧输尿管结石,对侧肾结石,应

284、先处理输尿管结石。答案:( V )1 8 40 、大于2 c m 的肾结石亦可行体外震波碎石。答案:( V )1 8 41 、后尿道损伤的早期手术可行高位耻骨上膀胱造痰,亦可早期施行尿道复位手术。答案:( V )1 8 42 、隐睾症易于发生恶变。答案:( V )1 8 43、直径大于5 m m 的输尿管结石即不可能经尿道排出。答案:( X )1 8 44、肱骨牌上骨折多见于老年人。答案:( X )1 8 45 、柯雷骨折多发生于儿童。答案:( V )1 8 46 、股骨颈骨折晚期最常见的并发症是股骨头缺血性坏死。答案:(Q1 8 47 、疑有腹腔脏器破裂的骨盆骨折病人最重要的检查是腹部C T

285、 检查。答案:( X )1 8 48 、骨盆骨折原则上应手术治疗。答案:( X )1 8 49 . 在脊柱骨折中,胸腰段脊柱骨折最常见。答案:( V )1 8 5 0 、脊髓震荡时,损伤平面以下立即发生弛缓性瘫痪。答案:( J )1 8 5 1 、颈椎病可分为神经根型、交感神经型、脊髓型、椎动脉型和食管型。答案:( V )1 8 5 2 、急性血源性骨髓炎最常见的致病菌是乙型链球菌。答案:( X )1 8 5 3、会阴指阴道与肛门间的软组织,包括皮肤、肌肉及筋膜。答案:( V )1 8 5 4、妊娠期阴道上皮增生,糖原积聚,阴道酸度增高,最适宜于阴道毛滴虫繁殖。答案:( X )1 8 5 5

286、、为了从母体获得足够氧气、营养物质和排泄代谢产物,胎儿有一条脐动脉和两条脐静脉。答案:( X )1 8 5 6 、产科四步触诊法检查时检查者应站在孕妇左侧。答案:( X )新版医生三基题库第1 3 3 页1857、雌激素刺激乳腺腺泡发育,孕激素刺激乳腺腺管发育。答案:( X)1 8 5 8 .自然流产最常见的原因为遗传因素。答 案 : (V)1859、妊娠晚期无痛性出血是胎盘早剥的特征。答案:( X)1860、羊水过多的常见胎儿异常为神经管畸形,而羊水过少者则为泌尿道畸形。答案:( V)1861、凡妊娠期有某种高危因素危害孕妇健康者称为高危妊娠。答案:(X)1862、分娩24小时后的10天内体

287、温一次达到或超过38称为产褥病率。答案:( X)1863、念珠菌阴道炎病人白带稀薄呈泡沫状。答案:( X)1864、阴道上皮在雌激素作用下增生角化、富含糖原,后者在阴道杆菌作用下变成乳酸以维持阴道呈酸性环境。答案:(V)1865、慢性盆腔炎中输卵管、卵巢囊肿应行手术治疗。答案:(V)1866、黄素化未破裂卵泡综合征、黄体功能不足、自身免疫反应及前列腺素分泌过多是内膜异位症引起不孕的常见原因。答 案 : (V)1867、无排卵型功能性出血的止血方法可用雌激素口服,每3天减量1次,每次减量不超过原用量的1 /3 。答案:(V)1868、 子宫脱垂的主要原因为卵巢功能衰退, 缺乏雌激素,导致肌肉筋膜

288、及韧带张力减退。答案:( X)1869、妇科病普查应每1 2年1次,以普查生殖道癌为重点。答案:(V)1870、人流综合反应常表现为心律失常、心动过速。答案:( X)1871、早吸吮是生后20分钟以内开始吸吮母亲乳房。答案:( X)1872、母乳营养丰富,可喂奶至2岁再停奶。答案:( X)1873、母婴同室母婴分离时,母亲应每天挤奶,24小时内应挤6 8次或更多。答案:(V)1874、婴儿睡觉后应待其醒后再哺乳。答案:( X)1875、母亲患有严重疾病如精神病、子痫时,婴儿可适当添加补充食品。答案:(V)新版医生三基题库第134页1 8 7 6 、小儿第1 年身长的增长最主要是躯干的增长。答案

289、:( X)1 8 7 7 、6 个月以内婴儿可不接种麻疹减毒活疫苗。答 案 : ( V )1 8 7 8 、对于低渗性脱水病人第1 个2 4小时内静脉输液成分宜用2 / 3 张含钠液。答案:( X)1 8 7 9 、对营养不良伴腹泻病人静脉补液宜按实际体重计算。答案:( V )1 8 8 0 、世界卫生组织规定,新生儿系指在新生儿期内的婴儿。答案:( V )1 8 8 1 、佝偻病的早期预防措施是及早肌内注射维生素D s 3 0 万U,每周1 次,共3 次。答案:( X)1 8 8 2 、第I I I 度营养不良儿的饮食调整应按标准体重计算热量。答案:( 义)1 8 8 3 、慢性腹泻病程大于

290、3 个月。答案:( X)1 8 8 4、房间隔缺损胸骨左缘第2 第 3 肋间出现收缩期杂音的机制是血流通过缺损处形成涡流所致。答案:( X)1 8 8 5 、治疗肾病综合征首选的糖皮质激素是泼尼松。答案:( J)1 8 8 6 、小儿慢性肾炎急性发作常与感染有关,其前驱期同急性肾炎,但有持续贫血、高血压和肾功能不全,可有血尿和明显蛋白尿,尿相对密度低且固定。答案:( X)1 8 8 7 、缺铁性贫血血红蛋白降低比红细胞降低明显;巨幼红细胞贫血则相反。答案:( V)1 8 8 8 、风湿舞蹈病系风湿活动的主要表现。答案:( J)1 8 8 9 、结核性脑膜炎脑脊液多核白细胞多仅见于早期病例。答案

291、:( X)1 8 9 0 、化脑并发硬脑膜下积液多见于肺炎球菌及流感嗜血杆菌脑膜炎的学龄前小儿。答案:( 义)1 8 9 1 、对接触了麻疹的易感儿应隔离观察21 天,做了被动免疫者无需观察隔离。答案:( X)1 8 9 2、联合免疫缺陷病患儿接受预防接种可增强抵抗力。答案:( X)1 8 9 3 、脊髓灰质炎病人近端大肌群常较远端小肌群瘫痪出现得早而重,功能恢复则从远端渐及近端肌群。答案:( J)1 8 9 4 、H B s Ag 具有抗原性,无感染性,它可诱导机体产生保护性抗体。答案:( V)新版医生三基题库第1 3 5 页1 8 9 5 、乙型病毒肝炎是全身性病毒感染,可出现肝外多脏器损

292、害。答案:( J)1 8 9 6 、慢性H B s Ag 携带者血清均具有传染性。答案:( V)1 8 9 7 、B 型超声检查是阿米巴肝脓肿的确诊方法。答案:( X)1 8 9 8 、钩体病的临床表现复杂,同型钩体可以引起完全不同的临床表现,而不同型的钩体又可引起极为相似的综合征。答案:( J)1 8 9 9 、皮肤淤斑是确诊流脑败血症的唯一条件。答案:( X)1 9 0 0 、肥达反应对伤寒有确诊价值。答案:( X)1 9 0 1 、检测特异性抗体I g G , 可用于某些传染病的早期诊断。答案:( 义)1 9 0 2 、霍乱病人在恢复期出现反应性发热,是由于循环改善后大量肠毒素被吸收所致

293、。答案:( V)1 9 0 3 、肝肺综合征是指慢性肝炎和肝硬化病人可出现气促、呼吸困难、肺水肿、间质性肺炎、胸腔积液和低氧血症等病理和功能改变。答案:( J)1 9 0 4 、原发性三叉神经痛为闪电样疼痛,每次发作时间仅数秒钟至2 分钊J 常伴有其他颅神经麻痹的症状和体征。答案:( X)1 9 0 5、多发性神经病是指主要表现为四肢对称性末梢型感觉障碍、下运动神经元瘫痪及自主神经障碍的临床综合征。答案:( V)1 9 0 6、脑血栓形成最常见的病因为脑动脉粥样硬化。答案:( J)1 9 0 7 、脑出血多在活动时发病,逐渐出现肢体无力等神经系统定位症状和体征,多在1 周后达高峰。答案:( X

294、)1 9 0 8 、蛛网膜下腔出血病人的主要体征为脑膜刺激征。答案:( J)1 9 0 9 、震颤性麻痹病人肌张力呈齿轮样增高。答案:( V)1 9 1 0 、多数性硬化是一种中枢神经系统脱髓鞘疾病。答案:( J)1 9 1 1 、偏头痛发作时口服非留体消炎镇痛药有效。答案:( J)1 9 1 2 、有无偏瘫是区别蛛网膜下腔出血和脑出血的重要体征。答案:( V)1 9 1 3 、病人本身没动却感到被人推动或自己动不起来了,此症状是感知综合障碍。答案:( X)1 9 1 4 、一位脑瘤病人,住院后常找不到自己的床位与厕所,也记不得当日进食的内容,此症状是逆行性遗忘。答案:( X)1 9 1 5、

295、路易体是位于细胞内的异常包涵体。答 案 : ( V)1 9 1 6、阿尔茨海默病的首发症状为近记忆力障碍。答案:( J)1 9 1 7 、病人某夜回家把房前的小树看成是有人在监视他,此症状是幻觉。答案:( X)1 9 1 8 、某病人常抱暖器管说是与工人阶段相结合,此症状称病理性象征性思维。答案:( J)1 9 1 9 、患者肢体经人摆布成极不舒服的姿势而长时间维持不变,此症状就是蜡样屈曲。答案:( J)1 9 2 0 、三环类抗抑郁剂的主要不良反应是锥体外系反应。答案:( X)1 9 2 1 、治疗剂量与中毒剂量很接近的药物是碳酸锂。答案:( J)1 9 2 2 、神经衰弱的病程特点是逐渐加

296、重的。答案:( X)1 9 2 3 、掌跖部的表皮不具有透明层。答案:( X)1 9 2 4 、急性湿疹的糜烂渗出期,外用药的剂型宜采用洗剂。答案:( X)1 9 2 5 、由青霉素引起的过敏性休克,首选措施是氢化可的松静脉滴注。答案:( X)1 9 2 6 、真菌般分为浅部真菌和深部真菌两大类。答案:( Q1 9 2 7 、牛皮癣( 银屑病) 和体癣、手足癣一样,也是一种癣,皮屑镜检真菌阳性。答案:( X)1 9 2 8 、淋病、尖锐湿疣、阴虱都属性传播疾病。答案:( J)1 9 2 9 、白塞综合征病人的皮肤针刺同形反应阳性。答案:( J)1 9 3 0 、寻常型天疱疮的水疱是在表皮下,棘

297、层松解征( 尼可斯基征) 阴性。答案:( X)1 9 3 1 、斑贴试验的目的是测验病人皮肤是否对某物质具有过敏性。答案:( J)1 9 3 2 、白瘢风和花斑癣的鉴别要点是前者可以查到真菌。答案:( X)新版医生三基题库第1 3 6 页1 9 33、由于视盘没有感光细胞,所以形成了生理盲点。答案:( V)1 9 34 、沙眼I I I 期表明沙眼严重,具有传染性。( 按我国分期法)答案:( X)1 9 35 、眶上裂主要有第n 、第ni 、第I V 、第 V、第V I 对脑神经通过。答案:( X)1 9 36 、睑板腺囊肿就是慢性睑板腺炎。答案:( X)1 9 37 、眼压计分为压陷式及压平

298、式两大类。答案:( V)1 9 38 、眼底检查时,每差3个屈光度约等于L 5 mm。答案:( X)1 9 39 、最常见的泪腺瘤是泪腺混合瘤。答案:( J)1 9 4 0 、树枝状角膜炎,可以使用皮质类固醇滴眼剂。答案:( X)1 9 4 1 、核性白内障病人视力低于0 . 1 时 即可行手术治疗。答案:( V)1 9 4 2、毛果芸香碱可以抑制房水产生,从而降低眼压。答案:( X)1 9 4 3、前房积脓性虹膜炎,口腔粘膜溃疡及外生殖器溃疡为毕夏病的三大主症。答案:( J)1 9 4 4 、原发性视网膜色素变性是一种具有遗传倾向的慢性、进行性视网膜色素上皮及光感受器变性的疾病。答 案 :(

299、 V)1 9 4 5 、眼底视盘边缘模糊就叫视盘水肿。答案:( X)1 9 4 6 、有一个4 5 岁病人需做眼底检查,用2 %后马托品扩瞳。答案:( X)1 9 4 7 、 眼内铜性异物常表现在角膜后弹力层棕黄色色素沉着, 有时晶体前囊呈葵花状混浊,称铜屑沉着症。答案:( J)1 9 4 8 、急性化脓性中耳炎常为急性上呼吸道感染或急性传染病的并发症。答 案 :( V)1 9 4 9 、单纯型慢性化脓性中耳炎一般均应考虑行乳突根治术,以清除病灶。答案:( X)1 9 5 0 、胆脂瘤型中耳炎的重要诊断依据是听力改变。答案:( X)1 9 5 k 迷路炎和面瘫属耳源性颅内并发症。答案:( X)

300、1 9 5 2 、由于鼻窦开口解剖位置彼此邻近,因此一窦感染常累及多窦。新版医生三基题库第1 3 7页答案:( Q1 9 5 3 、变应性鼻炎可分为常年性及季节性两型,季节性鼻炎又称花粉性鼻炎,且较常见。答案:( X)1 9 5 4 、鼻腔、鼻窦恶性肿瘤以鳞癌为主,肉瘤较少见。答案:( V)1 9 5 5 、成人咽后结核性脓疡,病变位于咽后间隙。答案:( X)1 9 5 6 、临床诊断为鼻咽纤维血管瘤的病人都应做病理活检,以求获得确切诊断。答案:( X)1 9 5 7、喉癌的病因可能与严重吸烟、饮酒、空气污染、病毒感染及癌前期病变有关。答案:( Q1 9 5 8、乳牙早失需做间隙保持器,其要求

301、只需保持缺隙的近远中距离适宜。答案:( X)1 9 5 9 、高分化粘液表皮样癌手术方法是腮腺全叶及肿瘤切除,应考虑做选择性颈淋巴清扫术。答案:( X)I 9 6 0 , 覆盖义齿的主要优点是保留了患牙的牙极,保持了牙周膜本体感受器。答案:( J)1 9 6 1 、颌骨中央性骨髓炎多由于急性冠周炎所致。答案:( x)1 9 6 2 、膜状突滑出关节窝以外,向前越过关节结节,称为撅下颌关节强直。答案:( X)1 9 6 3 、牙源性角化囊肿镜下所见表层的角化主要是不全角化,呈波状或皱纹状。答案:( J)1 9 6 4 、在拔除下颌阻生第3 磨牙时必须进行阻力分析。答案:( J)1 9 6 5 、

302、乳牙酷病治疗的目的是终止病变的发展,保持乳牙的正常替换。答案:( J)1 9 6 6 、牙周炎X 线照片见齿槽骨多呈垂直吸收,牙周创伤X 线片见齿槽骨多呈水平吸收。答案:( X)1 96 7 、植牙术包括牙再植术、牙移植术和牙种植术。答案:( J)1 96 8、被动运动是全靠外力帮助来完成的运动。答 案 : ( V)1 96 9、助力运动是以助力为主,主动运动为辅的运动。答案:( X)1 97 0 、被动运动时,运动要达到有疼痛才能起到治疗效果。答案:( X)1 97 1 、剪刀步态是小儿麻痹后遗症常见的步态。答案:( X)1 97 2 、跑步、爬山、划船、骑自行车、游泳都是一种无氧训练。新版

303、医生三基题库第1 3 8页答案:( 义)1973、康复医学和医疗康复在概念上有不同。答案:(J )1974、矫形器的使用目的只是为了预防畸形,而不能作为补偿功能活动。答案:(X )1975、短波、超短波电容场作用深度可达骨骼,而直流电、低中频电流主要在皮肤层,其次为肌层。答案:( V )1976、紫外线的脱敏作用主要通过皮肤固醇一向维生素D转化一血钙增高,组织胺释出,组织胺酶增加。而高频电流主要通过肾上腺皮质,皮质激素释出,使过敏性渗出性降低而起作用。答案:(J )1977、小剂量紫外线促进人体组织细胞生长繁殖,中剂量紫外线则杀伤细胞或致癌。答案:(X )1978、目前认为激光对生物体的作用主

304、要有热效应、电磁效应、光化效应、机械效用。答案:( V )1979、 上行直流电是将正极置于躯体头端( 如颈后) , 负极置于远端( 如腰般部) 的通电方法。答案:(X )1980、间动电流是将50Hz正弦电流经半波或全波整流后,经正弦脉冲仪以不同方式叠加在直流电上组成的中频电流。答案:(X )1981、高压氧下心率增快,心排血量增加。答案:(X )1982、高压氧治疗时; 采用间歇吸氧是为了防止减压病。答案:(X )1983、高压氧舱内禁用二氧化碳灭火器。答 案 :( V )1984、减压时,舱内病人身上的引流管都要关闭。答案:(X )1985、随着血氧张力的不断提高,血中的氧和血红蛋白量也

305、不断增加I。答案:(X )1986、国家标准规定,加压用的压缩空气中二氧化碳浓度应低于0.05%。答案:(J )1987、妊娠者发生中度以上一氧化碳中毒时,原则上应做高压氧治疗。答案:(J )1988、附加压等于表压。答案:(J )1989、吸收剂量概念适用于任何类型和任何能量的电离辐射,并适用于受到照射的任何物质。答案:( V )1990、直线加速器能产生高能电子束、高能X线和丫射线。新版医生三基题库第139页答案:( 义)1991、靶区勾画时,GTV勾画不应包括转移淋巴结。答案:(X)1992、姑息性放疗的目的是完全治愈肿瘤,达到较高的5年生存率。答案:(X )1993、处于临床死亡期的病

306、人是可以被复苏的。答 案 :( V )1994、心搏骤停是指心脏突然衰竭而不能搏出足量的血液以保证脑的存活。答案:( V )1995、心跳停止58分钟内,称临床死亡期。答案:( Q1996、 在常温、 常压下人脑所能耐受完全性缺血缺氧而不出现死亡或不可逆脑损害的时间,最长不超过45分钟。答案:(J )1997、心源性休克主要系心室功能严重减退,心排血量急剧下降和周围循环衰竭。答案:( V )1998、肾性急性肾衰竭主要是由于各种原因引起血容量不足和循环衰竭所致的肾血流灌注量不足而导致的肾功能损害。答案:( 义)1999、每天尿量少于400mL者称为无尿。答案:(X )2000、急性肾小管坏死少

307、尿期尿液检查尿相对密度高而固定,多在L 030以上。答案:(X )2001、血液透析和腹膜透析是目前治疗急性肾衰竭高钾血症的最有效疗法。答 案 :( V )2002、屈氏韧带以上的消化道出血,称为上消化道出血。答 案 :( V )2003、多器官衰竭是指在严重创伤、大手术、休克及感染后,机体在短期内出现两个或两个以上的器官同时或先后发生功能不全以致衰竭的临床综合病征。答案:(J )2004、饮入汽油、煤油等有机溶剂时,可先服用液状石蜡150200mL,使其溶解而不被吸收,然后进行洗胃。答案:(J )2005、腹膜透析疗法对于具有脂溶性的毒物( 如有机磷杀虫药等) 中毒透析效果很好。答案:(X)

308、2006、一般在中毒后12小时内进行人工透析效果较好。答案:( V )2007、催眠药及安定药中毒病人大多具有昏迷、肌肉松弛性麻痹、瞳孔反射存在、呼吸抑制、体温降低等临床表现。答案:( V )2008、对各个衰竭器官的支持疗法是防治多器官衰竭最重要的手段。新版医生三基题库第140页答案:(J )2009、猝死型冠心病经过及时有效的心脏复苏,是可能挽救病人生命的。答案:( V )2010、全血胆碱酯酎测定是诊断有机磷杀虫药中毒和判断其中毒程度的重要指标。答案:( V )2011、五步蛇( 尖吻蝮) 系神经毒类毒蛇。答案:(X )2012、水莽草中毒晚期出现类似破伤风样的痉挛,最后因呼吸麻痹、窒息

309、、甚至死亡。答案:(J )2013、葡萄球菌及肉毒杆菌食物中毒仅由细菌毒素致病,病人无传染性。答案:32014、淡水淹溺所致低氧血症主要机制是肺泡改变了表面张力而萎陷。答案:( V )2015、在抢救电击伤所致心搏骤停时,肾上腺素应列为禁忌。答案:(X )2016、 般说来,低压电触电常引起心室纤颤。答案:( V )2017、AB0血型不合新生儿溶血病常发生于第1胎,Rh血型不合则发生于第1胎以后。答案:(J )2018、采血可能诱发疾病发作或加重的病人不宜进行储存式自身输血。答案:(J )2019、溶血性输血反应可出现畏寒、发热、腰痛、血尿、黄疸等临床表现。答案:(J )2020、阴阳是对相

310、关事物或现象相对属性或同一事物内部对立双方属性的概括。答案:(J )2021、藏象学说的藏“ 是藏于人体内的气血, 象” 是指人体的形象。答案:(X )2022、奇恒之腑为脑、髓、骨、骼、脉、女子胞的合称。答案:(X )2023、经络系统是由动脉、静脉组成的。答案:(X )2024、正常的六气不使人致病。答案:(J )2025、面部白色乃脾虚湿蕴之象。因脾失健运,水湿内停,气血不充。答案:(X)2026、黑色为阴寒水盛之色,主肾虚、水饮和瘀血。答案:(J )2027、浮垢苔、 脓腐苔 、 霉腐苔” 多见于食积痰浊、内痈、湿热口糜。答案:(J )2028、寸口脉分为寸、关、尺三部,每部又分为浮、

311、中、沉三候。答案:( V )2 0 2 9 、温病T l分证的主要症状是脉浮、头项强痛而恶寒。答案:( X)2 0 3 0 、寒证与热证的鉴别主要从寒热、口渴、面色、四肢、二便、舌脉六个方面着手。答案:( V)2 0 3 1 、胃为水谷之海,主纳谷,以升为顺。答案:( X)2 0 3 2 、肺乃娇脏, 其性喜燥恶湿, 喜刚恶柔。答案:( X)2 0 3 3 、肾藏精的主要生理作用是摄纳肺所吸入的清气。答案:( 义)2 0 3 4 、厥证的主要病机是由于久病气血虚衰所致。答案:( X)2 0 3 5 、凡是中风都可使用活血化瘀药物治疗。答案:( X)2 0 3 6 、支饮是喘证的一个证候。答案:

312、( X)2 0 3 7 、呼吸喘促,畏寒发热,身体疼痛,咽痛,眼睑或全身浮肿,小便不利,此是喘证。答案:( X)2 0 3 8 、1 周内服用过阿司匹林,对计数血小板有影响。答案:( J)2 0 3 9 、服用维牛.素C 后,用试带法检验尿葡萄糖,可出现假阳性。答案:( X)2 0 4 0 、应在病人发热期从血中找回归热螺旋体。答案:( J)2 0 4 1 、血中H B s Ag阳性即可诊断被检者为乙型病毒性肝炎。答案:( X)2 0 4 2 、6 0 岁以上高龄老人,常见E S R 增快。答案:( C2 0 4 3 、作血清冷凝集素试验的血标本不能冷藏。答案:( J)2 0 4 4 、甲胎蛋

313、白对原发性肝癌有特异性的诊断价值。答案:( X)2 0 4 5 、尿糖定量测定可用甲醛防腐。答案:( X)2 0 4 6 、血红蛋白报告惯用单位为g / d L , 改为法定单位制应为g / L 。答案:( V)2 0 4 7 、伤寒病人血中未培养出伤寒沙门菌即不能确诊伤寒。答案:( X)2 0 4 8 、使用酚酸对P S P 试验无影响。答案:( x)2 0 4 9 、自然对比缺乏的部位,人为地将对比剂引入器官内或其周围,造成人工对比影像,新版医生三基题库 第1 4 1 页称为造影检查。答案:(J )2050、碘过敏试验阴性者,在造影检查过程中仍有出现严重反应的可能。答案:( V )2051

314、、根据CT值测量可以分辨体层层面不同软组织结构及病变。答案:(X)2052、螺旋CT容积扫描是三维立体图像重建、CT血管成像、CT仿真内镜等技术的基础。答案:( V )2053、由于X线通过左侧或者右侧胸腔的行程相同,所以左、右侧位胸片上的影像都一样。答案:( 义)2054、 表示组织密度的CT值单位为H u ,软组织的CT值为2050Hu,脂肪为-70-90Hu之间。答案:(J )2055、观察右侧颈椎椎间孔,宜摄取左后斜位颈椎照片。答案:(J )2056、关节结核好发于四肢小关节,双侧对称性受累。答案:(X)2057、CT扫描上的密度高低与MRI成像的信号强弱,木质是一样的。答案:(X )

315、2058、腹部脏器疾病以CT扫描为首选检查方法,胃肠道疾病则以钢剂造影为主。答案:(J )2059、未成熟儿颅内侧脑室前角出现回声是颅内出血的表现。答案:(J )2060、超声探测肝脏时、 常利用显示出的肝中静脉而将肝脏分为右肝前叶和后叶。答案:(X )2061、二维B超发现心脏左室明显扩大,其他房室腔亦大,室间隔及左室后壁普遍性运动减弱,二尖瓣开放幅度小等,常提示为扩张型心肌病。答案:( V )2062、氨基甘类抗生素与尿道碱化剂配伍,能增强尿路感染的抗菌效能。答案:32063、依托红霉素和红霉素相比,前者口服吸收好,血浓度高,维持时间长,而且长期服用不良反应也比红霉素为少。答案:(X )2

316、064、四环素类引起的二重感染是由于抑制了敏感的细菌,破坏了细菌间的共生平衡状态所致。答案:( V )2065、氯丙嗪及苯巴比妥均可阻断网状结构上行激活系统,但由于两者作用部位不同,故氯丙嗪引起的睡眠状态易被唤醒。答案:(J )2066、异丙肾上腺素可用于治疗哮喘,普蔡洛尔则禁用于哮喘病人。答案:(J )2067、应用阿托品可见心率加快,这是由于阿托品直接兴奋心脏的结果。新版医生三基题库第142页答案:( X)20 68、医师开处方时,药品名称可以使用中文、英文和代码。答案:( X)20 69 、西药和中成药可以分别开具处方,也可以开具在同一张处方上。答案:( X)20 70 、特殊情况可以超

317、剂量使用药品,但需要医师再次签字。答 案 : ( V)20 71 、医疗机构的执业医师有权开具麻醉药品和第一类精神药品。答案:( X)20 72、进修医师可以由接受进修的医疗机构认定授予相应的处方权。答案:320 73、为门( 急) 诊病人开具的麻醉药品注射剂,每张处方为一天常用剂量。答案:( X)20 74 、三级医院必须建立符合标准的临床微生物实验室,配备相应设备及专业技术人员,开展病原微生物培养、分离、鉴定及细菌药敏试验工作。答案:( J)20 75、清洁手术在手术野无污染的情况下通常不需预防用抗菌药物。答案:( J)20 76、危重病人在未获知病原菌及药敏结果前,可根据病人的发病情况、

318、发病场所、原发病灶、基础疾病等推断最可能的病原菌,并结合当地细菌耐药状况先给予抗菌药物经验治疗。答案:( J)20 77、 C a n c er ” 一词是所有恶性肿瘤的统称。答案:( J)20 78、原发性肝癌仅指肝细胞发生的恶性肿瘤。答案:( X)20 79 、宫颈原位癌累及腺体仍属于原位癌。答案:( C20 80 、良性胶质瘤有包膜,恶性胶质瘤呈浸润性生长。答案:( X)20 81 、恶性淋巴瘤与淋巴细胞白血病存在重叠,组成一个连续的谱系。答案:( J )2 0 8 2 、成年人肺结核主要通过支气管播散。答 案 : ( V)2 0 8 3 、放射性核素显像,不是单纯形态结构的显像,而是一

319、种独特的功能性显像。答案:( V)2 0 8 4 、放射性核素显像诊断的优越性在于图像清晰、显示组织结构比X - C T 和磁共振的图像要好。答案:( X)2 0 8 5 、放射性核素显像和放射免疫分析检测,是核医学的两项重要内容,它们都是利用核射线在体处进行检杳的诊断方法。新版医生三基题库第1 4 3 页答案:(X )2086、心血池动态、心肌灌注平面和断层显像对心肌缺血、 心肌梗死均有较好的临床价值。答案:( V )2087、凡进行核医学检杳的病人,无须做任何准备,这是核医学检测的最大优点。答案:(X )2088、动物蛋白质中肉类产氨最多,蛋类次之,奶类最少。答 案 :( V )2089、

320、富含植物纤维的藻类和豆类食品食后吸收快,血糖升高也快。答案:(X )2090、人只能吸收单糖,双糖以上的糖首先要在小肠消化成单糖才能吸收。答案:32091、低喋吟饮食禁用的食物有肝、肾、沙丁鱼、牛奶、肉汁等喋吟含量高的食物。答案:(X )2092、多不饱和脂肪酸是指含有两个以上不饱和键的脂肪酸。答案:(J )2093、预后营养指数是评价外科病人术前营养状况及预测术后并发症发生危险性的综合指标。答案:( V )2094、肝性脑病食物蛋白质选择以大豆蛋白为主,严重时暂禁动物蛋白。答案:(J )2095、肠外营养液配制时可将磷酸盐、安达美和电解质都加入氨基酸内。答案:(X )2096、妊娠糖尿病总热

321、能按理想体重的3035kcal / ( kg d)计算。答案:(X )2097、赖氨酸是谷类食物蛋白质中的第一限制氨基酸。答案:(J )2098、对于有明确潜伏期的感染,病人自住院第1天算起,超过其平均潜伏期而发病者属于医院感染。答案:32099、发生在入院12小时后的败血症或化脓性脑膜炎属医院感染。答案:(X )2100、出院1个月以内的手术切口感染属医院感染。答案:32101、皮肤粘膜开放性伤口,无明显红肿痛热,但从分泌物中培养出金黄色葡萄球菌,应列为医院切口感染。答案:(X )2102、病人咳嗽吐痰,痰培养阳性,应诊断为医院肺部感染。答案:(X )2103、不论有无症状的菌尿症,发生在出

322、院第6天,属医院尿路感染。答案:( V )2104、厌氧菌是消化道内最多的细菌,对机体有利,医疗过程中应注意保护。新版医生三基题库第144页答案:(J )2105、住院中发生阑尾炎穿孔性腹膜炎和肝脓肿穿破性肺脓肿属于医院感染。答案:(X )2106、伤口异物所致的局部红肿痛热不属医院感染,但在此基础上继发的细菌感染属医院感染。答案:(J )2107、不论成人或儿童的肺炎,均无需隔离。答案:(X )2108、医院感染乂称医院内获得性感染或交叉感染。答案:( 义)2109、消毒是指杀灭物品上所有的微生物,包括细菌芽抱和真菌抱子。答案:(X )2110、在医院中出生的新生儿感染,均属医院感染。答案:

323、(X )2111、重复使用的医疗器械,用完后应立即送中心供应室灭菌处理。答案:(X)2112、纤维支气管镜检查能直视气管及各级支气管。答案:(X )2113、纤维胃镜目视诊断为胃癌而活检未发现癌细胞可排除胃癌。答案:(X )2114、 萎缩性胃炎活检发现中度不典型增生时, 应于36个月后追踪观察以发现早期胃癌。答案:(J )2115、阴道镜上的接目镜能将宫颈放大540倍,以利观察宫颈病变。答案:(J )2116、阴道镜检是宫颈癌辅助诊断的重要方法。答案:( V )2117、通过膀胱镜可测定分侧肾功能及向肾盂内灌注药物。答案:( V )2118、电凝、电切结肠息肉前,可采用20%甘露醇口服清洁肠

324、道。答案:(X )2119、结肠纤维镜是目前发现大肠癌的最可靠的诊断方法。答案:(J )2120、用肺活量计可以测定小呼吸道的通气功能。答案:(X )2121、脑电图不仅可帮助癫痫的诊断,也可帮助脑瘤的病因诊断。答案:(X )2122、12岁儿童枕部脑电频率不应低于6次/ s。答案:(J )2123、肌电位纤颤电位和正峰波,只有在失神经支配的肌肉才能发现。答案:(X )2124、足的内翻和外翻运动主要产生于距跟关节和距跟舟关节。新版医生三基题库第145页答 案 :2 1 2 5、答案:2 1 2 6、答案:( J )心包裸区是心包前面无胸膜遮盖的部分。( V )门静脉收集腹盆腔内所有不成对脏器

325、的静脉血o( X )第一篇医学影像诊断第一部分三基知识总论单项选择题1 . 透视检查的基础是利用X 线的是:( E )A . 光学作用 B . 生物作用 C . 穿透作用D . 电离作用 E . 荧光作用2 . 孕妇需避免X 线检查,是因为:A. X 线的光化学效应 B. X 线的荧光作用 C. X 线的生物效应D. X 线的感光作用 E . 以上都不是(C)3. CR与普通X 线成像比较其优点,叙述错误的是:A . 提高了图像密度分辨率 B . 提高了图像显示能力 C . 降低X 线曝光量D . 提高了图像空间分辨率 E . 曝光宽容度增加(D)4 . 关于DR成像优点的叙述,错误的是:A

326、. 成像慢 B . 噪声小 C . 图像质量好D . 可转入PACS中 E. X 线信息损失小(A)5 . 根据CT工作原理,X 线穿过人体后首先接收到的部分是:A . 计算机 B . 阵列处理机 C . 探测器D . 磁盘 E . 照相机(C)6 . 螺旋CT扫描与传统CT扫描相比最重要的优势是:A . 扫描速度快 B . 二维或三维成像效果好 C . 重建速度快D . 容积扫描 E . 单层或多层连续扫描(A)7 . 关于碘过敏反应的描述,错误的是: (A . 检查室应备足抢救设备和药品 B . 造影前用抗过敏药可减少碘过敏反应C . 碘过敏试验阴性则不会发生碘过敏反应D . 多见于血管内

327、给予含碘造影剂E . 用同一批号含碘造影剂也要做碘过敏试验C)8 . 下列MRI检查的禁忌证,除了:A . 有心脏起搏器 B . 幽闭恐惧症患者 C . 有心肺监护仪者D . 有碘过敏史 E . 体内有铁磁金属植入物(D)9 . 下列关于窗位的概念,正确的是:A . 窗位相当于显示灰阶的中心 B . 窗位规定所显示CT值的范围C . 不同机器的窗位值不同 D . 窗位与所显示的组织CT值无关E . 通常窗位选择以水的CT值为标准(A)1 0 .下列关于CT位的概念,正确的是:(E )A . 根据CT值可以对病变做出定性诊断 B. CT值反映了物质内的成分C. CT值是物质密度的绝对值 D .

328、不同机器产生的CT值不同E. CT值反映了物质的密度1 1 .人体MRI最常用的成像原子核是:A . 氢原子核 B . 钠原子核 C . 钙原子核D . 磷原子核 E . 氮原子核(A )1 2 .关于窗宽,下列说法不正确的是:A . 窗宽规定了显示CT值的范围B . 调节窗宽的大小可改变图像中组织的密度对比C . 组织CT值超过窗宽规定的范围时为全白密度D .缩小窗宽使图像中组织的密度对比缩小E . 通常窗宽除以16等于每个灰阶包含的CT值(D )1 3 .以下临床表现中,不是血管内应用碘对比剂的不良反应:A . 尊麻疹 B . 而部潮红 C . 腹泻D . 球结膜水肿 E . 呕吐(C )

329、1 4 .根据CT值的定义公式。空气的CT值为:A. -700 HU B. -800 HU C. -900 HUD. -1000 HU E. -1100 HU(D )1 5 .当窗宽为2 5 0 ,窗位为50时,其 CT值显示范围为:A. 50 250 HU B. -7 5 -1 7 5 HU C. 125 125 HUD. 0-250 HU E. 50 300 HU(B )16. CT基本设备不包括:A . 扫描架、扫描床和X 线发生系统 B . 计算机和数据采集、阵列处理系统C . 操作台 D . 图像显示系统 E . 照相机(E )17. CT值的定义是以_ _ _ _ _ _ _ _

330、的衰减系数为标准来计算各种组织的CT值:A . 空气 B . 骨 C . 水D .脑组织 E . 以上都不是(C )1 8 .减轻或消除C T图像部分容积效应,可采用:A . 提高扫描矩阵 B . 缩小扫描层厚 C . 骨算法重建D .提高扫描X 线条件 E . 加大扫描层距(B )1 9 .在日常工作中,通过调节窗宽窗位来观察图像,要提高观察范围内组织的对比分辨率,取:( A )A . 适当减小窗宽 B . 适当增大窗宽 C . 适当提高窗位D . 适当降低窗位 E . 以上都不是首先应采2 0 .下 列 “ 碘过敏反应”中,最危险的是:A . 面部潮红 B . 喉部发痒 C . 皮肤瘙痒D

331、 . 打喷嚏 E . 皮肤丘疹(B )第二部分神经系统影像诊断单项选择题1 . 颈内动脉虹吸段一般分为:( D )A. 2 段D. 5 段B. 3 段E. 6 段C. 4 段2 . 颅内较常见的生理钙化,除了:( E )A . 大脑镰钙化B . 床突间韧带钙化C . 松果体钙化D . 脉络膜从钙化E . 垂体钙化3 . 蝶鞍侧位片上可测量其前后径及深径,其平均值分别为:( B )A. 10. 5 mm, 9. 5 mmB. 11. 7 mm, 9. 5 mmC. 12. 0 mm, 10. 0 mmD. 5. 0 mm, 10. 0 mmE. 10. 0 mm, 20. 0 mm4 . 确诊早

332、期听神经瘤的理想方法是:( E )A. X 线平片B . 桥小脑池空气造影C . 脑池造影CT检查D .超声检查E. MRI检查5 . 头颅CT所示与肿瘤无关的是:( E )A . 中线结构移位B . 脑水肿C . 颅骨吸收或破坏D .脑积水E . 脑沟增宽6 . 脑脓肿壁形成早期,CT表现为:( D )A . 水肿最明显B . 结节样增强C . 不均匀环形增强D . 均匀环形增强E . 无水肿7 . 神经胶质瘤又称胶质瘤,起源于神经胶质细胞,不包括:( B )A . 星形细胞瘤B . 脑膜瘤C . 少突胶质细胞瘤D . 室管膜瘤E . 髓母细胞瘤8 . 新鲜出血的CT值为:( E )A. 2

333、0-40 HUB. 30 50 HUC. 40 60 HUD. 50-70 HUE. 60 80 HU9 . 下列选项中,脑膜瘤的典型CT表现是:( E )A . 钙化、囊变坏死B . 骨质破坏C . 明显瘤周水肿D . 占位效应明显E . 多为等密度,明显均匀增强1 0 .髓母细胞瘤好发于:( A )A . 儿童B . 青壮年C . 中年D . 中老年E . 老年1 1 .女,6 岁,头痛,C T示小脑蚓部有- 3 cmX3. 5 cm 高密度影,明显增强,四脑室受压变窄,前移,幕上脑积水。最可能的诊断为:( A )A . 髓母细胞瘤B . 脑膜瘤C . 脉络丛乳头状瘤D .脑结核E . 血

334、管网状细胞瘤1 2 .胶质瘤的CT表现为:( E )1 9 .女,30岁,腰痛2 年。CT示腰4 右侧锥旁有一 2 cmX3 cm 软组织密度肿块,呈哑铃形,强化明A . 可为不规则低密度 B . 可呈囊状低密度C.可强化D . 可不强化 E . 以上都是1 3 .急性硬膜外出血典型的CT表现是:( B )A . 颅骨内板下方双凸形低密度区 B . 颅骨内板下方双凸形高密度区C . 颅骨内板下方新月形高密度区 D . 颅骨内板下方新月形低密度区E . 颅骨内板下方双凹形高密度区1 4 .急性硬膜下出血典型的C T表现是:( C )A .卢页骨内板下方双凸形低密度区 B . 颅骨内板下方双凸形高

335、密度区C . 颅骨内板下方新月形高密度区 D . 颅骨内板下方新月形低密度区E . 颅骨内板下方双凹形低密度区15. 下列选项中,不是化脓性脑脓肿CT表现的是:( D )A . 可有占位效应 B . 强化环完整、均匀、 光滑C . 可见气液平 D . 强化环厚薄不均E.周围可有低密度水肿1 6 .头颅CT上高密度病灶可以是:( E )A . 室管膜瘤 B . 出血C.钙化D .脑膜瘤 E . 以上都是1 7 .中线结构同侧移位提示同侧:( A )A . 脑萎缩 B . 胶质瘤C.脑脓肿D . 脑膜瘤 E . 硬膜下血肿1 8 .急性脑中风影像诊断宜选择:( D )A .颅骨平片 B . 脑室造

336、影C.血管造影D. CT扫描 E . 超声成像显,腰 4、5 右侧椎间孔扩大。最可能诊断为:( A )A . 神经源性肿瘤 B . 转移瘤 C.D . 淋巴瘤 E . 椎间盘突出巨细胞瘤2 0 .最容易损伤脊髓的外伤是: (A .横突孔骨折伴附件骨折 B . 椎体压缩性骨折伴横突骨折C . 胸椎骨折伴肋骨骨折 D . 下腰椎骨折,骨折碎片进入椎管E . 胸椎附件骨折,骨折碎片进入椎管E )21. 最常见的椎管内神经源性肿瘤是:A . 神经鞘瘤 B . 神经纤维瘤 C.D . 神经节细胞瘤 E . 神经节母细胞瘤(神经纤维肉瘤A )2 2 .急性颅脑损伤影像宜选择:A .颅骨平片 B . 脑室造

337、影 C.D. CT扫描 E. MRI成像(血管造影D )2 3 .当疑有脑转移瘤时,优先的影像检查方法是:A . 头颅平片 B . 脑血管造影 C.D . 脑室造影 E . 核素扫描(CTC )2 4 .髓鞘形成异常病不包括: ( B )良A .髓鞘形成不足 B . 脱髓鞘病变 C . 肾上腺脑白质营养不D . 海绵状变性 E . 异染性脑白质营养不良2 5 .脑挫裂伤CT表现为: (A . 局部呈低密度改变 B . 散在点片状出血 C . 占位表现D . 可合并蛛网膜下隙出血 E . 以上都对E )2 6 .关于颅咽管瘤的描述,不正确的是: (A . 常见于儿童 B . 多为囊性 C . 对

338、有钙化D . 均发生于鞍上 E . 可出现脑积水D )27. 下列关于脑膜瘤的描述,不正确的是: (A . 女性多见 B . 与硬脑膜粘连紧 C . 多有包膜D . 均为实性肿块 E . 多呈较高密度D )28. 脑 内多发性硬化最常见的部位是: (A .侧脑室周围白质 B . 视交叉及视神经 C . 基底节D . 脑干 E . 小脑白质A )2 9 .有关腔隙性脑梗死错误的描述是: (A . 好发于大脑深部、基底节区 B . 皮层支动脉梗死C . 大小约0.5 1.5 cm D . 常多发 E . 可无临床症状B )3 0 .颅脑先天性畸形神经管闭合障碍见于以下各病,错误的是: (A . 脑

339、膨出 B. Chiari 11畸形 C . 脑裂畸形D .颅裂畸形 E . 脱月氐体发育不全C )3 1 .男,36岁,头痛,癫痫,C T 示脑实质多数高密度点状影和0.5 1 cm 圆形低密度灶,呈环形增强。最可能的诊断是: (D )A . 多发性硬化 B . 脑脓肿 C . 结节性硬化D . 脑囊虫病 E . 病毒性脑炎3 2 .女,25岁,癫痫,CT示侧脑室旁多发低密度灶,结节样或块状增强。最大可能是:(A . 转移瘤 B . 脑囊虫 C . 脑脓肿D . 多发性硬化 E . 髓鞘形成异常D )3 3 .有关颅内压增高平片表现的描述,错误是: (A . 颅缝增宽 B . 脑沟增宽 C .

340、 蝶鞍增大D .脑回压迹增多 E . 颅壁变薄B )3 4 .脑转移瘤好发于 (A . 大脑皮层 B . 小脑皮层 C . 大、小脑皮层D . 脑髓质 E . 皮髓质交界区E )3 5 .头颅平片确定颅内压增高的主要依据是: (A . 颅骨骨质吸收,脑回压迹增多、增 深 B . 蝶鞍改变,颅骨骨质吸收D )C . 颅缝增宽,脑回压迹增多、增深 D . 颅缝增宽,蝶鞍改变,脑回压迹增多、E . 颅缝增宽,颅骨骨质吸收增深36. MRI上、短 T、 短 T?信号主要见于: ( D )A . 胶质瘤 B . 脑膜瘤D . 黑色素瘤 E:听神经瘤C . 转移瘤37. 脑脓肿形成期增强扫描脓肿壁明显强化

341、具有以下特点,除了 : ( D )A . 完整 B . 光滑C . 壁薄D .壁厚 E . 均匀3 8 .头颅CT上低密度病灶常见于以下病变,错误的是:( C )A . 组织坏死 B . 脑水肿C . 急性出血D . 软化灶 E . 脂肪瘤3 9 .常见的占位征象有以下病变:( E )A . 中线结构移位 B . 脑室移位C . 脑室变形D .脑沟变化 E . 以上都是4 0 .星形细胞瘤与脑梗死的鉴别最有意义的是:( E )A . 低密度 B . 不增强C . 单脑叶分布D . 多脑叶分布 E . 不按血管支配区分布4 1 .确认小脑扁桃体延髓联合畸形,一般认为扁桃体低于枕大孔: ( D )

342、A. 10 mm B. 7. 5 mmC. 6. 0 mmD. 5. 0 mm E. 4. 0 mm42. 脑血管畸形最常见的是:( C )A . 静脉畸形 B . 烟雾病C . 动静脉畸形D . 海绵状血管瘤 E . 毛细血管扩张症4 3 .颅咽管瘤和髓母细胞瘤都好发于:( A )A . 儿童 B . 青壮年C . 中年D . 中老年 E . 老年4 4 .脑转移瘤的常见CT表现不包括:( E )A . 多发 B . 瘤周水肿显著C . 出血、坏死D . 不同形式的强化并存 E . 钙化45. CT脊髓造影示脊髓增粗,蛛网膜下隙变窄,定位诊断为:( A )A . 髓内 B . 髓外硬膜外C

343、. 髓外硬膜内D . 髓外硬膜内外 E . 髓内和硬膜内4 6 .增强CT扫描,下列病变不增强的是:( D )A . 垂体腺瘤 B . 动脉瘤C . 脑膜瘤D . 蛛网膜囊肿 E . 髓质细胞瘤4 7 .由于血脑屏障破坏所致的肿瘤增强是:( D )A . 脑膜瘤 B . 垂体瘤C . 听神经瘤D .胶质瘤 E . 三叉神经瘤48. 患者男,26岁。头颅外伤4 h。CT轴位平扫示右题骨骨折,头皮血肿,右颜部梭形不均匀高密度,中线结构左移。最可能的诊断为:( A )A . 右颗部急性硬膜外血肿 B . 右颗部急性硬膜下血肿C . 右颗叶急性脑血肿 D . 右颗叶脑挫裂伤伴硬膜下血肿E . 右题叶亚

344、急性硬膜外血肿4 9 .儿童最常见的脊髓肿瘤是: ( C )A . 髓母细胞瘤B . 少突胶质细胞瘤 C . 星形细胞瘤D . 生殖细胞瘤E . 畸胎瘤5 0 .脊髓损伤包括以下各项,错误的是: (E )A . 脊髓震荡B . 脊髓挫裂伤 C . 脊髓压迫D . 脊髓横断E . 脊髓空洞第三部分头颈部影像诊断单项选择题1 . 内耳道狭窄的诊断依据是内耳道小于:( B )A. 1 mm B. 3 mmD. 7 mm E. 9 mm2 . 鼻咽癌常规X 线检查位置为:C. 5 mm( E )A . 鼻咽侧位 B . 颅底位D . 汤氏位 E . 鼻咽侧位+颁顶位3 . 颅底前后分3 个颅凹,后颅凹

345、有:C . 颈侧位( D )A . 破裂孔 B . 卵圆孔D . 枕大孔 E . 视神经孔4 . 成人最常见的眼眶内原发性肿瘤是:C . 棘孔( B )A . 泪腺多形性腺瘤 B . 海绵状血管瘤D . 脑膜瘤 E . 神经鞘膜瘤5 . 副鼻窦中恶性肿瘤最多发生于:C . 视神经胶质瘤( D )A . 额窦 B . 前组筛窦D . 上颌窦 E . 蝶窦6 . 最常见的喉癌类型为:C . 后组筛窦( B )A . 黑色素瘤 B . 鳞癌D . 未分化癌 E . 淋巴瘤7 . 关于鼻窦开口的叙述正确的是:C . 腺癌( A )A . 额窦和上颌窦开口于中鼻道 B . 额窦开口于下鼻道C . 筛窦和

346、蝶窦开口于中鼻道 D . 额窦和蝶窦开口于中鼻道E . 额窦开口于上鼻道8 . 继发性胆脂瘤多发生于:( C )A . 鼓窦B . 外耳道C . 上鼓室D . 中鼓室E . 下鼓室9 . 鼻咽常见良性肿瘤是:( A )A . 纤维血管瘤B . 出血坏死性息肉C . 乳头状瘤D . 多形性腺瘤 E . 神经鞘膜瘤1 0 .颈部肿块最多见的是:(E )A . 甲状腺肿 B . 神经鞘瘤D . 神经纤维瘤 E . 肿大的淋巴结C.颈动脉瘤1 1 .观察上颌窦首选X 线投照位置为:(D )A . 头颅正位 B . 头颅侧位D . 华氏位 E . 柯氏位C.颅底位12. 诊断鼻窦霉菌的最佳检查方法是:(

347、B )A . 平片 B. CT平扫D. MRI E. B 超C. CT增强扫描1 3 .视网膜母细胞瘤的首选检查方法是:(B )A. B 超 B. CTD . 平片 E . 眼底荧光摄影C.MRI1 4 .下列有关眼眶内海绵状血管瘤,错误的是:(C )A . 是成人最常见的眼眶内原发性肿瘤 B . 中年女性多见C . 眼外肌明显增粗 D . 可有眼球突出E. M R 上明显长”信号1 5 .下列有关眼眶内炎性假瘤,错误的是:(C )A. 一般为单侧 B . 泪腺增大C . 眼外肌增粗,以肌腹增粗为主,肌腱正常 D . 可有眼球突出E . 增强后轻中度强化1 6 .下列有关眼型Graves病的描

348、述,错误的是:(E )A . 多双侧发病 B . 中年女性多见C . 眼外肌增粗,以肌腹增粗为主,肌腱正常D . 可有眼球突出E . 眼外肌增粗,以肌腹、肌腱同时增粗为主17. 诊断视神经胶质瘤,下列检查方法中最佳的是:(D )A . 平片 B. CT平扫D. MRI E. B 超C. CT增强扫描1 8 .关于甲状腺肿物CT检查的描述,不正确的是:(A )A . 确定肿瘤性与非肿瘤性病变非常困难B . 不能鉴别肿瘤的良恶性 C . 不能确定肿瘤的范围D . 增强扫描有助于病变检出 E . 可判断有无淋巴结转移1 9 .下列选项中,不是视网膜母细胞瘤的常见CT表现的是:(C )A . 位于眼球

349、壁内 B . 肿瘤内团块状钙化D . 肿瘤增强明显 E . 肿瘤可向眼球外膨出C.肿瘤密度较低20. 下列选项中关于鼻咽癌CT表现的描述,不正确的是:(D )A. 一侧咽隐窝变浅 B. 一侧腭肌肥大D . 咽旁间隙向内移位 E . 两侧咽腔不对称C.同侧鼓室含气消失第四部分呼吸系统影像诊断单项选择题1. 肺癌引起的阻塞性肺炎与一般肺炎区别有以下几点,不正确的是:(B )A . 体积缩小 B . 有支气管充气像D . 合并肿块 E . 常有淋巴结肿大C . 同一部位反复发作2. 肺内直径小于2 cm病灶的最佳检查方法是:(A )A. HRCT B . 透视D. MRI E . 螺旋 CTC .

350、胸部摄片3.胸部增强CT有利于以下病变的诊断:(A )A . 纵隔淋巴结肿大 B . 肺炎D . 粟粒型肺结核 E . 肺挫伤C . 胸水性质的判断4.胸部增强CT不用于以下病变的诊断:(E )A . 发现肺门淋巴结 B . 纵隔淋巴结肿大C . 肺内结节性病变的区别D . 肺血管性病变 E . 胸水性质的判断5. CT纵隔淋巴结肿大的标准为其短径大于:(E )A. 1 mm B. 0. 5 mmC. 8 mmD. 10 mm E. 15 mm6.构成肺门阴影最主要的结构是:(A )A .肺动、静脉 B . 神经D .肺组织 E . 淋巴结C . 主支气管7. CT肺门淋巴结肿大的标准为其短径

351、大于:(D )A. 1 mm B. 5 mmC. 8 mmD. 10 mm E. 15 mm8. 肺部空腔性病变常见于:(D )A . 肺癌 B . 结核厚壁空洞D . 支气管扩张 E . 气胸C . 虫蚀样空洞9.下列关于病变空洞的描述,错误的是:A . 肺气囊:薄壁空洞 B . 肺脓肿: 均匀厚壁空洞(A )C . 浸润性肺结核:薄壁空洞,有卫星灶 D .肺癌:E , 慢性纤维空洞性肺结核:纤维厚壁空洞偏心空洞,有壁结节10.下列支气管扩张的影像特点,不正确的是:A . 局部肺纹理增多 B . 环状或蜂窝状影C . 斑片状、索条状影(E )D . 可有葡萄征、手套征 E . 普通胸片正常可

352、除外本病11. 大叶性肺炎的典型影像学变化见于病变的:A . 充血期 B . 实变期D . 消散期之初 E . 病变全程C . 消散期(B )12. 肺部肿块内有一90一 50 HU密度区,常见于:A . 结核球 B . 外围型肺癌伴坏死D .错构瘤 E . 肺囊肿C . 炎性假瘤(D )13. 肺间质疾病无纤维化的有:(C )A . 结节病 B . 系统性红斑狼疮D . 矽肺 E . 类风湿关节炎C. 癌性淋巴管炎14. 肺内球形病灶称为肿块的直径应大于:(A )A. 2 cm B. 3 cmC. 4 cmD. 5 cm E. 6 cm1 5 .两肺粟粒型病变除肺结核外,还可见于:(B )A

353、 . 小叶性肺炎 B . 过敏性肺炎D . 吸入性肺炎 E . 中央性肺癌C.支原体性肺炎1 6 .无壁空洞最常见于:(B )A . 周围性肺癌 B . 干酪性肺炎D .肺脓肿 E . 浸润性肺结核C. 肺转移瘤1 7 .粟粒型肺结核是指:(E )A . 原发性肺结核 B . 浸润性肺结核C.慢性纤维空洞型肺结核D . 结核性胸膜炎 E . 血行播散型肺结核1 8 .急性血行播散型肺结核的特点为:(B )A . 渗出病灶为主 B . 增殖性病灶为主D .钙化病灶为主 E . 混合性病灶为主C.纤维性病灶为主19. 下列关于结核瘤的特点,错误的是:(E )A . 较高密度,可有钙化 B . 为

354、2 cm 以下球形影D . 可有小空洞形成 E . 边缘不清晰,有分叶C.常有卫星灶20. 肺 部 X 线表现为锁骨下区纤维条索状阴影,应属于:(C )A . 原发性肺结核 B . 血行播散型肺结核D . 结核性胸膜炎 E . 慢性纤维空洞型肺结核C.浸润性肺结核21. 肺癌性淋巴管炎之微结节常位于:(C )A . 肺门周围 B . 肺底部D . 肺尖部 E . 纵隔旁C.支气管血管束周围2 2 .影像学表现不是浸润性肺结核常见征象的是:(D )A . 肺部病变位于下叶背段 B . 虫蚀样空洞D . 结核球在CT增强时增强明显 E . 胸腔积液C. 球形病灶2 3 .游离性胸膜腔积液在胸部摄片

355、上就可见到,一般体积应为(E )A. 10 ml B. 30 mlC.100 mlD. 200 ml E. 300 ml24. 肺部良性肿瘤和肿瘤样病变最常见的是:(D )A . 平滑肌瘤 B . 纤维瘤D . 错构瘤 E . 腺瘤C.脂肪瘤2 5 .双肺弥漫粟粒样阴影可见于下列病变,除了:(C )A . 血源播散性肺结核 B . 肺转移瘤D . 结节病 E . 矽肺C.支气管肺炎2 6 .外围性错构瘤的特征性表现为:(D )E . 出现胸腔积液和胸膜肿瘤结节3 8 .纵隔畸胎瘤的CT、MRI表现特点如下,其中错误的是: (E )A . 囊性病灶 B . 圆形、椭圆形或不规则形C . 其中可有

356、脂肪、骨骼A . 肿瘤小于2 cmD . 内有爆米花样钙化B . 肿块有分叶E . 空洞形成C . 边缘光滑2 7 .细支气管肺泡癌属于:( D )A . 鳞癌B . 大细胞癌C . 小细胞癌D . 腺癌E . 表皮样癌28. 中晚期肺癌生长方式常表现为:( D )A . 管内型B . 管壁型C . 管外型D . 混合型E . 都不是2 9 .厚壁空洞的洞壁厚度超过:( A )A. 3 mmB. 5 mmC. 7 mmD. 9 mmE. 10 mm3 0 .早期中央型肺癌常表现为:( C )A .肺门淋巴结肿大B . 肺门部肿块C . 支气管腔内结节D . 胸水E . 肾上腺转移3 1 .支气

357、管阻塞引起的支气管扩张常表现为:( E )A . 囊状透亮影B . 卷发状透亮影C . 蜂窝状透亮影D . 含气支气管征E . 柱状条索状3 2 .局限性肺气肿不发生于:( D )A . 中央型肺癌B . 支气管异物C . 中央型错构瘤D . 肺炎E . 肺结核纤维化周围3 3 .中央型肺癌的直接征象不包括:( C )A . 肺门增大B . 肺门肿块C . 阻塞性肺不张D . 支气管壁增厚E . 支气管狭窄3 4 .异物引起的阻塞性肺炎常不出现:( E )A . 小叶或小叶融合阴影B . 肺段实变C . 肺叶实变D . 肺体积缩小E . 肺门部肿块35. 肺转移性肿瘤常不出现:( E )A .

358、 多发性结节B . 粟粒样结节C . 转移灶可有空洞D . 转移灶可有钙化E . 肺叶实变36. 肺挫伤不出现:( D )A . 肺纹理模糊B . 斑片状阴影C . 病灶不按肺段分布D . 肺门淋巴结肿大E . 可合并肺撕裂伤37. CR、MRI上有以下表现,可提示胸腺瘤恶变,错误的是:( C )A . 肿瘤短期内明显增大B . 肿瘤边界不清,向临近组织浸润C . 点状、曲线状钙化D . 轮廓不规则、,分叶状D . 囊壁可有钙化E. MRI不仅可显示囊变、脂肪,而且可显示钙化3 9 .下列关于纵隔神经源性肿瘤的描述,错误的是: (E )A . 多位于后纵隔,以良性肿瘤为主B. MRI对显示伴椎

359、管内的肿瘤更好C. CT呈边缘清楚光滑的肿块,有时可见椎间孔扩大D. MRI上信号复杂,可有不同信号强度E . 有明显钙化40 . 纵隔囊肿常见的有以下几种,错误的是: (E )A . 淋巴管囊肿 B . 支气管囊肿 C . 心包囊肿D . 食管囊肿 E . 假性动脉瘤41 . MRI在肺癌诊断上有以下作用,错误的是: (E )A . 根据流空效应判断肺门及纵隔淋巴结转移B . 判断肿瘤与血管的关系 C. 了解胸壁的侵犯D . 可增强,以便与结核球区别 E . 根据钙化状况,以便与结核球区别42 . 纵隔恶性淋巴瘤有以下表现,错误的是: (C )A . 上纵隔增宽呈波浪状 B . 常多组淋巴结

360、受累 C . 常有包膜钙化D . 可侵犯心包 E . 向肺内浸润呈线状、小结节状43 . 胸内甲状腺有以下表现,错误的是: (D )A . 大多位于胸骨后 B . 恶性者较少见D . 气管常向前侧方移位 E . 气管常向后侧方移位C . 因甲状腺含碘CT值较高4 4 .周围型肺癌可有以下表现,错误的是:A . 分叶 B . 有毛刺D . 空洞形成 E. CT不增强C.有小泡征(E )4 5 .周围型肺癌CT三维重建可有以下显示,错误的是:A . 肿块分叶 B . 胸膜凹陷D . 肿块与血管的关系 E . 清晰显示纵隔淋巴结C.肿块累及胸膜(E )46. 下列胸部CT检查特点的描述,错误的是:A

361、 .横断面显示,克服胸片上的影像重叠B . 对纵隔旁、横膈周围及胸膜下病灶显示教胸片好C . 是肺部疾病诊断的首选方法D . 冠状面和矢状面重建图像分辨率较低E . 对纵隔内软组织分辨率不高(C )47. 胸片上所示的密度减低阴影是指病变密度低于:A . 肺组织 B . 胸大肌D . 肋骨 E . 心脏C.胸壁软组织(A )4 8 .由3 5 支终末细支气管组成的肺结构称为:A .肺小叶 B . 初级小叶D .肺段 E . 肺腺泡C.次级小叶(A )4 9 .构成肺门阴影最主要的结构是:A . 肺动、静脉 B . 神经C.主支气管(A )D . 肺组织 E . 淋巴结50. 胸片上双侧肺门影增

362、大可见于:( B )A . 大叶性肺炎 B . 慢性肺原性心脏病D . 法洛四联征 E . 肺隔离症C . 慢性纤维空洞型肺结核5 1 .无壁空洞最常见于:( B)A . 周围性肺癌 B . 干酪性肺炎D . 肺脓肿 E . 浸润性肺结核C . 肺转移瘤5 2 .最易出现空洞内液平的病变是:( B)A . 浸润性肺结核 B . 急性肺脓肿D . 肺癌 E . 肺囊肿合并感染C . 慢性纤维空洞型肺结核5 3 .双肺弥漫粟粒样阴影可见于下列病变,除了:( C)A . 血行播散型肺结核 B . 肺转移瘤D . 结节病 E . 矽肺C . 支气管肺炎5 4 .大叶性肺炎的典型影像学变化见于病变的:(

363、 B)A . 充血期 B . 实变期D . 消散期之初 E . 病变全程C . 消散期5 5 .下述选项不是原发综合征的影像特点的是:A . 病灶呈哑铃状B . 原发灶多位于右肺上叶下部和下叶肺上部C . 肺门淋巴结炎吸收比肺内病灶快D . 原发灶为中央密度深、边缘淡而模糊的片状阴影E. 一般淋巴管炎不易显示( C)5 6 .继发性肺结核影像的特点,除了:( D)A . 纤维灶 B . 增殖灶D . 肺门淋巴结肿大 E . 肺内病灶新旧不一C . 钙化57. 下 列有关周围性肺癌的胸片影像,错误的是:A . 病变早期与炎症有时难以鉴别 B . 反 “S” 征( B)C . 短小毛刺 D . 癌

364、性空洞,E . 合并炎症时,其周缘模糊少有液平面5 8 .下列关于肺转移瘤的观点,错误的是:( B)A .肺动脉是最常见的转移途径 B . 胃癌是最常见的原发肿瘤C. CT检出率高于胸片 D . 对肺内小灶显示,MRI不及CTE . 肺尖、胸膜下、肋膈角等处病变,胸片易漏诊59. 肺转移瘤最常见的CT表现是:( A)A . 多发结节影 B . 粟粒状影D . 蜂窝状影 E . 孤立性结节影C . 小斑片状影6 0 .下列关于气胸的描述,正确的是:( B)A . 气胸区域可见到肺纹理 B . 张力性气胸,纵隔可向健侧移位C. MRI对本病的诊断价值最高 D . 不同病因气胸的影像表现不同E .

365、单纯性气胸:立位胸片示外高内低致密影6 1 .关于纵隔神经源性肿瘤的诊断要点,错误的是:A . 好发于后上纵隔脊柱旁 B . 多为恶性肿瘤D . 平扫CT值 30 50HU E . 形态规则,边缘光滑C . 多为均匀强化(B)6 2 .下列检查中,最有助于支气管扩张诊断的是:A . 胸片 B . 超声D. MRI D. DSAC . 高分辨率CT扫描(C)6 3 .中央型肺癌的胸片表现,最早出现的通常是:A . 阻塞性肺炎 B . 局限性肺不张D . 肺门肿块影 E . 反 “S” 征C . 局限性肺气肿(C)6 4 .中央型肺癌一般发生在:A . 主支气管 B . 叶支气管D . 段以下支气

366、管 E . 段以上支气管C . 段支气管(E)第五部分循环系统影像诊断单项选择题A . 左、右心缘至体中线的最大距离之和 B . 左心缘至右心缘的距离C . 左心缘至体中线的最大距离 D . 右心缘至体中线的最大距离1. 一般不用于检查心脏的体位是:(E )A . 后前立位片 B . 左前斜位 C . 右前斜位D . 左侧位 E . 右侧位2 . 在远达片后前立位上,右心缘上段为:( A )A . 上腔静脉及升主动脉的复合投影 B . 上腔静脉C . 升主动脉 D . 右心房 E . 左心房3 . 在远达片后前立位上,正常情况下左心缘看不到:( D )A . 主动脉结 B . 肺动脉段 C .

367、 左心耳D . 右心室 E . 左心室4. 4 5 右前斜位片,正常情况下心前缘自上而下为:( A )A . 升主动脉、主肺动脉干、右心室圆锥部、右心室B . 升主动脉、右心室圆锥部、右心室、左心室C . 升主动脉、主肺动脉干、右心室、左心室D . 主肺动脉干、右心室圆锥部、左心房、左心室E . 主肺动脉干、右心室圆锥部、右心房、右心室5. 60。左前斜位片,正常情况下心后缘自上而下为:( C )A . 右心房、右心室 B . 右心房、左心房 C . 左心房、左心室D . 右心室、左心室 E . 右心房、左心室6. X 线平片判断心脏增大的方法,心脏横径是:( A )E . 右膈顶水平胸廓的内

368、径7 . 下列关于二尖瓣型心脏的描述,错误的是: (E )A . 肺动脉段凸出及心尖上翘 B . 主动脉结缩小C . 形如梨形 D . 反映右心负荷或以其为主的心腔变化E . 肺动脉段凹陷及心尖下移8 . 气管隆凸开大,一般见于: (B )A . 右心房增大 B . 左心房增大 C . 右心室增大D . 左心室增大 E . 膈肌上台9 . 右前斜位,服假检查示中下段食管有局限性压迹和移位,此征像表示: (B )A . 右心房增大 B . 左心房增大 C . 右心室增大D . 左心室增大 E . 右心增大10 . 在远达片上,心尖下移、相反搏动点上移、心腰凹陷,此征像表示: (D )A . 右心

369、房增大 B . 左心房增大 C . 右心室增大D .左心室增大 E . 右心增大11 . 在远达片上出现双方影,此征像表示: (B )A . 右心房增大 B . 左心房增大 C . 右心室增大D . 左心室增大 E . 胸主动脉屈曲延长12 . 下列X 线征像中,不是肺血增多的表现的是: (E )A . 肺血管纹理增多 B . 肺动脉段凸出,两肺门动脉扩张,搏动增强C . 扩张的血管边缘清楚 D . 肺野透明度正常E . 肺野内出现扭曲而紊乱的血管影13 . 下列X 线征像中,不是肺血减少的表现的是: (E )A . 肺血管纹理变细 B . 肺门动脉正常 C. 肺野透明度增加D . 肺动脉段凸

370、出 E . 肺静脉扩张14 . 下列X 线征像中,不是肺动脉高压的表现的是: (E )A . 肺动脉段明显凸出 B . 肺门动脉扩张、搏动增强C . 出现肺门截断现象D . 右心室增大 E . 肺纹理增多,边缘模糊15 . 下列关于K 氏B 线的描述错误的是: (D )A . 是一种间隔线 B . 是肺泡间隔水肿增厚的投影C . 见于间质性肺水肿D . 呈网格状影 E . 常见于二尖瓣狭窄和慢性左心衰竭16 . 关于肺泡性肺水肿的描述,正确的是: (D )A. 上 肺静脉扩张、下肺静脉正常或缩窄B . 胸膜下或( 和)胸腔少量积液C . 肺门影增大,边缘模糊D. 一侧或两侧广泛分布的斑片状阴影

371、,边缘模糊,在短期内变化较大E . 肺野透明度增加1 7 .左心衰竭的X 线表现,应除外: (C )A . 较重的肺淤血+ 肋膈角或( 和)叶间少量积液 B . 间质性肺水肿C . 上腔静脉扩张 D . 肺泡性肺水肿 E . 左室房的增大D . 心脏近似靴形 E . 肺动脉段一心腰部膨隆,心尖圆隆2 3 .用于心脏的检查,与 C T 比较,MRI具有下列优势,除了: ( D )A . 可获得标准的三维图像和斜位图像 B . 改变扫描序列可提高病变的识别和鉴别能力C . 无 X 射线的副作用 D . 检查的时间更短E . 对心脏形态、结构和病变的显示能力更强18.目前首选的无创检查房间隔缺损的方

372、法是:(D )A. X 线平片B. CTC. MRID . 超声心动图E . 心血管造影19. 迄今,诊断动脉导管未闭的 “ 金标准”是:(E )A. X 线平片B. CTC. MRID .超声心动图E . 心血管造影20.目前,诊断肺动脉瓣狭窄首选的无创影像技术是:(E )A. X 线平片B. CTC. PETD. MRIE . 超声心动图21.下列畸形,不属于法洛四联征的是:(C )A . 肺动脉狭窄B . 右心室肥厚C . 房间隔缺损D . 主动脉骑跨E . 室间隔缺损22.下列X 线征像,不属于法洛四联征的是:(E )A .肺血减少B . 两肺门动脉细小C . 主动脉升弓部增宽、凸出2

373、4. 下 列 X 线征像,不属于单纯典型风湿性心脏病二尖瓣狭窄的表现:( E )A . 肺淤血 B . 心脏呈“ 二尖瓣”型C.左心房增大D . 右心室增大 E . 肺纹理稀疏2 5 .风湿性心脏病二尖瓣狭窄时,心影呈:( A )A . 梨形心 B . 横位心C.靴形心D . 悬滴状心 E . 球形心2 6 .风湿性心脏病最常见的瓣膜病变是:( A )A . 二尖瓣狭窄 B . 二尖瓣关闭不全C.三尖瓣狭窄D . 主动脉瓣狭窄 E . 主动脉瓣关闭不全27. 心室壁瘤的X 线表现不包括:( E )A. 左室缘局限性膨凸 B . 左心室“ 不自然”增大C.左室缘博动异常D .左室壁钙化 E .

374、心腰部膨凸,心尖圆隆上翘2 8 .下列疾病特点是心脏局部出现矛盾运动的是:( E )A . 缩窄性心包炎 B . 心肌炎C.心包积液D . 克山病 E . 室壁瘤29. 肺原性心脏病,心影有时可呈:( C )A . 梨形心 B . 横位心C.悬垂型心D . 靴形心 E . 主动脉型心3 0 .下列对于肥厚型心肌病的描述,错误的是:( E )A . 心肌肥厚 B . 心腔不扩张,且多缩小、 变形C . 最常累及肌部室间隔 D . 心室舒张受限E . 心室的收缩功能降低大量心包积液的典型X 线征象,应除外:A . 肺纹理增多、增粗 B . 心影向两侧扩大C . 心腰及心缘各弓的正常分界消失 D .

375、 心缘搏动普遍减低E . 短期内心影大小可有明显变化诊断缩窄性心包炎较可靠的X 线征象是:A . 心脏搏动减弱D . 间质性肺水肿B . 心缘僵直,正常弧形消失C . 心包钙化E . 上腔静脉影增宽用 CT来诊断缩窄性心包炎,错误的是:A . 平扫显示心包异常不规则增厚 B . 不能发现X 线平片未能显示的钙化灶C. EBCT增强扫描能观察到左、右心室内径缩小D . 可显示腔静脉扩张,左、右心房扩大 E . 可显示腹水及胸腔积液关于胸主动脉瘤的描述,错误的是:A . 可分为真性和假性动脉瘤B . 真性动脉瘤可分为囊状、梭形和混合型C . 假性动脉瘤的瘤壁由动脉壁的3 层组织构成D . 基本X

376、线征象是纵隔阴影增宽或形成局限性块影E . 肿块或纵隔增宽影可见扩张性搏动3 5 .关于胸主动脉夹层动脉瘤的描述,错误的是: (C )A. Debakey将主动脉夹层分为三型B . 主要症状是胸背痛、气短、咳嗽、声音嘶哑等C . 特征性CT征象是内膜钙化外移,并显示真假腔及内膜瓣片D. CT增强扫描可显示附壁血栓E. CT三维重建可从不同解剖角度观察其主要征象和病变范围36 . 下列征象中,不属于右心室增大的表现的是: (E )A . 心尖圆隆、上翘 B . 心腰凸出 C . 右前斜位示圆锥部膨凸D . 左前斜位示心前缘右室段向前膨凸 E . 左前斜位示心室间沟向前下移位37 . 一般来说,成

377、人心胸比率的正常上限为: (B )A. 0. 40 B. 0. 50 C. 0. 55D. 0. 60 E. 0. 6538 . 下列哪个征象不属于肺栓塞的X 线表现: (E )A. 一侧或某个区域肺血管纹理显著稀疏 B . 同侧肺门或相应叶、段动脉阴影细小C . 对侧肺门阴影扩张 D . 叶、段动脉或分支粗细不均,走行异常E . 心影呈主动脉型39 . 正常成人右下肺动脉主干直径一般为: (D )A. W 3 mm B. W 8 mm C. W 10 mmD. W 15 mm E. W 20 mm4 0 .右前斜位X 线摄片上食管压迹加深,提示: (E )A . 左心室增大 B . 右心房增

378、大 C . 右心室增大D . 主动脉增宽,扭曲 E . 左心房增大4 1 .风湿性心脏病二尖瓣病变时肺血管的改变是:( C )A . 肺充血 B . 肺出血 C . 肺淤血D .肺栓塞 E . 肺缺血4 2 .下述风湿性心脏病二尖瓣狭窄的影像,错误的是:( E )A . 间质性肺水肿 B . 肺门影增大 C . 梨形心D . 双房影 E . 主动脉结增大4 3 .左前斜位上,心后间隙消失见于:(B )A . 左心房增大 B . 左心室增大 C . 右心房增大D . 右心室增大 E . 二尖瓣狭窄44. 肺血减少的X 线平片表现,除了:( C )A. 肺门缩小 B . 右下肺动脉干变细 C .

379、肺野透亮度减低D . 肺野内见走行紊乱的网状血管影 E . 肺纹理稀疏4 5 .下列关于MRI检查循环系统的描述错误的是:( A )A. MRI仅可从冠状面、矢状面与横断面来显示心脏大血管的解剖结构B. MRI在循环系统的临床应用广泛C. MRI可用于检查心脏瓣膜疾病D . 屏气法MRI电影可观察左室收缩率的变化E . 心肌标记技术可以鉴别慢速血流和血栓46. MRI用于诊断主动脉瘤,错误的是:( B )B . 可显示瘤壁钙化 C . 可显示瘤内附壁血栓E . 可显示受累的动脉大分支A . 可显示瘤体大小D . 可显示瘤腔外的情况47. MRI可显示下列心脏结构,除了:A . 房室间隔B .

380、心肌壁厚度(E )C . 心腔大小D . 人工生物瓣膜E . 金属瓣膜48. M RI显示主动脉夹层真腔与假腔主要基于两者血流速度不同。以下叙述中正确的是:(C )A . 真腔内血流速度快, 一般显示为高信号B . 真腔内血流速度慢, 一般显示为低信号C . 真腔内血流速度快,一般显示为无信号D . 真腔内血流速度快,- - 般显示为低信号E . 真腔内血流速度慢,一般显示为高信号49. MRI诊断心包积液的优点,除了: ( C )A . 发现少量心包积液敏感 B . 定位准确 C . 价格便宜D . 可作半定量分析 E . 对包裹性积液定位较佳50. MRI诊断法洛四联征的优点是: (E )

381、A . 大视野 B . 多体位直接成像C . 空间分辨率优于超声心动图 D . 对左右肺动脉观察优于超声心动图E . 以上都是第六部分腹部影像诊断单项选择题I . 关于食管的影像解剖描述,下列错误的是: (C )A . 食管入口处相当于颈6 椎体水平,止于胸10 11椎体水平与贲门连接B . 食管全长约25 30 c m ,自口齿至贲门约40 45 c m ,食管宽约2 3 cmC . 左前斜位,吞钢造影食管前缘可见主动脉弓压迹、左主支气管压迹和左心房压迹D . 第三蠕动波常见于老年人,边缘可呈波浪状或锯齿状E . 吞领造影时,食管黏膜像可见3 5 条食管黏膜线2 . 下述正常食管X 线解剖中

382、,错误的是: (D )A .通常有3 个狭窄 B . 老年人可见第三收缩波 C . 通常分颈段、胸段、腹段D . 黏膜纹与胃相似 E . 钢剂检查时可见到3 个生理压迹3 . 下述胃X 线解剖知识,错误的是: (D )A . 角切迹至幽门间为胃窦 B . 仰卧位检查,胃窦靠右侧,胃底靠左侧C . 胃大弯黏膜皱裳比胃小弯黏膜皱髅粗 D . 胃底黏膜皱裳走向比胃窦部黏膜皱裳规则E . 胃窦部胃小区显示率最高4 . 正常胃体部黏膜皱裳宽度一般不超过: (C )A. 3 mm B. 4 mm C. 5 mmD. 6 mm E. 8 mm5 . 下列关于胃的领剂检查的说法,错误的是: (A )A . 正

383、常胃一般可同时有3 4 个收缩波 B . 胃蠕动一般由胃体上部开始C . 胃蠕动为向心性收缩 D . 胃排空时间一般为1.5 3 hE . 胃排空与胃张力、蠕动和幽门功能等有关6 . 上消化道钢餐检查,正常空肠黏膜皱殿多呈: (C )A . 弹簧状 B . 腊肠状 C . 羽毛状或雪花状D . 鱼骨状 E . 线状7 . 右侧膈下游离气主要须相鉴别的症状是: (B )A . 胃泡 B . 间位结肠 C . 先天性巨结肠D . 气胸 E . 气肿性胆囊炎8 . 立位片上,腹部游离气体的典型影像为: (C )A . 球形 B . 不规则形 C . 新月形D . 半球形,有 “ 气液面” E . 与

384、胸腔间距大于15 m m9 . 下列疾病中,没有龛影表现的是: (A )A . 食管憩室 B . 胃溃疡 C . 胃癌D . 食管癌 E . 肠结核10 . 食管静脉曲张的影像学检查方法中最常应用的简便有效的方法是: (A )A . 食管铁餐检查 B . 食管CT检查 C . 食管MRI检查D . 血管造影检查 E . 超声检查11 . 下列食管癌与食管静脉曲张的鉴别诊断中,最有价值的是: (E )A . 发生部位 B . 呕血 C . 男性多见D . 中老年多见E . 食管饮餐透视观察食管的蠕动情况1 2 .关于食管静脉曲张的描述,不正确的是: (E )A . 是门静脉高压的主要并发症,常见

385、于肝硬化患者B . 轻度患者,病变主要局限于食管下段,黏膜皱裳稍增宽,管腔边缘稍不平整C . 中度患者,病变累及食管中下段,黏膜皱裳增粗呈结节状或串珠状充盈缺损D . 重度患者,可累及食管全长,黏膜皱嚷增粗呈结节状或串珠状充盈缺损,排空延迟E .只有中度或重度患者才能在吞领造影时发现13 . 早期食管癌的检出率在检查中较高的方法是: (A )A. X 线 B. CT C. MR1D . 超声 E. DSA14 . 疑有食管异物,在 X 线检查时,下列说法错误的是: (C )A . 食管异物最易停留在食管的生理狭窄处B . 透视可清晰显示如金属或骨类的不透X 线异物影C .透 X 线的异物需要用

386、钢棉检查,无挂棉现象即可排除异物D . 食管异物可引起食管周围脓肿E . 食管异物可引起大血管破裂导致大出血15 . 关于食管平滑肌瘤,下列描述错误的是: (C )A . 患者病史较长,可以有进食梗阻和胸骨后疼痛感B . 肿瘤边界锐利,充盈缺损明显C . 肿瘤表面领剂涂抹不均匀,可有小的溃疡形成D . 食管壁光滑柔软,食管蠕动正常E . 食管黏膜皱裳无黏膜破坏,典型征象可见“ 环形征”16 . 早期食管癌的描述,错误的是: (B )A . 指局限于黏膜和黏膜下层的病变B . 不论病变范围多大,无周围及远处淋巴结转移C . 吞饮检查可见黏膜有中断和破坏D . 吞铁检查可见小的龛影和充盈缺损E .

387、双对比造影和食管吞钏连续摄影检查可以提高阳性检出率17 . CT可对食管癌进行分期,对其可切除性和预后的评估有重要作用。下列叙述中恰当的是:(E )A. 一期:腔内肿块,食管壁无增厚,无纵隔蔓延或转移,食管周围脂肪清晰B . 二期:食管壁增厚超过5 mm,无纵隔蔓延或转移,食管周围脂肪清晰C . 三期:食管壁增厚并累及周围组织,纵隔淋巴结转移,无远处转移D . 四期:有远处转移E . 以上都对1 8 .反流性食管炎,下列叙述错误的是: (E )A . 反流性食管炎又称消化性食管炎,可以分为生理性或病理性两种B . 轻度患者,仅见食管下段痉挛性收缩和黏膜皱裳增粗以及食管反流C . 中度患者,食管

388、下段痉挛明显,可见小结节样充盈缺损D . 重度患者,食管狭窄明显,可呈漏斗状,食管壁毛糙并可见小的溃疡形成E .钢餐检查时,只有立位看到食管反流,才能诊断反流性食管炎19. 3 8 岁女性患者,钢餐示食管壁张力减低,蠕动减弱,领剂排空延迟,并在食管下段见到串珠状充盈缺损影。首先考虑:A . 反流性食管炎D . 食管癌B . 食管下段平滑肌痫E . 食管裂孔疝(C )C . 食管下段静脉曲张20. 26岁,因进食困难,加 重 1 年就诊。钢餐检查示食管重度扩张,黏膜完整,蠕动减弱,食管下段贲门开口处呈鸟嘴样狭窄,A . 贲门失弛缓症D . 食管下段静脉曲张胃溃疡好发于:A . 胃底部D . 胃窦

389、部大弯侧钢剂通过缓慢。最困难的诊断为:B . 贲门癌E . 食管下段憩室B . 胃体部E . 幽门部C . 反流性食管炎C . 胃窦部小弯侧胃溃疡的征象中,下列叙述最为恰当的是:A . 胃溃疡可以见到龛影,其形态可以多变,可呈乳头状、半圆形或锥形B . 根据龛影周围水肿带的不同龛影有黏膜线、项圈征和狭颈征C . 溃疡周围黏膜可以呈放射状改变,为溃疡修复性改变D . 可以引起胃的变形,幽门梗阻可导致胃扩张和胃潴留E . 以上叙述都正确2 3 .溃疡出现下列征象中,提示溃疡可能发生恶变的是: (D )A . 溃疡龛影形态呈半圆形突出于胃腔之外B . 龛影周围发现项圈征和狭颈征C . 溃疡周围胃变形

390、或胃潴留D . 溃疡周围出现充盈缺损、黏膜破坏或环堤形成E . 以上都不正确24 . 胃癌好发于: (D )A . 贲门部 B . 胃底部 C . 胃体部D . 胃窦部 E . 幽门部25 . 下列关于早期胃癌的描述,错误的是: (E )A . 病变局限于黏膜层 B . 病变局限性黏膜层或黏膜下层C . 病变的大小与早期胃癌的定义无关 D . 有无周围淋巴结转移与早期胃癌的定义无关E . 病变仅累及黏膜肌层26 . 早期胃癌,下列影像学检查方法中首选的是: (C )A . 传统的钢餐检查技术 B . 气钢双重检查技术 C . 低张气饮双重检查技术D. CT E. MRI27 . 溃疡型胃癌的X

391、 表现,下列选项错误的是: (C )A . 溃疡大而浅,位于胃轮廓之内,称为腔内龛影B . 龛影周围可见环堤和指压迹样改变 C . 溃疡周围出现项圈征或狭颈征D . 溃疡周围胃黏膜中断、破坏 E . 病变部位胃壁僵硬,蠕动消失28 . 下列X 线征象中,可作为食管静脉曲张与食管癌的鉴别要点的是: (D )A . 充盈缺损 B . 管腔边缘凹凸不平 C . 钢剂通过缓慢D . 管壁仍见扩张与收缩现象E . 纵行黏膜皱联消失2 9 .食管贲门失弛缓症与食管下段癌的鉴别要点是:( C )A . 狭窄上方食管明显扩张 B . 食管蠕动减弱或消失C . 狭窄段黏膜完整,管壁柔软 D . 钢剂通过缓慢E

392、. 扩张食管内出现液平30.31.32.33.34.35.36.37.38.39.关于胃窦炎的描述,错误的是:A . 肩胛征 B . 胃窦黏膜粗大C . 黏膜皱裳斜行、横行或交叉排列 D . 胃窦张力增高E . 充盈像,胃窦轮廓呈细齿状下述胃溃疡征象中,提示为恶性的是:A . 项圈征 B . 狭颈征 C . 黏膜纠集D .腔外龛影 E . 局部胃壁僵硬下列中晚期胃癌分型中,不包括:A . 增生型 B . 表浅型 C . 浸润型D . 溃疡型 E . 混合型下述小肠Crohn病的影像,应除外:A . 环肠壁对称性侵犯 B . 小肠多发狭窄 C . 鹅卵石征D .肠壁增厚 E. 口疮样小溃疡十二指

393、肠溃疡好发于:A . 球部 B . 球后部 C . 降部D . 水平部 E . 升部下列有关溃疡型胃癌的X 线征象,错误的是:A . 放射状黏膜纠集,直抵龛影口部 B . 不规则状黏膜纠集,杵状增粗C . 腔内龛影 D . 指压迹 E . 蠕动消失下列为良性溃疡的X 线表现,错误的是:A . 狭颈征 B . 项圈征 C . 肩胛征D . 放射状黏膜纠集 E . 溃疡口部黏膜线肠道结核好发于:A . 空肠 B . 回肠 C . 回盲部D . 横结肠 E . 乙状结肠关于单纯性小肠梗阻的X 线表现描述,错误的是:A . 阶梯状液气平面 B . 大跨度的洋 C . 鱼肋征D . 透视下可见液面上下移

394、动E . 假肿瘤征绞窄性肠梗阻的X 表现,错误的是:A .假肿瘤征 B . 咖啡豆征 C . 大跨度的襟D . 多个小跨度卷曲肠襟 E . 空回肠换位征( A )( E )( B )( A )( A )( A )( C )( C )( E )( C )40.腹部外伤,尤其是疑有实质性脏器损伤时,影像学检查应首选:A. X 平片 B . 超声或CT C. MRID. DSA E , 同位素( B )(E )4 1 .大肠癌好发于:(A )A . 直肠和乙状结肠 B . 乙状结肠和降结肠D . 升结肠 E . 盲肠和回盲部C . 横结肠42 . 消化道息肉最好发于:A . 食管 B . 胃 C .

395、 空肠和回肠D . 升结肠和横结肠 E . 直肠和乙状结肠43 . 下列不同类型肠梗阻的影像描述,错误的是:A . 不完全性肠梗阻:结肠内可见到少量气体影B . 粘连性肠梗阻:假肿瘤征C . 绞窄性肠梗阻:空回肠换位征D . 小肠高位梗阻:消化管无明显积气、扩张E . 麻痹性肠梗阻:全消化管不同程度积气、扩张44 . 皮革胃属于:A . 溃疡型胃癌 B . 簟伞型胃癌 C . 浸润型胃癌D . 混合型胃癌 E . 平坦型胃癌45 . 关于结肠息肉,下列描述中错误的是:A . 好发于直肠和乙状结肠 B . 多见于儿童C . 息肉病有明显的家族遗传性 D . 病理上是炎性增生性病变E . 双重造影

396、时呈环形阴影,轮廓光整,密度均匀46 . 下列关于结肠癌的描述,错误的是:A . 好发于降结肠 B . 大多数结肠癌是腺癌C . 浸润型表现为肠腔狭窄,局限于一侧或呈环形D . 增生型表现为腔内充盈缺损 E . 混合型多是晚期表现47 . CT平扫时,正常肝实质平均CT值为:A. 0-30 HU B. 30 40 HU C. 50-70 HUD. 75100 HU E. 100-130 HU48 . 增强扫描动脉期,肝实质强化程度:A . 无明显强化 B . 比脾脏高 C . 比腹主动脉高D . 比肾脏高 E . 与脾脏强化程度相仿(EBCEACA)4 9 .男性患者,4 5 岁,突发性右上腹

397、痛,急诊C T 平扫发现肝右叶后下段有一约6 cmX6 cmX 7 cm大小低密度病变,境界较清楚,密度不均匀,病变内尚可见片状高密度影。应首先考虑为:A . 肝囊肿合并出血 B . 胆管细胞癌 C . 肝转移瘤D . 肝脓肿 E . 原发性肝癌破裂出血50 . 关于肝细胞癌的MRI表现,下列说法错误的是: (D )A. 1 W I呈低信号,病变信号不均匀 B. B W I呈稍高信号,比正常肝组织信号稍高C . 可见假包膜 D. TAVI随回波时间延长,病变信号逐渐增高E . 增强扫描病变可有轻到中度强化51 . 原发性肝癌的影像检查描述中,不正确的是: (E )A . 病变可以单发或多发,病

398、变密度多为低密度,但也可以是等密度或高密度B . 增强扫描的典型特点为“ 快进快出”C . 可有门静脉或下腔静脉瘤栓形成D . 可伴有肝硬化、门脉高压、腹水以及腹腔、肝门淋巴结肿大E. MR组织分辨率较高,对大部分肿瘤都能清楚地显示病变与正常肝组织的界限5 2 . CT平扫,肝脏密度一致性减低,CT值低于脾脏。应首先考虑为: (D )A . 弥漫性肝癌 B . 肝脓肿 C . 血色病D . 脂肪肝 E . 肝淋巴瘤5 3 .肝脏局灶性增生影像表现,下述说法不正确的是: (D )A . 病变多位于肝包膜下,密度均匀,呈等密度或低密度B . 病变较大时,其中心可见放射状分布的纤维瘢痕C . 增强早

399、期呈均匀一致的明显强化,中心部位可以不强化D . 静脉期或延期扫描,病变密度降低,最后呈相对低密度和肝脏密度相一致E. MR表现和CT扫描相似,【W I呈等信号或低信号,T,WI呈稍高信号,增强明显强化5 4 .中年女性,黄疸1 个月,实验室检查提示梗阻性黄疸,C T 扫描示肝脏一致明显增大,肝内外胆管无明显扩张。应首先考虑为: (D )A . 肝内胆管结石 B . 肝门部胆管癌 C . 壶腹癌D . 原发性胆汁性肝硬化 E . 胆总管结石5 5 .下列关于肝腺瘤的叙述,错误的是: (C )A . 女性多见,与服用类固醇药物有关B . 瘤内没有胆管结构和肿瘤细胞,主要由排列紊乱的正常肝细胞组成

400、C. CT平扫可以呈等密度或高密度、圆形或类圆形病变D . 增强扫描早期多呈明显均匀强化,随后密度下降变为等密度或低密度E . 因肿瘤富血供,有出血倾向,应手术切除5 6 . 下述肝腺瘤影像,错误的是: (C )A. CT平扫可以呈等密度或低密度、圆形或类圆形病变B. CT增强扫描表现为富血管肿瘤的强化特点C . 早期多呈明显不均匀强化,随后变为等密度或低密度D. M R平扫可见连续或不连续的包膜E. TTWI呈等信号或低信号,T?WI呈稍高信号,因出血等原因病变信号可以不均匀5 7 .肝脏血管瘤的MRI描述,错误的是: (E )A . 典型的血管瘤TiWI呈低信号,病变边界清楚B. TzWI

401、呈高信号C. T.WI随回波时间延长,病变信号逐渐增高,可 见 “ 灯泡征”D . 增强扫描病变呈明显强化E . 增强扫描一般不强化5 8 .靶征是哪种病变的影像特点: (E )A . 肝囊肿 B . 肝脓肿 C . 肝脏炎性假瘤D . 原发性肝癌 E . 转移性肝癌59. MRI扫描,包膜征最常见于 (A )A . 原发性肝癌 B . 转移性肝癌 C . 肝血管瘤D . 肝腺瘤 E . 炎性假痛6 0 . 1 加权像略低信号、质子加权像和TZ加权像明显高信号的现象,称为: (C )A . 晕环征 B . 牛眼征 C . 电灯泡征D . 不定形征 E . 靶征61 . 灯泡征最常见于: (B

402、)A . 原发性肝癌 B . 肝血管瘤 C . 肝脓肿D . 转移性肝癌 E . 肝囊肿合并出血62 . 下列关于肝脓肿的MR表现,错误的是: (C )A. T W I呈低信号 B. EW I呈高信号C . 脓肿腔内发现气体影不是肝脓肿的表现D . 增强扫描脓肿壁可以呈环状强化E . 脓肿壁的信号稍高于脓腔,但低于正常肝组织63 . 关于胆管细胞癌,错误的是: (E )A . 好发于肝门部,通常可引起梗阻性黄疸B. MRI能较好地显示肝门区的肿块C. MRCP可以显示胆管梗阻的部位 D. MRI能显示扩张的胆管E. MRI分辨肝门区肿大的淋巴结不如CT清楚64 . 增强CT扫描,转移性肝癌最常

403、见的强化类型是: (B )A . 充填式强化 B . 环形强化 C . 轻到中度不均匀性强化D . 明显均匀强化 E . 包膜征65 . 下列关于胆囊癌的描述正确的是: (C )A . 多为鳞癌 B . 直接侵犯以消化管最常见C . 多为浸润型,以肝脏浸润常见 D . 胆囊造影表现正常就可除外胆囊癌E . 胆囊壁增厚是本病特有影像66 . 女性,56 岁,右上腹隐痛3 个月就诊。腹部平片示右上腹一圆形致密影,侧位片示致密影位于脊柱前方。应首先考虑为: (B )A . 右肾结石 B . 胆囊结石 C . 淋巴结钙化D . 右输尿管结石 E . 以上都不是67 . B 超发现右肝一直径约3 cm低

404、回声,C T 平扫示右肝一直径3 cm 低密度病变,境界清楚,密度均匀,CT值为10 H U ,增强扫描无强化。首先考虑为: (B )A . 肝转移癌 B . 肝囊肿 C . 肝血管瘤D . 肝脓肿 E . 局灶性脂肪变性68 . 患 者 10岁,上腹痛三月余。体检右上腹可扪及一直径约5 cm 大包块,CT扫描发现肝门区有一梭形囊性包块,增强扫描无强化,肝内胆管轻度扩张。其最可能的诊断为:(B )A . 胆囊炎 B . 先天性胆总管囊肿 C . 胆囊癌D . 胰腺囊肿 E . 十二指肠憩室69 . 下列有关肝血管瘤CT表现的描述,错误的是: (B )A . 通常是超声首先发现,超声多为强回声表

405、现B. CT平扫通常呈高密度样团块C. CT增强扫描,早期呈结节样强化,随增强时间延长,病变不断被造影剂充填D . 增强呈快进慢出的特点E . 延迟扫描,病变甚至不能被发现70. 男性患者,56岁,右上腹隐痛一月余,伴有低热。 超声发现左肝内低回声病变, 直径约5cm 大小,CT平扫左肝内低密度病变,增强病变呈环状强化。应考虑为: (E )A . 转移癌 B . 囊肿 C . 肝脏亚急性血肿D . 肝脏血管瘤 E . 肝脓肿71.下列关于肝转移癌的描述中,错误的是: (E )A . 常见于消化道肿瘤的转移 B . 通常是多发C . 典型的CT征像是牛眼征或靶征 D . 增强扫描病变呈多样性表现

406、E . 常常有肝硬化的病变基础72.下列关于胰腺炎的CT检查的描述,错误的是: (E )A .胰腺多为弥漫性肿大 B . 胰腺密度减低 C . 胰腺边缘变模糊D . 胰腺坏死为低密度,出血为高密度 E . 增强扫描胰腺肿块呈明显强化73.4 3 岁男性,上腹隐痛半年就诊,ERCP示主胰管粗细不均,扭曲,僵硬,胆总管下端向内移位。应首先考虑: (B )A . 急性胰腺炎 B . 慢性胰腺炎 C . 胰腺假性囊肿D . 胰腺癌 E . 先天性胆总管囊样扩张症74.典型急性出血性坏死性胰腺炎患者,CT扫描可见到: (D )A .胰腺表面光滑 B . 胰腺萎缩 C . 胰腺边缘锐利D . 肾前筋膜增厚

407、 E . 胰腺密度均匀75.下列急性胰腺炎CT检查的表现,错误的是: (B )A . 胰腺弥漫性肿大 B . 胰腺假性囊肿形成 C . 胰腺轮廓不清D . 胰内、胰周出现边缘不清的低密度区 E . 肾前筋膜增厚76.中年男性, 上腹隐痛四月余就诊。CT示胰头肿大,密度尚均匀,肝内外胆管无明显扩张,半年后随访,病灶无明显变化。应首先考虑: (C )A .胆总管结石 B . 胰头癌 C . 胰头炎症D .胆管炎 E . 十二指肠憩室炎77.关于慢性胰腺炎CT表现的描述,错误的是: (B )A .胰管可呈串珠状扩张,偶可中断不连续 B . 胰腺明显肿大C . 胰腺内可见到钙化斑 D . 肾周筋膜增厚

408、 E . 可有胰腺假性囊肿形成78.下列关于胰腺癌的CT表现,错误的是: (C )A . 平扫时胰腺肿块为低密度 B . 可伴有肝内外胆管和胰管的扩张C . 增强扫描动脉期,病变由低密度肿块变为高密度肿块D . 可以侵犯胰周血管 E . 可伴有后腹膜淋巴结转移79.脾脏良性肿瘤最常见的是: (A )A . 血管瘤 B . 纤维瘤 C . 血管内皮细胞瘤D . 囊肿 E . 错构瘤80.脾肿大最常见于的病变是: (E )A . 炎症D . 白血病B . 寄生虫E . 门脉高压C . 肿瘤81. 下 列关于脾脏的影像学检查,叙述错误的是: (E )A . 脾脏血管瘤的CT、MR表现与肝脏血管瘤的C

409、T、M R表现相类似B . 脾脏转移瘤也可见“ 牛眼征”或 “ 靶征”C . 脾囊肿CT表现可为圆形低密度,CT值 0 20 HU,增强扫描无强化D . 脾梗死CT表现为脾内三角形低密度影,尖端指向脾门,增强扫描无强化E. CT增强扫描动脉期,脾脏转移瘤显示最清楚82 . 泌尿系统的有关解剖,下列说法错误的是: (D )A . 腹部平片通常仅能显示肾脏的轮廓,而输尿管和膀胱常常不能显示B . 输尿管有3 个生理性狭窄,分别是与肾盂连接处、越过骨盆边缘处和进入膀胱处C . 尿道也有3 个生理性狭窄,分别是尿道内口、尿道膜部和尿道外口D . 肾脏为腹膜后脏器,通常右侧肾脏比左侧肾脏位置稍高1 2

410、cmE . 肾盏可分为上、中、下三组,肾盂可分为喇叭形、分枝形和壶腹形三型83 . 肾 脏 MRI的表现,错误的是: (C )A. 由 于 肾脏周围有脂肪组织的衬托,肾脏显示比较清楚B. T,WI , 肾皮质信号比髓质信号高C . 冗W I上,皮、髓质信号都高,难以分辨D. TzWI上,肾皮质信号比髓质信号低E . 肾窦脂肪在T.WI和 T.WI上分别呈高信号和中等信号84 . 关于泌尿系统结石,叙述错误的是: (B )A .尿路结石多位于肾脏、输尿管或尿道的生理性狭窄处B . 肾脏结石表现为肾区内形态多变的高密度影,侧位片高密度影多位于脊柱的前方C . 输尿管结石表现为输尿管行径上高密度影,

411、其纵轴与输尿管走行常常一致D . 输尿管下端结石尿路造影可以有肾积水、肾脏延迟显影、不显影,也可以表现正常E . 尿路造影时肾盂、膀胱内结石可表现为充盈缺损85 . 关于肾脏结核的影像学检查,下列叙述错误的是:( E )A . 平片可发现肾脏的改变,包括肾脏体积的缩小、肾脏多发性形态不的钙化灶B . 肾小盏正常杯口结构消失,肾小盏变形或消失,可见虫蚀样破坏C . 因为发生干酪样坏死形成空洞,造影可有“ 云朵状” 、“ 小水潭样”改变D . 晚期可形成“ 肾自截” ,输尿管、膀胱也可受累E. CT可较好地显示肾脏结核病变的钙化和脂肪86 . 下列肾脏疾病中,常见不到钙化斑的是:A . 肾结石 B

412、 . 肾癌D . 肾结核 E . 肾错构瘤87 . 下列关于肾癌的MRI表现,叙述错误的是:A . 肿块在TiWI上,其信号要稍低于正常肾皮质(E )C . 肾囊肿(D )B . 小的肿块信号可以均匀C . 较大的肿块因出血、坏死和囊变,信号可以不均匀D. MRI增强扫描一般不强化E . 有时可以有假包膜征88 . 关于肾盂癌的描述,不恰当的是: (E )A .1 加权,肾盂内可见肿块影 B .1 加权,肿块信号比尿液信号高C. R 加权,肿块信号比尿液信号低 D . 可以伴有肾盂积水E . 造影时肾盂内充盈缺损就可诊断为肾盂肿瘤89 . 关于肾错构瘤的MRI表现,错误的是: (B )A .

413、肿块内有脂肪信号即可诊断为错构瘤B . 肿块内脂肪信号TiWI为低信号,R W I为高信号C . 增强扫描,肿块可以有不均匀性强化D . 肿块可以有出血,为长1W I、长 RW I信号E. CT扫描也能发现肿块内脂肪组织90 . 下列关于肾上腺病变MRI表现的描述,错误的是: (B )A . 长 1W I、长 T?WI信号,并且无强化的类圆形肿块,通常是肾上腺囊肿B . 较大的类圆形肿块,信号不均匀,并且呈明显均一的强化,多为嗜铭细胞瘤C . 不均质性肿块,高信号可以被脂肪抑制序列抑制的病变,可为肾上腺髓质瘤D. 肾上腺肿块T.WI和 T.WI均和肝实质的信号相类似,可为肾上腺腺瘤或转移瘤E.

414、 MRI除了横断面外,可以矢状面和冠状面扫描,可以清楚地显示肾上腺病变91 . 肾上腺疾病,下列叙述错误的是: (C )A . 肾上腺皮质癌,肿块常大于5 c m ,肿块内密度多不均匀,易发生囊变、坏死和出血B . 肾上腺腺瘤肿块多较小,多为1 3 c m ,增强呈轻度强化C . 肾上腺转移瘤,可为单侧或双侧,MRI表现其信号和肝脏基本相似D . 嗜倍细胞瘤,可单侧或异位,CT扫描其密度不均匀,增强有明显强化E. Cushing综合征可由肾上腺增生、肾上腺腺瘤和肾上腺皮质癌引起92 . 可同时累及两侧肾上腺的疾病有: (E )A . 转移癌 B . 结核 C . 增生D . 嗜格细胞瘤 E .

415、 以上都可以93 . 关于前列腺癌的MRI描述,错误的是: (B )A . 前列腺癌多发生于边缘区 B . 1 加权,瘤体为等高信号C . 加权,瘤体为稍低信号D. T?加权,瘤体信号有所增高,瘤体周围有低信号E . 前列腺癌检出,主要以T?加权为主94 . 前列腺癌的影像学检查,下列叙述中错误的是: (D )A . 肾溢造影膀胱显像期无助于前列腺增生和前列腺癌的鉴别B . 前列腺癌多发生在前列腺的外围叶,MR组织分辨率高,有助于病变的显示C. T2加权像,外围带呈高信号改变,内如发现低信号结节影要考虑前列腺癌D. T2加权像,外围带呈低信号改变,内如发现高信号结节影要考虑前列腺癌E . 用直

416、肠线圈来进行前列腺MRI检查,可以提高前列腺癌的检出率95. 下列关于腹膜后肿瘤,叙述错误的是: (D )A . 腹膜后肿瘤,常因临床症状出现迟,发现时肿块较大,因此常常难以作出定性诊断B . 脂肪肉瘤仍然可以发现脂肪成分,但增强扫描时肿块会有明显强化C . 平滑肌肉瘤常常较大,增强扫描呈不均匀性强化,肿瘤中心可发生坏死囊变和出血D . 神经源性肿瘤一般境界不清楚,可发生钙化,增强扫描呈均一性明显强化E . 腹膜后纤维化病程进展性改变,可累及输尿管而导致肾盂积水9 6 .上消化道检查应检查至:( C )A . 食管B . 胃C.十二指肠D . 空肠E . 回肠9 7 .下列X 线征象中,提示为

417、恶性的是:( B )A . 放射状黏膜纠集B . 腔外龛影C.环堤D . 项圈征E . 狭颈征98. 下 列消化道功能异常,根餐不能检出的是:( B )A . 张力增高B . 分泌减少C.逆蠕动D . 排空加快E . 分泌增加9 9 .肩胛征常见于:( B )A . 胃溃疡B . 胃窦癌C.十二指肠溃疡D . 胃体部癌E . 结肠癌1 0 0 .发现尿路阳性结石最常用的方法是:( B )A. B 超检查B . 腹部平片检查C. 腹部CT检查D . 静脉尿路造影检查E . 逆行尿路造影检查1 0 1 .自截肾常见于:( B)A. 慢性肾炎B . 肾结核C.肾结石D . 肾癌E . 慢性肾盂肾炎1

418、02. 诊断肾上腺疾病的最佳方法是:( C )A . 腹膜后充气造影检查B . 腹部平片检查C.腹部薄层CT检查D . 肾上腺血管造影检查E . 静脉肾盂造影检查1 0 3 .腹部平片,对下列状况诊断价值最低的是:( D )A .尿路结石B . 肾结核C.肾萎缩D . 膀胱肿瘤E. 了解节育环位置104.尿路平片不能显示:( C )A .尿路结石B . 腰大肌轮廓C.肾功能D . 肾轮廓E . 腹腔、盆腔异常钙化1 0 5 .双侧肾影增大则可除外:( A )A . 自截肾B . 双侧重复肾C.多囊肾D . 肾囊肿E . 双侧肾盂积水106. 下述尿路结石知识,错误的是:( C )A . 常有肾

419、绞痛B . 常有肾盂积水C.多为阴性结石D . 常有血尿E . 平片检出率高1 0 7 .超声提示肾实质小占位, 进一步检查应首选:( B )A. IVPB. CTC.KUBD . 同位素扫描E . 血管造影1 0 8 .下列关于乳房纤维瘤的描述,错误的是:( E )A . 多为青壮年患者 B . 妊辰期肿瘤增大 C . 绝经后肿瘤缩小D . 常为双侧、多发 E . 分叶状肿块1 0 9 .下列关于乳腺癌的诊断要点,错误的是: (E )A. 40 60岁妇女 B . 局部皮肤增厚、回缩 C . 沙粒样钙化D. CT增强明显强化 E . 乳腺内圆形肿块,边缘规则第七部分骨关节影像诊断单项选择题1

420、. 下列不是成人长骨结构的是:( E )A . 骨膜B . 骨皮质 C . 骨髓腔D . 骨端E . 骨雕2 . 下列不是儿童长骨结构的是:( B )A . 骨箭B . 骨端 C . 干甑端D . 骨干E . 骨雕板3 . 骨质疏松是指:( E )A .骨的有机成分减少B . 骨的无机成分减少 C . 骨的有机成分增加D . 骨的无机成分增加E . 骨的有机成分和无机成分都减少4. 骨质软化是指:( C )A骨的有机成分增加B . 骨的无机成分减少C . 骨的有机成分正常,骨的无机成分减少D . 骨的有机成分减少E . 骨的有机成分和无机成分都减少5 . 鉴别骨质软化与骨质疏松的X 线表现是:

421、 (E )C . 尺骨上端骨折合并槎骨小头脱位E . 槎骨远端3 cm 骨折断端向背侧成角9 . 急性化脓性骨髓炎最早出现的X 线征象为:A . 干箭端骨质破坏A . 骨小梁数目减少 B . 骨小梁间隙增宽D . 骨皮质变薄,骨髓腔增宽E . 骨小梁骨皮质边缘模糊C . 骨小梁变细6 . 关节破坏的早期X 线表现是:A . 关节间隙增宽 B . 关节间隙变窄D . 骨端松质骨破坏 E . 关节而糜烂( B )C . 软骨下骨质破坏7.“ 骨性”与 “ 纤维性”关节强直的X 线鉴别点是:A . 关节间隙变窄 B . 关节面糜烂D . 关节狭窄,关节缘骨破坏E . 骨小梁通过关节间隙( E )C

422、. 关节狭窄,硬化增生8 . 柯 雷 氏 (Colles)骨折是指:A . 楼骨远端2 cm 以内的伸展型骨折 B , 横骨远端3( D )cm 以内的伸展型骨折D . 槎骨远端2 3 cm骨折断端向掌侧成角( C )B . 在骨皮质旁见平行带状骨膜反应C . 弥漫性软组织肿块 D . 局限性骨质疏松E . 死骨片形成1 0 .脊椎结核典型X 线表现为:( D )A . 胸椎多见B . 骨质密度增高C . 椎体楔形改变D . 多累及相邻两个椎体,椎间隙变窄E . 附近受累较多1 1 .下述作为良性骨肿瘤的X 线表现,错误的是:( D )A . 膨胀性骨质破坏 B . 病变边缘有硬化 C . 局

423、部骨皮质断裂D . 病变边缘骨膜反应 E . 病变边界清楚1 2 .下述骨肿瘤的X 线表现中,恶性骨肿瘤的表现为:( C )A .类圆形透光区边缘增生硬化 B . 骨膨胀性破坏C . 骨破坏区边缘模糊 D . 骨破坏区内有残留骨小梁E . 骨破坏区边缘清晰13. 下 述骨巨细胞瘤的描述中,不正确的是:( E )A . 病变骨皮质变薄并向外膨出 B . 可见囊状透光区向一侧生长C . 起源于间充质细胞 D . 多见于股骨、胫骨、梳骨骨端E . 发生在干箭端1 4 .下述骨肉瘤的影像学描述中,错误的是:( C )A . 原发性恶性骨肿瘤中发病率最高 B . 发生在长管状骨干好端C . 溶骨性骨破坏

424、,无肿瘤性成骨性改变 D. 葱皮样骨膜反应E . 肿瘤内骨化和钙化15. 膝关节在X 线平片所见到的下述解剖结构,不正确的是:( E )A . 关节间隙 B . 骨性关节面 C . 籽骨D . 关节内外脂肪层 E . 关节软骨和关节盘16. 骨质破坏的影像学表现,不正确的是:( E )A . 局部密度减低 B . 骨小梁稀疏消失 C . 骨轮廓膨胀D . 斑片状骨缺损 E . 骨小梁增粗,骨质增厚1 7 .骨质增生硬化的影像学表现,不正确的是:( D )A . 骨密度增高 B . 骨骼增大变形C . 骨刺、骨桥、骨骼缘唇样改变 D . 骨小梁变细减少,骨皮质变薄E. MRIT1和 TaWI呈低

425、信号影1 8 .下述不是急性化脓性骨髓炎的影像学表现的项是:( E )A . 软组织肿胀 B . 骨质破坏与骨质疏松并存C . 片状或条状死骨D .层状及花边状骨膜反应 E . 骨包壳形成,骨干不规则增粗19. 化 脓性关节炎最早出现的影像学征象是:( C )A . 关节间隙增宽B . 关节囊增大,密度增高 C . 关节周围软组织炎性肿胀D . 关节软骨下骨质破坏 E . 局部骨质疏松2 0 .关节结核的影像学不常见的表现是:A . 关节面的骨性破坏及关节间隙不对称狭窄B . 骨型关节结核以髅、肘关节多见C . 滑膜型关节结核以膝、踝关节多见D . 骨质破坏与骨膜增生并存伴软组织肿胀( D )

426、A . 常见于四肢长骨干E . 关节积液,关节囊肿胀,冷脓肿形成21. 下列为化脓性关节炎与关节结核所致骨质破坏的鉴别要点是(D )A . 筛孔状骨质破坏 B . 斑片状骨质破坏C.虫蚀样骨质破坏D . 承重关节面及非承重关节面的骨质破坏E.不规则溶骨性破坏2 2 .强直性脊柱炎最早受累的部位是:(C )A . 腕关节 B . 指间关节D .髅关节 E . 脊椎小关节突关节C.舐能关节23.“ 竹节样脊柱”可见于下列哪些疾病:(E )A . 化脓性腰椎炎 B . 退行性骨关节病D . 肥大性脊椎炎、骨桥形成E . 强直性脊柱炎C.类风湿性脊柱炎2 4 .骨肿瘤及肿瘤样病变的影像学首选方法是:(

427、C )A. MRI B. CTC. X 线平片D. ECT E. DSA2 5 .多发性穿凿样骨质破坏多见于:(B )A . 嗜酸性肉芽肿 B . 多发性骨髓瘤D . 骨转移瘤 E . 骨肉瘤C.内生软骨瘤2 6 .恶性骨肿瘤中最常见的是:(B )A . 骨肉瘤 B . 骨转移瘤D . 纤维肉瘤 E . 骨巨细胞瘤C. 骨髓瘤2 7 .局部骨组织被病理组织所取代称为:(C )A . 骨质疏松 B . 骨质软化D .骨质坏死 E . 骨质增生硬化C. 骨质破坏2 8 .骨组织代谢停止称为:(D )A . 骨质疏松 B . 骨质软化D . 骨质坏死 E . 骨质增生硬化C.骨质破坏2 9 .关节软

428、骨及其下方的骨质为病理组织所侵犯、代替称为:(B )A . 关节肿胀 B . 关节破坏D . 关节强直 E . 关节脱位C.关节退行性变3 0 .关节软骨变性坏死,逐渐被纤维组织取代,引起不同程度的关节间隙狭窄称为:(C )A . 关节肿胀 B . 关节破坏D . 关节强直 E . 关节脱位C.关节退行性变31 . 诊断骨折最简便有效而常用的影像学检查方法是:(A )A. X 线平片 B. CTD . 超声 E . 核素显像C.MRI3 2 .椎体骨折最常见的表现为:(D )A . 横形骨折 B . 斜形骨折D .压缩骨折 E . 凹陷骨折C.螺旋形骨折3 3 .关于儿童青枝骨折的说法,错误的

429、是:(E )B . 骨皮质发生皱褶或凹陷或隆起C . 可以看不见骨折线 D . 发生原因是儿童骨柔韧性较大E . 属于完全骨折34 . 关于骨髓骨折的描述,不正确的是: (B )A . 发生在髓板软骨 B . 不累及干怖端 C. X 线上显示随线增宽D. X 线上显示骨怖干萌端对位异常 E. MR有助于骨雕骨折的诊断35 . 关于脊柱结核影像学表现,叙述正确的是: (B )A . 最多见于胸椎 B . 骨破坏可以开始于椎体内或终板C . 椎间隙保持正常 D . 骨质增生硬化明显 E . 椎体周围软组织正常36 . 关节结构MRI的描述,不正确的是: (C )A. X 线上关节软骨不显影,MRI

430、上为光滑线状低信号影B . 韧带MRI上为低信号影C . 膝关节半月板TiWI为低信号,T:W I为高信号D . 关节囊MRI上为线状低信号影E . 关节内滑液TiWI为低信号,T?WI为高信号37 . 下列关节脊柱MRI不正确的是: (C )A . 骨皮质TiWI、T?WI均为低信号B . 椎间盘髓核TzWI为高信号C . 椎间盘在T.WI上能区分出纤维环和髓核D . 前纵韧带和后纵韧带在T,WI和 T.WI上均为低信号E . 脊髓在TWI上信号低于脑脊液38 . 纤维性骨痂在骨折后形成时间是: (A )A. 23 天 B. 57 天 C. 714天D. 45 周 E. 23 月39 . 关

431、于骨折X 线诊断的叙述,不正确的是: (B )A . 需摄正侧位片 B . 摄片时至少包括邻近的2 个关节C . 注意两侧对比 D . 必要时加拍特殊位置E . 摄片时至少包括邻近的1 个关节40 . 关于脊柱骨折不正确的是: (C )A . 椎体压缩呈楔形 B . 可以不见骨折线 C . 上下椎间隙常有狭窄D . 常合并棘间韧带撕裂 E . 横突可发生骨折41 . 关于脊柱骨折正确的是: (C )A. X 线能够显示椎管内碎片和椎管内血肿B. CT可显示脊髓出血利水肿C. CT可显示韧带的撕裂D. CT检查的重点是骨折片对脊髓和神经根的压迫E. X 线可充分显示骨折类型和骨折片移位情况42

432、. 下列关于椎间盘突出的描述,正确的是: (B )A. X 线常表现为椎间隙对称性狭窄B . 椎体上下缘Schmorl结节是髓核向椎体内脱出形成的C. CT上椎间盘的密度低于硬膜囊D . 常合并椎间孔扩大E . 正常椎间盘TiWI、T?WI均为低信号4 3 . 化脓性骨髓炎骨改变出现的时间是:( C )A. 3 天后 B. 1周后D. 1 个月后 E. 2 周后C. 2 周后4 4 .下列关于慢性骨脓肿的说法,不正确的是:( E )A . 大多发生于长骨的干雕端 B.最常见于胫骨下端和楼骨远端C . 骨破坏区周围常有硬化环 D.破坏区内少有死骨E . 多有骨膜增生4 5 .化脓性关节炎影像学表

433、现,错误的是:( C )A . 可出现关节脱位或半脱位 B.软骨破坏可引起关节间隙的狭窄C . 骨破坏多发生在关节的非承重面 D.骨修复开始后出现骨质增生硬化E. MRI可显示滑膜的炎症46. 化脓性关节炎关节面骨质破坏发生在发病后:( A )A. 1 个月 B. 2 个月C.3 个月D. 4 个月 E. 5 个月4 7 .骨气鼓征见于:( A )A . 骨结核 B . 骨髓炎C.痛风D . 类风湿关节炎 E . 骨巨细胞瘤4 8 .下列选项中,不是关节结核的表现:( D )A . 最常累及股关节和膝关节 B.邻近骨骼骨质疏松C . 周围组织形成冷性脓肿 D.关节面的破坏开始于关节的承重面E

434、. 病变愈合后可有关节强直4 9 .关于类风湿关节炎的影像学表现,正确的是:( D )A . 多发生在大关节 B . 骨破坏呈穿凿样C.常累及远端指间关节D . 关节邻近骨骼发生骨质疏松E.常有死骨和窦道5 0 .色素沉着绒毛结节滑膜炎,最常见于:( A )A . 膝关节 B . 踝关节C. 肩关节D . 肘关节 E . 腕关节5 1 .痛风常引起的关节骨质破坏是:( B )A . 部骼关节 B . 第一跖趾关节C.第五跖趾关节D . 远端指间关节 E . 掌指关节52. 下列选项中,不是佝偻病活动期的X 线表现的是:( E )A . 临时钙化带不规则 B . 干箭端增宽, 中心凹陷C. 骨肺

435、出现延迟D . 下肢畸形 E . 临时钙化带增厚5 3 .最常见的原发恶性骨肿瘤是:( A )A . 骨肉瘤 B . 骨纤维肉瘤C.软骨肉瘤D . 脊索瘤E . 骨髓瘤5 4 .关于骨巨细胞瘤,错误的是:( D )A . 常侵犯骨端B . 内可有骨崎形成的分房 C . 骨破坏呈偏侧性D . 肿瘤内常有钙化,有骨膜增生E . 肿瘤一般不穿破软骨5 5 .提示骨巨细胞瘤恶变的征象是:( E )A . 有骨膜增生B . 包壳不完整C . 无骨崎,呈单一的骨质破坏D . 肿瘤膨明显E . 肿瘤边缘出现筛孔样骨质破坏5 6 .最常见的良性骨肿瘤是:( A )A . 骨软骨瘤 B . 软骨瘤C . 骨瘤D

436、 . 成软骨细胞瘤 E . 骨巨细胞瘤5 7 .关于骨软骨瘤的影像学表现,错误的是:( E )A . 好发于股骨下端和胫骨上端B . 多位于干随端C . 瘤体有蒂与骨相连D . 瘤顶部有不规则高密度影为软骨帽的钙化E . 常有骨膜反应5 8 .下列征象提示骨软骨瘤恶变的是:( C )A . 软骨帽内钙化B . 肿瘤较大,压迫邻近骨骼C . 瘤体出现骨皮质或边缘部骨质破坏D . 瘤体有宽基底蒂与骨相连E . 病灶呈分叶状59 . 下列关于骨纤维异常增殖症的X 线表现,不正确的是: (B )A .股骨、胫骨和肋骨多见 B . 不累及面颅骨C . 呈囊状改变的骨纤维异常增殖症边缘有硬化 D . 内可

437、有粗大骨纹E . 可呈单发或多发的虫蚀样骨破坏60 . 下列关于骨肉瘤的X 线表现,不正确的是: (D )A . 混合型骨肉瘤最多见B . 溶骨型骨肉瘤以骨破坏为主,很少有骨质增生C . 成骨型骨肉瘤以肿瘤骨形成为主D. MRI发现细小瘤骨优于X 线E. MRI可以显示血管、神经和肌肉的关系第二篇介入放射学一、名词解释1 . 介入放射学:是以影像诊断为基础,在医学影像诊断设备引导下,利用穿刺针、导管及其他介入器材,对疾病进行治疗或采集组织学、细菌学及生理、生化资料进行诊断的学科。2 . 经导管动脉栓塞术:X 线电视透视下经导管向靶血管内注入或送入栓塞物质,使之闭塞从而达到预期治疗目的的技术。3

438、 . 经皮经腔血管成形术:是采用导管技术扩张或再通发生粥样硬化的动脉或其他原因所致的血管狭窄或闭塞性病变的方法。4 . 药物血管造影:为了提高诊断的准确性,在介入血管造影中,局部使用血管活性药物后再进行血管造影的方法,称为药物血管造影。5 . 肿瘤染色:富血管性肿瘤在动脉造影的实质期,肿瘤被造影剂充填,形成染色,密度常高于其周围组织,称为肿瘤染色。6 . 首过效应:药物第一次通过靶器官时被提取和代谢的现象,也包括一些其他效应。7 . 层流现象:药物的比重通常比血液小,当药物进入血管后并不能很快和血液混合,特别在卧位时,药物常在血柱的上层流动,优先进入向人体腹侧开口的血管或优先分布于靶器官的腹侧

439、部分的现象。二、单项选择题1 . 介入放射学的概念中,下列选项中错误的是: (E )A .介入放射学是以影像诊断为基础,医学影像诊断设备作引导,利用介入器材对疾病进行诊断与治疗的学科B . 介入放射学是通过临床与影像诊断结合进行微创治疗的医学专业C . 介入放射医师是利用影像引导进行微创治疗的专业临床医师D . 介入放射治疗是已经在许多方面取代了开放性手术的现代医学E . 介入放射学是以影像诊断为主要任务的临床辅助学科2 . 第一个系统阐述了介入放射学概念,并形成共识的学者是: (B )A. Margulis B. Wallance C. DotterD. Forssmann E. Seidi

440、nger3 . 介入放射学中最基本的技术“ 经皮穿刺动脉插管术”的发明者是: (E )A. Andreas Gruntzig B. Charles Dotter C. Alexander MargulisD. Werner Forssmann E. SvenIvar Seidinger4 . 堪称血管成形术之父,1964年首先发明同轴导管的是: (B )A. Andreas Gruntzig B. Charles Dotter C. Alexander MargulisD. Werner Forssmann E. Sven-Ivar Seidinger5. 1977年首先使用双腔球囊导管进行冠

441、状动脉成形术的是: (A )A. Andreas Gruntzig B. Charles Dotter C. Alexander MargulisD. Werner Forssmann E. SvenIvar Seidinger6 . 在介入血管造影中,常使用血管活性药物提高造影诊断的准确性。下面药物中,不是血管扩张剂的是:(C )A . 罂粟碱 B . 前列腺素 C . 肾上腺素D . 妥拉苏林 E . 缓激肽7 . 介入治疗中常用的抗凝药物是: (D )A . 低分子肝素 B . 鱼精蛋白 C . 阿司匹林D . 肝素钠 E . 华法林钠8 . 下面的药物中不是溶栓药物的是: (A )A

442、. 双香豆素 B . 尿激酶 C . 组织纤溶酶原激活因子D . 链激酶E . 冬凌克栓酶9. 肿瘤介入常用的抗肿瘤药中,细胞周期特异性药物是: (D )A . 顺钳 B . 表阿霉素 C . 环磷酰胺D. 5-FU E . 丝裂霉素10 . 介入放射学按照介入治疗的方法进行分类,应分为: (B )A . 穿刺术 B . 肿瘤介入 C . 灌注术D . 栓塞术 E . 成形术11 . 根据治疗领域分类,下列手术中不属于血管系统介入放射学的是: (D )A . 颅内动脉瘤电解式可脱弹簧圈栓塞术 B . 原发性肝癌化疗栓塞术C . 下腔静脉滤器置入术 D . 原发性肝癌射频消融术 E. TIPSS

443、术12 . 下列关于经导管动脉栓塞术治疗的机制中,不正确的是: (B )A . 阻塞靶血管,使肿瘤或靶器官造成缺血、坏死B . 提高病变局部的药物浓度,可以增强治疗效果,减少药物对全身的不良反应C . 阻塞或破坏异常血管床、腔隙或通道使血流动力学恢复正常D . 阻塞血管使之压力下降利于止血E . 血管内封堵破裂的血管利于止血13 . 下列选项中,不符合介入术放射学中使用的栓塞物质的说法是: (D )A . 能顺利通过导管注入 B . 无毒或低毒C . 人体组织相容性好,不产生排斥反应 D . 能够短时间内被吸收或代谢E . 无致畸和致癌14 . 根据药代动力学原理,药物经过静脉注入体内后的分布

444、为:( C )A. 1 期B. 2 期C. 3 期D. 4 期E. 5 期1 5 .药物经动脉注入后,药物分布【 相 ( 吸收期)中,关于靶器官内药物分布量叙述中正确的是:(C )A . 受到药物脂溶性和蛋白结合性的影响 B . 受到血压和心率的影响C . 不受血流分布的影响 D . 受到血流分布的影响E . 是全身药物分布量最大的器官16 . 药物经动脉注入血管后,下列叙述中正确的是: (B )A . 能够很快和血液混合均匀 B . 药物能在血柱的上层流动C . 药物能在血柱的下层流动 D . 药物能在血柱的中层流动E . 患者卧位时,药物优先进入向背侧开口的血管分支17 . 下列选项不是球

445、囊血管成形术的治疗机制的是: (D )A . 血管壁内一中膜局限性撕裂 B . 中膜组织的过度伸展C . 动脉粥样硬化斑撕裂 D . 血管外膜部分性损伤E . 控制性损伤学说是球囊血管成形术的治疗机制18 . 血管支架扩张血管的机制中,下列叙述正确的是: (C )A . 目前血管支架是使狭窄血管管腔扩张的主要手段B . 血管支架置入后,容易刺激动脉粥样斑块形成C . 血管支架置入后,分支血管口不发生阻塞D . 血管支架置入后,狭窄血管不会发生再狭窄E . 血管支架成形术后血管再狭窄的几率比球囊扩张血管成形术后再狭窄几率高19. 关于肾血管性高血压的形成机制,下列叙述错误的是:A . 肾动脉狭窄

446、造成肾灌注压降低,刺激肾素分泌量增加B . 肾素转化为血管紧张素1c . 血管紧张素I 在水解酶的作用下转化为血管紧张素n( D)D . 血管紧张素I 刺激肾上腺皮质分泌醛固酮,造成水钠潴留,使血压进一步升高E.血管紧张素n 刺激肾上腺皮质分泌醛固酮,造成水钠潴留,使血压进一步升高20.下面影像设备在血管系统介入放射学最常用的是:A . 彩色多普勒 B . 多排螺旋CT机D . 开放型MR机 E . 电视透视C. DSA 机( C)21.穿刺活检术常用的导引设备是:A. B 超 B. CT 机D . 开放型MR机 E . 电视透视机C. DSA 机( A)22.介入手术中,通常描述导丝直径的单

447、位是:A . 厘米 B . 毫米D. French E . 英寸C. Gauge( E)23.介入手术中,通常描述穿刺针直径的单位是:A . 厘米 B . 毫米D. French E . 英寸C. Gauge( C)24.介入手术中,通常描述导管直径的单位是:A . 厘米 B . 毫米D. French E . 英寸C. Gauge( D)25.下列选项不是介入治疗前术前的常规准备项目的是:( D)A . 术前禁食4 h B . 血常规和凝血功能检查C . 肝功能、生化、电解质检查 D. CT或 MR检查E . 术前谈话,患者或家属在治疗同意书上签字26. 股动脉穿刺插管术后,通常穿刺点压迫多

448、长时间后局部加压包扎:( D)A. 3 5 min B. 5 8 minC. 6 10 minD. 10 15 min E. 15 20 min27. 股动脉穿刺插管术后的处理中,下列选项中不正确的是:( C)A . 术后观察穿刺点有无出血B . 穿刺侧下肢制动8 hC . 穿刺侧下肢制动24 hD . 观察下肢皮肤温度与足背动脉搏动情况E. 24 h 后患者可下床活动28. 经皮经肝胆道引流术的适应证中,下列选项中正确的是:A . 肝内胆管扩张10 m m ,血清胆红素120 m ol/LB . 肝内胆管扩张23 m m ,血清胆红素170/mol / LC . 血清胆红素升高以间接胆红素升

449、高尤为严重时D . 大量腹水( B)E . 全身极度衰弱2 9 .下列情况下不是肾盂造瘦术的适应证的是:(C )A . 解除尿路梗阻所致的肾盂肾盏扩张 B . 输尿管屡做上段分流C . 结核性肾积水 D . 肾盂输尿管的刷片活检E . 诊断尿路梗阻的原因和部位3 0 .下列选项中,不是富血管肿瘤动脉造影的典型表现的是:(E )A . 肿瘤供血动脉增粗B . 肿瘤内可见到排列紊乱、增生的新生血管C . 肿瘤内可见异常引流静脉,动静脉瘦D . 肿瘤染色E . 邻近血管被推移包绕3 1 .下列选项中,不是部分脾动脉栓塞术适应证的是:(D )A . 脾功能亢进 B . 地中海贫血C . 特发性血小板减

450、少性紫瘢D . 门静脉高压引起的上消化道大出血 E . 脾破裂3 2 .部分脾动脉栓塞术如果过量栓塞,最容易引发的并发症是:(E )A . 腹腔出血 B . 白细胞下降D . 黄疸 E . 脾脓肿3 3 .下列栓塞剂中,最容易造成靶器官组织缺血坏死的是:C . 血小板升高(D )A . 明胶海绵 B . 弹簧圈D . 无水乙醇 E . 超液化碘油3 4 .下列栓塞剂中,哪一种不是永久栓塞剂:C . 可脱球囊(A )A . 明胶海绵 B . 弹簧圈D . 无水乙醇 E . 碘化油化疗药物乳剂3 5 .下列哪项不是液体栓塞剂:C . 可脱球囊(D )A . 碘化油化疗药物乳剂 B . 无水乙醇D

451、. 泡沫聚乙烯醇 E . 组织胶3 6 .介入性栓塞术可用于以下情况,不适宜的是:C . 鱼肝油酸钠(E )A . 产后大出血 B . 肾挫伤D . 蛛网膜下隙出血 E . 脑实质内出血3 7 .恶性肿瘤适应做动脉栓塞治疗的,不是适应证的是:C . 胃冠状静脉出血(B )A . 原发性肝细胞性肝癌 B . 原发性胆管细胞性肝癌D . 肾上腺肿瘤 E . 盆腔恶性富血管性肿瘤3 8 .下列恶性肿瘤中不能进行栓塞治疗的是:C . 肾脏肿瘤(E )A . 原发性肝细胞肝癌 B . 肾上腺癌C . 盆腔富血管性恶性肿瘤D . 颌面部恶性肿瘤 E . 大脑星性胶质瘤3 9 .良性肿瘤适应做动脉栓塞治疗,

452、以下选项中不属于适应证的是:(D )A .脑膜瘤 B . 鼻咽纤维血管瘤D . 小肠平滑肌瘤 E . 症状性子宫肌瘤4 0 .关于栓塞后并发症的叙述中,不正确的是:C . 颈动脉球瘤(C )A . 栓塞后并发症是术后不希望发生的症状或体征B . 误栓的后果与被误栓的器官的重要性和误栓的程度有关C . 栓塞后出现的发热是正常现象,不属栓塞后并发症D . 过度栓塞是引起栓塞后并发症的原因之E . 误检分为反流性误栓和顺流性误栓41 . 动脉药物灌注术中,选择化疗药物时,下列叙述不正确的是: (D )A . 细胞周期特异性药物对癌细胞的杀伤作用较弱且缓慢B . 细胞周期非特异性药物对癌细胞的杀伤作用

453、较强烈C . 细胞周期非特异性药物为剂量依赖型,适宜一次性冲击性动脉灌注化疗D . 细胞周期特异性药物为剂量依赖型,适宜一次性冲击性动脉灌注化疗E . 动脉灌注化疗术中,应该联合应用不同种类的化疗药物,以提高疗效42 . 局部灌注溶拴药物治疗血栓形成性疾病时,需要监测凝血酶原时间,当凝血酶原时间延长至正常的几倍时,应该停止减慢溶栓剂的注入速度或停止溶栓: (C )A. 0.5 倍 B. 1 倍 C. 2 倍D. 4 倍 E. 8 倍43 . 下列溶栓药物中,溶栓药物对血栓作用的选择性最好,溶栓并发症少的是: (C )A . 蝮蛇抗栓酶 B . 尿激酶 C . 组织纤溶酶原激活因子D . 链激酶

454、 E . 冬凌克栓酶44 . 下列溶栓药物中,目前临床最为常用的是: (B )A . 蝮蛇抗栓酶 B . 尿激酶 C . 组织纤溶酶原激活因子D . 链激酶 E . 冬凌克栓酶45 . 下列溶栓药物中,溶栓药物过敏反应发生率高,临床现已很少使用的是: (D )A . 蝮蛇抗栓酶 B . 尿激酶 C . 组织纤溶酶原激活因子D . 链激酶 E . 冬凌克栓酶46 . 下列选项中,不是经皮经腔血管成形术适应证的一项是: (D )A . 动脉粥样硬化引起的有血流动力学意义的血管狭窄B . 血管搭桥术后所致的吻合口狭窄及移植血管狭窄C . 肾动脉狭窄所致的继发性高血压D . 动脉狭窄段合并有溃疡或钙化

455、者E . 血管肌性发育不良所致的局限性狭窄47 . 下列选项中,不是经皮经腔血管成形术常见并发症的一项是: (C )A . 穿刺部位血肿 B . 动脉瘤形成 C . 空气栓塞D . 血栓形成 E . 血管壁破裂48 . 布加综合征的概念,下列选项中叙述不正确的是: (C )A . 下腔静脉肝段发生的阻塞B . 可以合并有肝静脉或者不合并有肝静脉的阻塞C . 通常合并肾静脉狭窄,肾功能受到影响D . 下腔静脉阻塞可以有膜形或节段形E . 肝静脉狭窄时,可以出现门静脉高压症状4 9 .布加综合征通常分为:C)A. 2型D. 5型B. 3型E. 6型C. 4型50.布加综合征的临床表现中,不正确的一

456、项是:(B)A .以青壮年多见,男性多于女性B .以中老年多见,女性多于男性C .腹痛、腹胀、黄疸、腹水、肝脾肿大等口静脉高压的症状和体征51.52.53.54.D .双下肢肿胀E .活动后常感心悸、气短肾动脉球囊扩张成形术的适应证中,下列叙述正确的一项是:A .理想的适应证是单侧肾动脉长段、单发、B .理想的适应证是单侧肾动脉短段、单发、C .理想的适应证是单侧肾动脉短段、多发、D .理想的适应证是单侧肾动脉短段、多发、E .大动脉炎症活动期无钙化的狭窄,无钙化的狭窄,无钙化的狭窄,无钙化的狭窄,肾动脉支架扩张成形术的适应证中,下列叙述正确的是:A .理想的适应证是单侧肾动脉长段、B .理想

457、的适应证是单侧肾动脉短段、C .理想的适应证是单侧肾动脉短段、D .大动脉炎活动期E .年龄较小的少儿患者单发、单发、多发、无钙化的狭窄,无钙化的狭窄,无钙化的狭窄,下列途径中是介人手术中最常用的穿刺途径的是:A .税动脉D .颈总动脉B .肱动脉E .锁骨下动脉下列器材中,不在血管系统介入手术中使用的是:A .穿刺针D .内涵管B .导丝E .金属支架狭窄程度大于50%狭窄程度大于70%狭窄程度大于70%狭窄程度大于50%狭窄程度大于50%狭窄程度大于70%狭窄程度大于70%C.C.股动脉导管鞘(BBCD)55.在介入灌注与栓塞术中,下列介入器材中的常规器材是:D)56.A .同轴导管系统D

458、 .微导管B .灌注导丝E .球囊阻塞导管C.PCS下列造影剂中,离子型造影剂是:C)A .优维显D. CO2B .欧乃派克E .碘化油C .泛影葡族57.碘剂过敏的患者需要行血管造影,可以替代以:D)A .优维显D. CO,B .欧乃派克E .碘化油C .泛影葡胺58 .选择性腹腔干动脉造影中, 卜列动脉中不会显影的是:D)A .胃左动脉D .肠系膜上动脉B .肝总动脉E .胃右动脉C .脾动脉59 .血管内药物灌注术理论依据及适应证的描述中,不正确的是:D)A .血管内药物灌注术注重药物的局部效应,使药物直接作用于病变的局部B . 提高病变局部的药物浓度,可以增强治疗效果,减少药物对全身的

459、不良反应C . 治疗肝、脾、肾、消化道及盆腔脏器的出血D . 尿激酶灌注溶栓的风险比全身静脉溶栓的风险明显降低E . 转移性肿瘤的治疗6 0 .原发性肝癌,下列情况下适宜进行肝动脉栓塞治疗的是: (D )A . 大量腹水B . 门静脉主干有癌栓,没有明显的门静脉侧支循环形成C . 血清胆红素大于50 /mol / LD . 肝脏肿瘤体积巨大,占据肝脏体积的4 0 % ,肝功能Child B 级E . 弥漫性少血供型肝癌61 . 原发性肝癌进行肝动脉栓塞治疗时常用的栓塞剂是: (A )A . 碘化油化疗药物乳剂 B. PVA C . 无水乙醇D . 明胶海绵 E . 金属弹簧圈62 . 外伤性肾

460、挫伤做肾动脉栓塞止血,通常选用的栓塞物质是: (D )A . 碘化油化疗药物乳剂 B. PVA C . 无水乙醇D . 明胶海绵 E . 金属弹簧圈63 . 外伤性肾挫伤合并假性动脉瘤形成,通常选用的栓塞物质是: (E )A . 碘化油化疗药物乳剂 B. PVA C . 无水乙醇D . 明胶海绵 E . 金属弹簧圈64 . 症状性子宫肌瘤介入治疗时通常采用的栓塞剂是: (B )A . 碘化油化疗药物乳剂 B. PVA C . 无水乙醇D . 明胶海绵 E . 金属弹簧圈65 . 血管内金属支架的材料与类型描述,哪项不正确: (A )A . 根据制作材料分为不锈钢支架、银钛合金支架和内涵管B .

461、 根据展开的方式分为自膨式支架和球扩式C . 根据表面处理情况分为裸露型、涂层型和覆膜型D . 按照功能分为单纯支撑型和治疗型E . 根据制作工艺分为编织型和激光切割型66 . 下列选项不是血管内金属支架成形术的适应证的是: (E )A . 器动脉、股动脉、冠状动脉、肾动脉、肺动脉、腔静脉、主动脉狭窄性疾病B . 偏心性狭窄等不适宜做单纯球囊扩张成形术者C . 经皮腔内球囊扩张成形术后再狭窄或者急性闭塞以及发生动脉痉挛者D . 经皮腔内球囊扩张成形术后,为预防再狭窄或急性闭塞时E . 广泛的末梢血管狭窄67 . 下面对球囊血管成形术后再狭窄的描述,不正确的是: (C )A. PTA后再狭窄分为

462、急性血管闭塞、早期再狭窄、晚期再狭窄B . 早期再狭窄是PTA后 1 年内发生的再狭窄C . 早期再狭窄是PTA后 1 个月内发生的再狭窄D . 急性血管闭塞是指操作成功,但 PTA后立刻或不久发生血管闭塞E. PTA后再狭窄总平均发生率约为30%68 . 下列关于血管成形术后再狭窄防治措施的描述,不正确的是: (E )A. PTA后再狭窄确切机制尚不清楚B . 抗血栓形成、抗平滑肌细胞增生和放射治疗是目前的三大措施C . 防止血栓形成是防治再狭窄的重要环节D . 抗平滑肌细胞增生的基因治疗目前还没有进入临床E . 在防治PTA后血管再狭窄时,体外照射治疗比体内照射效果好69 . 球囊血管成形

463、术后再狭窄机制中,不是导致血管急性闭塞的原因的是: (A )A . 内- 中膜撕裂 B . 内膜剥离 C . 血管痉挛D . 血栓形成 E . 血管破裂70 . 下列哪种不是理想静脉滤器的特征: (E )A . 能够阻止较大的血栓块通过 B . 不影响正常的血流C . 置放容易 D . 置人后滤器稳定,不移位E . 滤器网孔越密越好71 . 功能性胰岛细胞腺瘤需要采血检查,采血导管的位置叙述不正确的是: (D )A . 门静脉主干远端 B . 门静脉主干近端 C . 肠系膜上静脉D .肠系膜下静脉 E . 脾静脉72 . 以下选项中,不是PTCD手术的禁忌证的是: (E )A . 有明显的出血

464、征象 B . 大量腹水 C . 肝功能衰竭D . 碘剂过敏 E . 胰腺十二指肠区恶性肿瘤侵犯、压迫了胆总管73 . 有关Seidinger穿刺法的叙述,不正确的是: (D )A . 穿刺前局部麻醉时,要先做皮内麻醉B . 尖刀片挑开皮肤2 m m ,穿刺针斜面朝向上C. 3 0 4 0 快速穿透血管前后壁,退出针心,缓慢退针至见血液从针尾射出D. 3 0 4 0 缓慢穿透血管前壁,见血液从针尾射出E . 引入导丝,退针置鞘74 . 经导管药物灌注化疗有以下儿种方法,能够兼顾提高靶器官局部药物浓度并能延长药物作用时间,减少正常组织药物接受量的方法是: (B )A. 一次冲击性动脉内药物灌注术

465、B . 动脉阻滞化疗C . 导管留置长期药物灌注 D . 经皮导管药盒系统植入术E . 降主动脉内药物灌注75 . 关于输卵管成形术,下列叙述不正确的一项是: (D )A . 输卵管再通适用于输卵管阻塞者B . 壶腹部远端、伞端组织不宜进行再通术C . 子宫角部严重闭塞者、结核性输卵管炎不宜进行再通术D . 金属支架成形术可用于输卵管局限性狭窄的治疗E . 输卵管再通后要继续给予药物性通水,维持输卵管通畅7 6 .下列消化道哪个部位还不能进行管腔成形术的介入治疗: (D )A . 食管 B . 胃 C . 十二指肠D . 空肠E . 直肠7 7 .食管狭窄金属支架成形术不适宜:( B )A .

466、 食管癌晚期B . 年龄偏小的儿童食管化学性烧伤C . 食管气管髅的病人D . 食管肿瘤术后复发病人E . 贲门癌7 8 .下列选项不属于介入治疗非血管管腔成形术范围:( E)A .胆道成形术B . 鼻泪管成形术C . 泌尿道成形术D . 消化道成形术E . 椎管成形术79. TIPSS手术主要用于:( C)A . 原发性肝癌B . 阻塞性黄疸C . 肝硬化、门静脉高压D . 下消化道出血E . 脾功能亢进80. 下列操作步骤不是TIPSS手术中的必须步骤:A . 经股静脉穿刺B . 经颈内静脉穿刺C . 肝静脉造影后经肝静脉向门静脉穿刺D. 门静脉造影后,引入球囊导管扩张分流道( A )E

467、. 分流道扩张后,留置支架,支架置放成功后,再次做门静脉造影第三篇影像技术一、名词解释1 . 射线对比度:射线本身是一束无信息的能源,当它透过人体时,射线被部分吸收和散射,高吸收区域透过的射线与低吸收区域透过的射线形成强度分布的差别,这种透过人体组织后形成的射线强度分布上的差异称为射线对比度。2 . 放大率:放大的影像比实际肢体增大的倍数叫放大率或称放大倍数。3 . 第一斜位:被检者身体右侧朝前倾斜贴暗盒面或立位摄影架面板,或者是摄影床的床面。左侧远离暗盒或床面,冠状面与暗盒面或床面倾斜一定角度。4 . 宽容度:是指连接特性曲线上指定两点密度所对应的曝光量范围。5 . 听眶线:外耳孔与眼眶下缘

468、的连线,此线为解剖学上的颅骨基底线,或水平线。6 . 透光率:透过照片的光强度与入射光强度之比。7 . 增感率:在照片上取得相同的密度值1 .0 时,无屏与有屏所需要的曝光量之比值。8 .平均斜率:连接胶片特性曲线上指定两点密度Di和 D?的直线与横坐标夹角的正切值。9 . 栅比:是铅条高度与铅条间距之比。10 . 定影:就是将未感光的卤化银溶解掉的过程。11 . 时间减影:用作减影的两图像是在不同显影时期获得的。12 . 球管热容量:X 线管处于最大冷却率时,允许承受的最大热量。13 . 均匀度:主磁场的均匀性系指B。 随空间位置的改变而发生的大小变化。14 . 空间分辨率:是指图像中可辨认

469、的邻接物体空间几何长度的最小极限,即对细微结构的分辩率。15 . CT值:CT影像中每个像素所对应的物质对X 线线性平均衰减量大小的指标。16 . 时间飞跃效应:是指流动的自旋流进静态组织区域而产生比静态组织高的MR信号。17 . 进动:原子自旋轴与主磁场的轴线有一小角度不完全平行,并围绕主磁场轴作较慢的旋转。18 . 纵向弛豫:通常将M z的恢复称为纵向弛豫,是自旋一晶格弛豫的反映,因此又称其为Ti弛豫。19 . 螺 距 :定义为扫描时床进速度与扫描层厚之比值。20 . 像素:又称像元,指组成图像矩阵中的基本单元。二、单项选择题1. 组成原子的是: (AA . 原于核和核外电子 B . 原子

470、和中子 C . 中子和核外电子D . 原子核和质子 E . 质子和核外电子)2.高速电子与阳极靶物质发生相互作用时: ( CA . 产生连续X 线 B . 只产生特性X 线C . 产生连续X 线和特性X 线 D . 不产生X 线 E . 产生电子线)3. X 线摄影中表示X 线量的是: (DA . 半值层 B . 靶物质 C. kVD. mAs E . 电压波形)4. X 线管中,当高速电子与阳极靶相互作用时,绝大部分高速电子的能量转变为: (BA. X 线的能量 B . 热能 C . 连续X 线的能量D .特性X 线的能量 E . 电子对的能量)5.导致X 线衰减的原因是: (AA . 物质

471、和距离 B. X 线不可见 C. X 线波长短D. X 线能量大 E. X 线是电磁渡)6. X 线在医学上利用的原理中不包括: (DA . 利用其穿透性进行X 线检查 B . 利用其荧光作用进行透视检查C . 利用其摄影作用进行照片检查 D . 利用其电离作用进行CT扫描E . 利用其生物效用进行肿瘤治疗)7.常用X 射线管的阴极类型有: (BA . 平面焦点型 B . 圆焦点型 C . 三角焦点型D . 螺旋管型 E . 有 B 或 C 项所述的两种类型)8. X 射线的量和质,其含意正确的是: (BA. X 射线的量和质之和表示X 射线的强度B . 量就是X 射线光子的数量,质则是X 射

472、线光子的能量C. X 射线的量和质之积表示X 射线的强度D. X 射线的量和质成反比关系E. X 射线的量和质成正比关系)9.作为X 线照片影像的可供诊断的密度范围是: (CA. 0. 51. 0 B. 0. 52. 0 C. 0. 252.0)D. 0. 252. 5 E. 0. 5 -2. 51 0 .对影像半影模糊的控制阈值为:( B)A. 0. 02 mm B. 0. 20 mmC. 0. 25 mmD. 0 5 mm E. 0 6 mm1 1 .与 X 线量无关的因素是:( E)A . 管电流 B . 管电压C. 给予X 线管的电能D . 靶物质的原子序数 E. X 线管阳极、阴极间

473、的距离1 2 .对 X 线产生效率的说法,正确的是:( B)A . 很高 B . 很低C. 一般D. 50% E. 100%1 3 .物质对X 线的总衰减系数包括:( E)A . 光电衰减系数 B . 相干散射衰减系数C.,康普顿衰减系数D . 电子对效应衰减系数 E . 以上都是1 4 .人体各组织对X 线的衰减,由大变小的顺序是:( B)A . 骨、脂肪、肌肉、空气 B . 骨、肌肉、脂肪、空气C . 脂肪、骨、肌肉、空气D . 肌肉、骨、脂肪、空气 E . 肌肉、脂肪、骨、空气1 5 .有效焦点与实际焦点的关系,正确的描述是:( D)A . 实际焦点一定等于有效焦点 B . 实际焦点的长

474、度越长,有效焦点也就越大C . 靶角越小,有效焦点面积也就越小 D . 实际焦点一定大于或等于有效焦点E . 实际焦点与有效焦点无定量关系1 6 .关于X 线照片影像的形成要素,不包括:( B)A . 照片密度 B . 照片的感度C.照片的对比度D . 照片的锐利度 E . 照片的放大与变形1 7 .观察小儿发育情况,需摄取:( D)A . 腕关节正位 B . 腕关节侧位C.双腕关节斜位D . 双腕关节正位 E . 双腕关节侧位1 8 .肩关节正位摄影,中心线正确射入点为:( D)A . 锁骨的中点 B . 关节盂C. 肩峰D .肩胛骨喙突 E . 肱骨头1 9 .锁骨正位片:锁骨显示为“ s

475、”形弯曲为:( A)A . 肩关节旋转所致 B . 肩关节抬高所致C. 肩关节外展所致D . 肩关节内收所致 E . 肩关节倾斜所致2 0 .膝关节侧位摄影,关节需屈曲多少度:( D)A. 105 B. 115C.125D. 135 E. 1452 1 .类风湿性关节炎,正确的摄影体位是:( E)A . 双手正位 B . 单侧腕关节正位C.双侧腕关节正位D . 单手正位,包括腕关节 E . 双手正位,包括腕关节2 2 .有关X 线束、照射野的叙述,错误的是;( C)A. X 线球管发射锥形X 线束 B. X 线束入射被照体曝光面的大小称照射野C . 摄影时照射野应尽量太 D . 摄影中的X 线

476、束有一定的穿透能力E. X 线束中心部分的X 线为中心线23 . 放大摄影X 线管焦点为0 .0 5 ,允许的最大放大率为 (C )A. 3 倍 B. 4 倍 C. 5 倍D. 6 倍 E. 7 倍24 . 关于滤线栅使用注意事项的叙述,错误的是: (A )A . 将滤线栅置于焦点和被照体之间 B . 焦点到滤线栅的距离与栅焦距相等C. X 线中心线对准滤线栅的中心 D . 原射线投射方向与滤线栅铅条排列间隙平行E . 原发X 线与滤线栅铅条平行2 5 .机洗的干燥温度一般设定在:(D )A. 18 2(TC B. 22 30D. 45 55 E. 60 65C. 34402 6 .放大摄影的

477、定位方法,错误的是:A . 解剖学定位法 B . 透视定位法D . 生理定位法 E . 透视、照片综合法C. X 线平片定位法( D )2 7 .幼儿胸部正位摄影,中心线经:A . 第 5 胸椎 B . 第 6 胸椎D . 胸骨角水平 E . 腋中线前5 cm 水平、C . 第 7 胸椎( D )2 8 .摄影中减小运动模糊的叙述,错误的是:A . 被照体固定 B . 选择适当的呼吸方法D . 短焦片距 E . 选用高速增感屏C . 缩短曝光时间( D )29. X 线透过被照体后形成的X 线强度的差异,称为;A . 人工对比度 B . 天然对比度D . 胶片对比度 E . 照片对比度C. X

478、 线射线对比度( C )3 0 .病人仰卧,影像增强器转至病人左前方的摄影方向称:A . 头足位 B . 足头位D .左前斜位 E . 右前斜位C . 复合位( D )3 1 .平静呼吸下屏气,常用检查:A .手 B . 下肢D . 心脏 E . 躯干C . 前臂( .D )3 2 .人体水平面又称为:A . 横切面 B . 矢状面D . 冠状面 E . 正中矢状面C . 额状面( A )33. X 线自被照体的前面投向后面称为:A . 前后方向投射 B . 后前方向投射D . 轴方向投射 E . 切线方向投射C . 冠状方向投射( A )34. X 线呈矢状方向从患者的后方垂直射人胶片时的体

479、位是:A . 侧位 B . 前后位C . 后前位( D )D . 颌顶位 E . 顶颌位35. 通常照片铅字标记应置于暗盒边缘内: (C )A. 0. 5 cm 处 B. 1. 0 cm 处 C. 1. 5 cm 处D. 3 cm 处 E. 5 cm 处36.外耳孔与同侧眼眶下缘的连线称为: (B )A . 听毗线 B . 听眶线 C . 昕鼻线D . 昕口线 E . 瞳间线37.脐上3 cm 与哪个腰椎同一平面: (D )A . 第 12胸椎 B . 第 1腰椎 C . 第 2 腰椎D . 第 3 腰椎 E . 第 4 腰椎38.关于胶片的叙述,错误的是: (A )A . 晶体颗粒大,感光度

480、高 B . 晶体颗粒分布均匀,对比度高C . 晶体颗粒大小不一,宽容度高 D . 晶体颗粒小,分辨率低E . 晶体颗粒小,涂层薄,清晰度好39. X 线胶片特性曲线的直线部是指: (B )A . 密度与照射量的变化不成比例的部分 B . 密度与照射量的变化成比例的部分C . 不是摄影中力求应用的部分 D . 密度与照射量没联系的部分E . 称为反转部40. 胶片特性曲线不能直接反映的是: (C )A . 反差系数 B . 宽容度 C . 感色性D . 分辨率 E . 本底灰雾41.不属于一般摄影用X 线胶片的是: (C )A . 感蓝胶片 B . 感绿胶片 C . 荧光缩影胶片D . 乳腺摄影

481、用正色胶片 E . 高清晰度摄影用胶片42.关于阴性对比剂的叙述,错误的是: (B )A . 空气是阴性对比剂 B . 空气在器官内吸收较快 C . 空气易产生气体栓塞D . 二氧化碳的溶度较大 E . 二氧化碳不易产生气体栓塞43.胸部摄影,FFD选用180 cm 的原因是避免因: (E )A . 左右径较窄、前后径较薄引起的影像放大B . 左右径较厚、前后径较宽引起的影像放大C . 左右径较短、前后径较长引起的影像放大D . 左右径较扁、前后径较窄引起的影像放大E . 左右径较宽、前后径较厚引起的影像放大44.膈上肋骨摄影,采用的呼吸方式为: (C )A . 浅呼吸屏气 B . 深呼气屏气

482、 C . 深吸气屏气D . 平静呼吸屏气 E . 腹式呼吸屏气45.心脏右前斜位,摄影的角度是: (A )A. 45 55 B. 56 65 C. 66 70D. 71 80 E. 81 854 6 .上颌窦摄影,常规首选位置是:( B )A . 柯氏位 B . 瓦氏位D . 劳氏位 E . 瑞氏位C . 斯氏位4 7 .正确选择乳突梅氏位的摄影角度,应是:A . 双 25 B . 双 35D . 双 55 E . 双 65C . 双 45( C )4 8 .外伤性颅底骨折,禁止使用的摄影体位是:A .卢页底侧位 B . 内页底颌顶位D . 头颅半轴位 E . 高颈椎颅底侧位C . 头颅汤氏位

483、( B )4 9 .腰椎椎弓峡部断裂,正确的摄影体位是:A . 腰椎正位 B . 腰椎侧位D . 胸腰段斜位 E . 胸腰段侧位C . 腰椎双斜位( C )5 0 .细小玻璃碎片进入眼内,该异物属于:A . 不透性异物 B . 半透性异物D . 金属性异物 E . 磁性异物C . 透过性异物( B )5 1 .有关乳腺摄影的叙述,错误的是:A . 需要加压 B . 使用单乳剂腔片D . 常规摄影取轴位和侧斜位E . 依发育期确定曝光条件C . 使用高速增感屏( C )5 2 .肱骨穿胸侧位,适用的病变是:A . 肱骨骨折 B . 肱骨骨疣D . 胧骨成骨内瘤 E . 肱骨外科颈骨折C . 肱骨

484、骨髓炎( E )5 3 .下列疾患,不能由腹部平片诊断的是:A . 胆囊阳性结石 B . 肠梗阻D . 消化道穿孔 E . 小儿先天性肛门闭锁C . 慢性胰腺炎( C )5 4 .减少和排除散射线的方法中,错误的是:( B )A . 缩小照射野减少散射线 B . 用金属后背盖的暗盒减少散射线C . 用遮线筒或缩光器来减少散射线 D . 选择高电压摄影的方法E . 使用滤线栅法排除散射线5 5 .造影前不需做肠道清洁准备的是;( E )A . 静脉肾盂造影 B . 膀胱造影 C . 肝动脉造影D . 输卵管造影 E . 四肢静脉造影5 6 .能较好地反映胆囊浓缩功能的造影方法是:( B )A .

485、 静脉法胆系造影 B. 口服法胆系造影 C . 术中胆管造影D . 经皮肝穿刺胆管造影5 7 .子宫输卵管造影的禁忌证是:( E )A . 子宫输卵管慢性炎症 B . 子宫输卵管结核C . 子宫输卵管良性肿瘤 D . 子宫输卵管位置、形态异常E . 子官输卵管出血58. 眼球异物定位检查的主要目的是:( C )A .确定有无异物 B .确定异物大小D .确定异物性质 E .确定手术方案C.确定异物位置59. 增感屏的保养、使用中,错误的是:(D)A .防高温、防潮湿 B .防阳光曝晒C.防止水或药液溅人D .发现灰尘即用口吹清除 E .暗盒应直立放置, 避免重压变形6 0 .不能显示跟骨影像的

486、摄影体位是:(A)A .足正位 B .足侧位D .跟骨侧位 E .跟骨轴位像C.全足正位61. CT与X线照片比较,X线照片不易显示的病变是;(E)A .骨折 B .异物D .骨肿瘤 E .软组织内出血C.骨结核6 2 .不适宜进行DSA检查的是:(E)A .出血性病变 B .血管性疾病的介入治疗C.冠状动脉病变D .良、恶性肿瘤的鉴别诊断E .严重的心、肝、肾疾病63. DSA常用的减影方法是:(A)A .时间减影法 B .能量减影法D .电视减影法 E .以上都不是C.混合减影法6 4 .关于前臂侧位摄影,错误的叙述是:(B)A .尺侧靠近暗盒 B .槎侧靠近暗盒D .肩部尽量放低 E .

487、掌面垂直暗盒C.肘部屈曲约906 5 .肩关节前后位摄影,中心线应对准:(B)A .肱骨头 B .喙突D .喙突下5 cm E .肩锁关节C.喙突下2 cm6 6 .足前后位摄影,中心线应对准:(B)A .第三跖骨头 B .第三跖骨基底部D .内、外踝连线中点 E .距骨中点C.第三跖趾关节6 7 .检查小儿腕关节脱位复位情况的体位是:(D)A .魏关节前后位 B .龌关节侧位D .髅关节蛙形位 E .髓关节后前斜位C.髅关节侧斜位6 8 .第3 7颈椎前后位摄影时,中心线应:(B)A .向头侧倾斜20 B .向头侧倾斜10D .向足侧倾斜10 E .向足侧倾斜20C.垂直入射胶片6 9 .舐

488、骨前后位摄影的中心线是:(B)A .垂直投射 B .向头侧倾斜15D .向头侧倾斜45 E .向足侧倾斜45C.向足侧倾斜15。7 0 .心脏左前斜位摄影,身体冠状位与胶片夹角为:(E)A. 15 20 B. 25 30C.,35 40D. 45 55 E. 55 657 1 .成人心脏摄影的焦片距为:(E)A . 防止对比剂流入膀胱 B . 压迫点为脐水平两侧C . 压迫球呈倒“ 八”字形放置 D . 压力为5. 3 8.0 kPaE . 观察全尿路时解除压迫A. 50 cmD. 120 cmB. 85 cmE. 200 cmC. 100 cm7 2 .关于头颅侧位摄影的叙述,错误的是:(

489、C )A . 可用于了解蝶鞍的大小和形态 B . 是头颅常规摄影位置C . 被检者取仰卧位D . 矢状面与床面平行E . 瞳间线与床面垂直7 3 .岩骨双4 5 轴位,又称为:( D )A . 劳氏位(Law s)B . 许氏位(Suhuller s)C . 低氏位(Runstrom s)D . 梅氏位(Mayer s)7 4 .乳突双1 5 侧位,亦称为:E. 斯氏位( Stenever s)( A )A . 劳氏位(Law s)B . 许氏位(Suhuller s)C . 低氏位(Runstrom s)D . 梅氏位(Mayer s)75. 下列组合中,正确的是:E. 斯氏位(Stenev

490、er s)( E )A . 视神经孔一 Rhees sB . 岩骨半轴位一ToWne sC . 鼻窦瓦氏位一Water sD . 鼻窦柯氏位一Caldweell7 6 .颈椎张口位摄影,中心线经:s E . 以上都是( A )A . 上颌切牙咬合面中点B . 下颌切牙咬合面中点C. 上颈磨牙咬合面中点D . 下颌磨牙咬合面中点 E . 上领尖牙咬台面中点7 7 .有关肾盂造影时压迫腹部的叙述,错误的是:( B )7 8 .解除因腹部加压引起迷走神经反应的最有效措施是:( D )A . 抗休克措施D . 立即解除压迫B . 注射阿托品E . 输液以加速对比剂的排泄C . 注射肾上腺素7 9 .阴

491、性对比剂的特点,错误的是:( D )A . 密度低B . 成本低C . 质量轻D . 原子序数高E. X 线易穿过80. 肾盂造影检查的禁忌证是:( E )A . 肾盂积水B . 肾结石C . 尿路狭窄D . 肾部肿瘤E . 肾功能严重损伤8 1 .用于胃肠道造影的对比剂是:( D )A . 硫化领B . 氯化领C . 碳酸钢D . 硫酸钢E . 氯化钳化8 2 .静脉肾盂造影中腹部压迫点,正确的是:( E )A . 脐水平两侧B . 第一腰椎水平两侧C . 耻骨联合上方3 cmD . 两侧能前上棘连线水平E . 脐下两侧,舐骨岬水平8 3 .下列选项,不属于膀胱造影的术前准备的是:( D )

492、A . 排尿B . 清洁肠道C . 备好导尿管D . 碘过敏试验 E . 备好注射用水和容器8 4 .下列属于无机碘对比剂的是:A . 泛影葡胺 B . 胆影钠C.碘化油(D)D . 碘化钠 E . 碘必乐8 5 .经肾脏排泄的非离子型对比剂是:A .胆影钠 B . 碘番酸C. 胆影葡胺( E)D . 泛影葡胺 E . 优维显8 6 .工作中常用的铛酸钙增感屏是:A . 超低速 B . 低速C.中速( C)D . 高速 E . 超高速87. 增感屏结构中反射层的作用是:A . 提高发光效率 B . 提高清晰度C.改善颗粒度( A)D . 控制量子斑点 E . 提高对比度8 8 .单纯型增感屏的

493、荧光体为:A . 铝酸钙 B . 硫氧化轧C. 硫氧化钮( A)D . 氟氯化钢 E . 浪氧化偶8 9 .利用碘化的闪烁体和非晶硅作为探测元的检测器是:A . 硒鼓检测器 B. IP成像转化器C.( D直接转化平板探测器)D . 间接转化平板探测器 E . 多丝正比室检测器90. DR使用的检测装置是:A. IP板 B . 影像增强器C.平板探测器( C)D . 电离室 E . 光电管91. DR系统与屏片系统相比,其优点不包括:A. X 线剂量低 B. X 线曝光宽,容度大C.( D.图像可进行后处理)D . 分辨率高 E . 图像可进行数字化存贮92. CR中可记录与读出X 线影像信息的

494、载体是:A .胶片 B . 磁盘C.IP板( C)D . 荧光屏 E . 平板探测器9 3 .用于读出C R 中X 线影像信息的光束是:A . 可见光 B . 红外线C.紫外线( D)D . 激光 E . 电子扫描线94. CR的调谐处理参数包括:A . 调谐类型 B . 旋转量C.旋转中心( E)D . 调谐移动量 E . 以上都是95. CR系统影像信息具有的动态范围是:A. 1: 10 B. 1: 100C.1: 1000( D)D. 1: 10 000 E. 1: 100 00096. CR经 X 线照射后在IP板上存留的是:A . 模拟信号 B . 数字信号C.黑臼影像( A)1 0

495、 9 .副鼻窦CT扫描参数选择,错误的是:D . 彩色影像 E . 电信号97. IP板的类型通常有:A . 标准型 B . 高分辨率型D . 多体层摄影型 E . 以上都是C.减影型(E)98. IP板的结构通常不包括:A . 保护层 B . 反射层D . 背衬层 E . 支持层C.荧光层(B)9 9 .决定DSA信号强度的主要因素是:A. X 线管电压 B. X 线量D .摄影部位 E . 血管内碘浓度C.曝光时间(E)100. MSCT的层厚最小可达:A. 5 mm B. 2 mmD 4 mm E. 0. 5 mmC.3 mm(E)1 0 1 .模拟量转换为数字量由模数转换器进行,称做:

496、A. B /D 转换 B. A /D 转换D. A /B 转换 E. A /C 转换C.D /B 转换(B)1 0 2 .下列技术中,不是CT扫描中常用的影像放大技术:A . 几何放大 B . 变域图像放大D . 内插图像放大 E . 外插图像放大C.宏观图像放大(E)1 0 3 .病人做CT增强扫描前的禁食时间为:A. 1 h B. 4 hD. 12 h E. 24 hC. 8 h(B)104. CT拍片时,通常需要放大照相的是:A . 图像模糊不清 B . 需重点观察、有病变的层面C . 图像分辨率不够 D . 图像的噪声较大E . 观察部位有异影重叠(B)1 0 5 .与图像显示上下位置

497、偏移有关的操作是:A . 病人摆位左右偏离 B . 扫描机架倾角过大D .床面进出调节有误 E . 扫描野选择有误C.(床面升降调节有误C)106. X 线管围绕人体腹背轴( 前后轴) 旋转的扫描方式为:A . 横断位扫描 B . 冠状位扫描D . 正位定位扫描 E . 侧位定位扫描C.矢状位扫描(B)1 0 7 .不属于颅脑平扫适应证的是:A .顿脑出血 B . 脑萎缩D . 脑血管畸形 E . 先天性无脑C.脑梗死(D)1 0 8 .不宜做眼及眶部CT扫描的是:A . 眼部外伤 B . 眼眶内异物D . 近视眼 E . 眼球及眶内肿物C.眼的先天性疾病(D)( E )A . 层厚5 cm

498、B . 层距5 cm C . 采用小FOVD . 观察薄骨有否破坏改用薄扫 E . 薄扫时需降低条件1 1 0 .喉部常规扫描,正确的屏气状态是:(C)A . 吸气后屏气扫描 B . 呼气后屏气扫描 C . 平静呼吸状态下扫描D . 发 “ 依”情况下扫描 E . 鼓气状态下扫描1 1 1 .人眼视觉的最大灰阶等级范围是:(B)A. 8 级 B. 16 级 C. 21 级D. 32 级 E. 64 级1 1 2 .将射线能量转换为可供记录的电信号的装置是:(B)A . 滤过器 B . 探测器 C . 准直器D. A /D 转换器 E . 显示器1 1 3 .下列哪种不是特殊CT造影增强法:(C

499、)A . 脑池造影CT B . 脑室造影CT C . 肝脏造影CTD . 关节造影CT E . 脊髓造影CT1 1 4 .蝶鞍CT的最佳扫描方式是:(A)A . 冠状扫描 B . 横断扫描 C . 矢状扫描D . 横断扫描+矢状扫描 E . 横断扫描多方位重建1 1 5 .水的CT值范围是:(D)A. 22 32 HU B. 46 66 HU C. 10-20 HUD. - 1 0 -0 HU E. 120-150 HU1 1 6 .实现人机对话的系统是:(C)A . 扫描系统 B . 图像处理系统 C . 视频显示系统D . 电视组件系统 E . 软盘系统117. CT扫描成像基本步骤中不包

500、括:(E)A. X 线产生 B . 采集数据 C . 重建图像D . 显示图像 E . 图像后处理1 1 8 .螺旋C T ,螺距为1 , 床速为10 m m /s ,层厚为:(C)A. 2. 5 mm B. 5 mm C. 10 mmD. 15 mm E. 20 mm1 1 9 .每日开机进行CT球管训练的目的是:(D)A . 升高球管温度 B . 启动旋转阳极 C . 测试球管好坏D . 保护X 线管 E . 增加阴极电子1 2 0 .不能在定位像( TOP像) 上确定的是:(E)A . 扫描起始线 B. 扫描终止线 C . 扫描范围D . 扫描角度 E . 扫描参数121. 肺 部 C

501、T图像拍摄时,对于窗的基本要求是:(E)A. 一般采用双窗拍摄 B . 只需拍摄肺窗 C . 必须要拍摄骨窗D . 必须包括肺的宽窗和窄窗各一套 E . 必须拍摄肺窗和纵隔软组织窗1 2 2 .在 CT扫描前需要大量饮水的检查部位是:(E)A . 肝胆D . 胰腺B . 肺E . 膀胱C . 肾脏123 . 肝、胆、胰检查口服稀释阳性对比剂的目的是: (A )A . 能使被观察部位与胃肠道区分开来 B . 观察肝胆消化液的分泌功能C . 观察肝胆内病变是否引起胃肠道阻塞 D . 保护胃黏膜不受辐射线的损伤E . 以上都是124 . 与普通CT相比,螺旋C T的缺点不包括: (E )A . 纵向

502、分辨率下降 B. X 线管损耗大 C. X 线剂量大D . 价格贵 E . 可回顾性地重建125 . 前列腺常用的扫描模式和层厚、层距是: (B )A . 高分辨率扫描模式,层厚层距各5 mm B . 软组织扫描模式,层厚层距各5 mmC . 标准扫描模式,层厚层距各5 mm D . 软组织扫描模式,层厚层距各10 mmE . 标准扫描模式,层厚层距各10 mm126. CT机的扫描方法有:(E )A . 低压滑环螺旋扫描B . 高压滑环螺旋扫描C . 球管旋转、探测器不动D . 旋转E . 以上都是127. C T的脊髓造影,其对比剂的注射方法是:(D )A . 动脉插管给药B . 静脉给药

503、C . 通过口服给药D . 经腰穿蛛网膜下隙给药E . 通过直肠灌注给药1 2 8 .常规CT扫描采取俯卧位的是:(D )A . 头颅轴扫B . 双膝轴扫C . 双踝轴扫D . 双腕轴扫E . 双雕轴扫129. CT平扫无法显示的是:(C )A . 软组织B . 骨骼C . 血掖D . 肾上腺E . 脑白质130. C T图像中从白到黑的灰度影像,称为:(C )A . 密度分辨率高B . 空间分辨率高C . 灰阶D . 窗宽窗位E . 噪声1 3 1 .图像未发现扫描层面外病灶, 原因是:(C )A . 密度分辨率高B . 空间分辨率高C . 无层面外结构干扰D . 灰阶范围小E . 矩阵大1

504、32. CT与常规X 线体层摄影比较,最主要的不同是:(B )A . 采用激光机拍照B . 无层面外组织结构干扰重叠C. X 线辐射剂量较小D . 可使用对比剂增加E . 病人摆位置较简便1 3 3 .关于部分容积效应的叙述,错误的是:( E )A . 高密度组织中的低密度病灶,测出的CT值偏高B . 低密度组织中的高密度病灶,测出的CT值偏低C. CT扫描中的部分容积效应是不可避免的D . 是同一层面中,不同密度的组织CTtfi均化现象E . 高档CT机,不存在部分容积效应1 3 4 .关于CT噪声的叙述,正确的是:( A )A . 噪声的大小与扫描层厚有关 B. CT的图像质量与噪声无关C

505、 . 噪声不受X 线照射剂量的影响 D . 噪声与激光胶片上的曝光量有关E . 噪声是一种外界干扰因素135. CT质控测试认为,理想的水模CT值应为:( D )A. 3 HU B. -1 000 HU C. +1 000 HUD. 0 HU E. 5 HU1 3 6 .影响空间分辨率的因素中,受人为因素制约的是:( D )A. X 射线扇形束的角度 B . 空间采样频率 C . 探测器孔径的大小D . 图像重建算法 E . 采样间距137. CT值为。时,其建立依据是:( D )A . 空气 B . 致密骨 C . 脂肪D . 水 E . 软组织1 3 8 .下列人体组织器官CT值低于0 的

506、是;( C )A . 肝脏 B . 脑白质 C . 肺D . 甲状腺 E . 肌肉1 3 9 .下列组合错误的是:( C )A . 模/ 数转换器模拟信号转换为数字信号B . 阵列处理器重建图像 C . 图像以数字形式存入硬盘D. X 线透过人体时衰减 E . 从球管射出的X 线首先通过准直器140. CT机空间分辨率的测量,常用的方法是:( C )A . 目测 B . 仿真人体体模 C . 分辨率体模D . 分辨率测试线对板 E . 星模卡1 4 1 .与 CT扫描伪影无关的是:( B )A . 扫描中病人的移动 B . 显示器故障 C . 探测器故障D . 体内有金属异物 E . 扫描层面

507、中有高密度骨出现1 4 2 .关于CT空间分辨率的解释,正确的是:( B )A . 分辨组织之间最小密度差别的能力 B . 高对比度时,区分相邻最小物体的能力C . 与 CT探测器孔径大小无关 D . 与图像重建的滤波函数无关E . 高于X 线屏/ 片摄影143 . 同一扫描层面,不同CT值的组织被平均计算,称为: (B )A . 射线能量均衡分配 B . 部分容积效应 C. CT值同化D. CT值的衰减 E . 体积同一性现象144 . 根据灰阶与CT值关系的叙述,正确的是: (B )A . 高 CT值部分被转换为黑色B . 高 CT值部分被转换为白色C. X 射线衰减越大,转换成灰阶后颜色

508、越深D . 图像中X 射线未被衰减,它将显示为白色图像E . 改变窗宽,也可改变被显示物体的CT值145. 使用CT增强扫描的主要目的是:( D )A . 可以减少扫描时毫安秒的用量B . 帮助确定病变的位置C . 提高病变组织的空间分辨率D . 增强物体的对比度和观察病变组织的血供E . 提高CT机的密度分辨率146. CT扫描中,边缘增强滤过的作用是: (A )A . 增强所显示图像组织结构的边缘 B . 只增强组织中血管的边缘C . 只增强病灶的边缘 D . 只增强高密度组织结构的进缘E . 低密度的组织结构都被去除147. C T中体素与像素区别的叙述,正确的是: (A )A . 体素

509、是三维的概念,像素是二维的概念B . 体素与像素的尺寸一致C . 体素只有模拟图像才有,像素属于数字图像D . 体素是图像重建过程中的产物E . 矩阵中的一个小方格,被称为体素148 . 为显示肝内高密度转移灶,窗宽的设置是: (A )A . 约在 100 250 HU B . 约在 0 350 HU C . 约在 250 450 HUD . 约在一 50 150 HU E . 约在 200 500 HU149 . 关于C T多方位和三维图像的叙述,正确的是: (B )A . 可直接用于影像诊断而不再需要横断面图像B . 需通过横断面图像的重组才能获得 C. 般都直接从原始扫描数据重建而成D

510、. 尚无任何诊断的参考价值 E . 其重组所需时间太长,无实用意义150. 扫描总剂量不变,C T的采样频率越高,结果是:C . 密度分辨率降低( C )A . 密度分辨率越高D . 空间分辨率降低B . 噪声越小E . 低对比度分辨率越高1 5 1 .在内耳CT扫描时,常选用:( C )A . 软组织算法B . 傅立叶图像重建算法C . 高分辨率算法D . 标准算法E . 低通滤过加权算法1 5 2 .将探测器接收的信息转换为数字信号的是:( C )A .信号放大器B . 探测器C. A /D 转换器D .信号比较器E . 后组探测器1 5 3 .表示空间分辨率的单位是:( E )A . 半

511、值全宽B . 对比度指数C . 百分线对数( LP%)D . 平方厘米线对数( LP/cm : ) E . 厘米线对数( LP/cm )154. CT扫描中常用的FOV是指:( D )A . 矩阵大小 B . 兴趣区 C . 灰阶标尺D . 扫描野 E . 激光胶片的分辨率155. CT扫描层厚的确定是通过改变X 线束的:A . 准直宽度 B . 硬度 C . 强度( A )D . 剂量E . 强弱1 5 6 .决定CT机连续工作时间长短的关键指标是:( C )A . 磁盘容量 B . 电源容量C. X 线管阳热容量D. X 线管焦点 E . 计算机性能157. CT成像设备即将发生的重大变化

512、是:( D )A . 全身螺旋CT B . 多排螺旋CTC.双排螺旋CTD . 平板探测器CT E . 大孔径CT158. CT扫描常见的伪影有:( D )A . 月牙状伪影 B . 卷褶伪影C.中心线伪影D . 放射状伪影 E . 以上全是1 5 9 .与梯度磁场强度变化无关的因素有:( D )A . 层厚 B . 扫描野C.空间分辨率D . 扫描时间 E . 磁共振信号强度1 6 0 .超导开放型MRI机优势的论述,错误的是:( E )A . 便于开展介人操作 B . 便于检查中观察病人C . 便于儿童及不合作病人的检查 D . 能使幽闭恐惧病人顺利进行MR检查E . 提供的固有磁场强度比

513、常导型低1 6 1 .组织磁化氢质子的取向是:(B )A . 杂乱无章 B . 顺磁场及逆磁场C.9 0 排列D. 30。排列 E . 顺磁场排列1 6 2 .生物组织中T?值约为:( B )A . 数千毫秒 B . 数十亳秒C.数百毫秒D . 数万毫秒 E . 数毫秒1 6 3 .在 MR成像装置中,共设计梯度磁场的数量有:( C )A. 1种 B. 2 种C. 3 种D. 4 种 E. 5 种1 6 4 .在层面选择、频率编码、相位编码梯度磁场之间:( D )A . 层面选择与频率编码间可以互换,相位编码不行B . 层面选择与相位编码间可以互换,相位编码不行C . 频率编码与相位编码间可以

514、互换,层面选择不行D. 3 种梯度磁场间可以互换E. 3 种梯度磁场间不可以互换1 6 5 .旋进的质子处于哪种状态时才产生回波:( B )A .相位离散 B . 相位重聚C.质子旋进D . 质子静息 E . 激励质子166. SE序列中180。脉冲的作用是:( E )A.使静磁化向量从z 轴转向x y 平面上作旋进运动,随之失相B . 使静磁化向量从XX平面转向Z 轴,随之失相c . 使静磁化向量从x 轴转向y 轴,随之相位重聚D.使静磁化向量从y 轴转向x 轴,随之相位重聚E . 相位重聚167. SE序列中,质子密度加权像是选用:( D )A . 短 TR短 TE所获得的图像 B . 短

515、 TR长 TE所获得的图像C . 长 TR长TE所获得的图像 D . 长 TR短 TE所获得的图像E . 以上均不是1 6 8 .在 1R序列中,如果某一组织的Ti值接近反转时间,则该组织在图像上表现为:A .信号增强 B . 信号较强 C . 信号抑制D . 中等信号 E . 无信号( C )1 6 9 .短时反转恢复( STIR) 序列常用于:A . 呼吸控制 B . 脂肪控制 C . 心电控制D . 抑制运动 E . 水抑制( B )1 7 0 .在快速自旋回波( TSE) 序列中:A . 回波链越长,扫描时间越长 B . 回波链越短,扫描时间越短C . 回波链越长,扫描时间越短 D .

516、 回波链可以无限制地长E . 扫描时间与回波链无关( C )1 7 1 .梯度回波( GRE) 与自旋回波( SE) 相比:A. GRE幅值较SE小,图像信噪比高 B. GRE幅值较SE小,图像信噪比低C. GRE幅位较SE大,图像信噪比高 D. GRE幅值较SE大,图像信噪比低E. GRE幅值较SE小,图像信噪比相同(A )1 7 2 .增加平均次数会导致:A . 延长扫描时间,提高信噪比 B . 延长扫描时间,降低信噪比C . 缩短扫描时间,增加信噪比 D . 缩短扫描时间,降低信噪比E . 以上均不是( A )173 . 为鉴别是否化学位移伪影可采用的方法是: (B )A . 变化频率编

517、码方向 B . 变化相位编码方向 C . 增大FOV ( 视察视野)D . 减小层厚 E . 全矩阵采集174 . 不属于磁共振伪影的是: (E )A . 化学位移伪影 B . 运动伪影 C . 卷褶伪影D . 截断伪影 E . 放射状伪影175 . 为消除部分容积效应的影响可采用: (D )A . 变化频率编码方向 B . 变化相位编码方向 C . 增大FOVD . 减小层厚 E . 全矩阵采集176 . 常规诊断剂量的Gd-DTPA,图像上所反映的主要为: (B )A. 1 延长 B. I 缩短 C. Tz延长D . 工缩短 E . 质子密度增加177 . 为使眼眶病变显示清楚,常在Ti和

518、 T?加权扫描时,利用的技术是: (C )A . 呼吸门控技术 B . 梯度运动相位重聚技术 C . 脂肪抑制技术D . 血流补偿技术 E. MRA技术178 . 与空间分辨率无关的因素是:A . 信噪比 B . 矩阵D . 数据读取次数 E . 磁场种类179 . 下列造影技术中,不属于MR水成像范畴的是:A. MR胰胆管造影 B. MR尿路造影D. MR泪道造影 E. MR腮腺管造影1 8 0 .在颈椎MR成像中,预饱和技术常用于抑制:A . 吞咽运动伪影 B . 心搏伪影D . 化学位移伪影 E . 逆向流动液体信号( E )C. FOV( C )C. MR血管造影( A )C . 呼吸运动伪影

展开阅读全文
相关资源
正为您匹配相似的精品文档
相关搜索

最新文档


当前位置:首页 > 建筑/环境 > 施工组织

电脑版 |金锄头文库版权所有
经营许可证:蜀ICP备13022795号 | 川公网安备 51140202000112号